PRINCE2 Practitioner Learner Workbook Digital

You might also like

Download as pdf or txt
Download as pdf or txt
You are on page 1of 179

PRINCE2® Practitioner

Learner
Workbook

Official Training Materials


Unlocking your
potential to Contents
achieve more
About this material 4

Module 1 Introduction 8

Module 2 Preparing for success 18

Module 3 Workshop Part 1 50


Module 4 Workshop Part 2 70
Module 5 Let’s review: Part 1 104
Welcome to the Learner The Official Training Materials for our
Workbook for the PRINCE2® courses enhance the learning experience Module 6 Workshop Part 3 112
Practitioner Official Training for both trainers and learners, enabling Module 7 Let’s review: Part 2 192
Materials. high quality and effective delivery of
training. PeopleCert products combine Module 8 Workshop Part 4
PeopleCert develops global best practice Other areas to explore 198
the knowledge and experience of
frameworks and certifications, manages
experts from around the world. Module 9 The PRINCE2 Practitioner sample
exams, and delivers certifications.
paper 204
Its product portfolio of more than Our courses use that knowledge to
700 certifications in IT & Digital transform learners from beginner Module 10 Exam preparation hints and tips 212
Transformation, Project Management, to practitioner, utilizing real-world Module 11 Course review 220
Business, and Languages includes ITIL® experience, product expertise, and years
and PRINCE2® - two of the most globally of training knowledge to bring the topic Appendix 1 – Handouts/activity sheets 232
recognised IP-protected frameworks, alive.
developed and evolved by the UK Activity Sheet 1 234
Consequently, learners will develop a
Government over a 30-year period.
solid foundation on which to develop Activity Sheet 2 238
their businesses and improve their
careers. Activity Sheet 3 242

Appendix 2 – Resources 246

PRINCE2® Candidate syllabus and index 248

Powering Sample papers 256

Best
Practice
Circumstances may dictate that your trainer decides to deliver the
modules or topics in a different order. These modules relate to the
PRINCE2 Practitioner syllabus, the PRINCE2 Practitioner Sample
Copyright © 2023 PeopleCert International Ltd. Paper 1, and the content of the Official Book.
All rights reserved. No part of this publication may be reproduced or transmitted in any form and by any means (electronic, photocopying, recording or otherwise) except as permitted in
writing by PeopleCert International Ltd. Enquiries for permission to reproduce, transmit or use for any purpose this material should be directed to the publisher. The PRINCE2 Practitioner certification builds upon information
Disclaimer
This publication is designed to provide helpful information to the reader. Although every care has been taken by PeopleCert International Ltd in the preparation of this publication, no taught during PRINCE2 Foundation. You should review the Official
Book and other relevant resources during the training course and
representation or warranty (express or implied) is given by PeopleCert International Ltd as publisher with respect as to the completeness, accuracy, reliability, suitability or availability
of the information contained within it and neither shall PeopleCert International Ltd be responsible or liable for any loss or damage whatsoever (indicatively but not limited to, special,
indirect, consequential) arising or resulting of virtue of information. instructions or advice contained within this publication.)
Copyright © 2023 PeopleCert International Ltd.
Second edition 2023
as you prepare for the examination.
About
this material
The Learner Workbook has been To help you identify what is core
designed to help you prepare for the content and what is extended content,
PRINCE2 Practitioner examination. It the module slide decks feature colour-
provides guidance and an overview coded borders. The slides that
of the course’s examinable content. feature a teal border are core
This Learner Workbook also includes content and those with a grey
questions/ quizzes, activities, and border are optional extended
sample papers with answers and content such as activities and
rationales. The Workbook is designed quizzes that will allow you
to reinforce learning and add value to test your knowledge.
your course experience. The Workbook
also includes valuable resources,
which will enhance your learning and
understanding of the material.

Do not forget to use the Official Book


during the course, as it provides
further information on the modules
explored in the Learner Workbook. The
Syllabus Index provided at the end of
the Workbook will help you identify the
sections of the Οfficial Βook referenced
throughout the material.
This course is for anyone who is preparing to sit the PRINCE2® Practitioner Information icons
PRINCE2® Practitioner certificate examination. The
goal is to provide you with a practical understanding
examination
of the format of the exam, build up your confidence
The PRINCE2 Practitioner examination is designed
and understanding, and ensure that you are able to
to assess whether you can demonstrate sufficient Objectives
analyse and apply PRINCE2 best practice in context.
application and analysis of the PRINCE2 method.
The PRINCE2 Foundation certification is a
prerequisite for the Practitioner course.
Aims of the course Timed activity
Upon completion of the course, you will be able to:

• demonstrate a wider appreciation of PRINCE2 Discuss/reflect/activity


skills and know-how, and apply this in a realistic
context
• tailor best practice in response to different project Think about/question and answer
circumstances
• efficiently navigate the PRINCE2 Official Book
Key learning points
• analyse information and reason whether a course
of action is effective/appropriate in accordance
with PRINCE2 best practice
Detailed PRINCE2 practice
• begin to apply PRINCE2 methodology to your own
projects
• explain the requirements of the PRINCE2 Review multiple-choice question
Practitioner exam.

The slides that feature a teal border are core content

The slides that feature a grey border are optional extended


content

PRINCE2® Practitioner | Copyright© PeopleCert International Ltd. 7


6 PRINCE2® Practitioner | Copyright© PeopleCert International Ltd.
Module 1
Introduction
Introduction

Introduction
Module 1

Module 1
Introductions

Welcome to the introductory module which gives you the opportunity to introduce Note that you will be able to refer to the Official Book during the exam, as it is an open
yourself. It will also provide key information on the PRINCE2 Practitioner course and book exam. One objective of the course is to help you to become adept at navigating the
exam. Official Book and applying information from the book during the exam.

Your course includes the PRINCE2 Official Book and your Learner Workbook. You will use
these throughout the training modules, as well as during exam preparation.

Notes Notes

10 PRINCE2® Practitioner | Copyright© PeopleCert International Ltd. PRINCE2® Practitioner | Copyright© PeopleCert International Ltd. 11
Introduction

Introduction
Module 1

Module 1
PRINCE2 Practitioner introduction

In this module, you will learn about how this course is designed and delivered, so that you will PRINCE2 is a globally-recognized project management method with over a million
know what to expect and what the key aspects and features of the course are. You will also professionals in 200 countries certified after passing the scenario-based examination.
get background information on the PRINCE2 Practitioner certification path and examination. PRINCE2 Practitioner certification is awarded based on a candidate’s ability to apply their
knowledge of PRINCE2 to a realistic scenario.
The examination format requires learners to demonstrate their understanding through
the ability to engage with the scenario and provide a rationale for their responses.
Tried and tested for 25 years, PRINCE2 has been continually developed and refined
through consultation with project management professionals to reflect their real-life
experiences. It helps organizations to understand how to incorporate the principles,
themes, and processes of PRINCE2 into their projects. It also provides a common
language that unites the project team with other stakeholders, regardless of language
and geographical location. Axelos, which is part of PeopleCert, owns the copyright and
has a range of PRINCE2 Official Books, such as:
• Managing Successful Projects with PRINCE2
• Effective Project Management: The PRINCE2 method (US version)
• Directing Successful Projects with PRINCE2
• PRINCE2 Agile®.
See axelos.com for more information.
Licensed trainers and consultants, Axelos’s Accredited training organizations (ATOs) offer
PRINCE2 training courses for all the certification levels or you can self-study.
PeopleCert is the IP owner of the Global Best Practice portfolio and administers the
exams.
PRINCE2: Foundation and Practitioner are the two main levels of the PRINCE2 project
management certification scheme. There is also the PRINCE2 Agile® certification
scheme which includes the PRINCE2 Agile Foundation and PRINCE2 Agile Practitioner
certifications.

Notes Notes

12 PRINCE2® Practitioner | Copyright© PeopleCert International Ltd. PRINCE2® Practitioner | Copyright© PeopleCert International Ltd. 13
Introduction

Introduction
Module 1

Module 1
The PRINCE2 Practitioner course and examination follow on from the PRINCE2 The PRINCE2 Practitioner certification is a fundamental step on the path to becoming a
Foundation level. The Foundation level course and exam focused on your ability to recall ProPath Project Expert, a path designed for project management professionals seeking
and understand essential PRINCE2 concepts. The Practitioner course requires you to a holistic view of projects, consistent results, and increasing efficiencies.
analyse realistic scenarios and to apply PRINCE2 concepts to those scenarios. As such,
The PRINCE2 Practitioner certification, alongside Management of Risk (MoR) and Managing
the course assumes that you have a strong grasp of the PRINCE2 Foundation concepts.
Successful Programmes 5th edition (MSP) certifications, combine to strengthen your
You will have the opportunity to assess your knowledge during Module 2: Preparing
ability to maximize benefits and manage risk, while planning, managing, and delivering
for success. If you do have gaps or need a refresh, the course includes optional brief
projects of any size or complexity.
reviews of the PRINCE2 Foundation information at relevant points. You may also benefit
from self-study of the Official Book to refresh your memory. There is a separate PRINCE2 course offered by Axelos: PRINCE2 Agile, which you can
take if you are interested in combining the PRINCE2 method with Agile ways of working.
Completing this certification would contribute to your certification as a ProPath Agile
Project Expert.
Further information about next steps regarding ProPath will be provided during the
course summary. For more information, visit axelos.com.

Notes Notes

14 PRINCE2® Practitioner | Copyright© PeopleCert International Ltd. PRINCE2® Practitioner | Copyright© PeopleCert International Ltd. 15
Introduction

Introduction
Module 1

Module 1
At the end of this course, you should be able to: The course is a four-part workshop designed to help you learn to prioritize and explore
areas of project management concern and interest in a flexible way. It includes activities,
• demonstrate a wider appreciation of PRINCE2 skills and know-how, and apply this in
quizzes, and a sample paper to help you to build up your ability to answer PRINCE2
a realistic context
Practitioner exam questions.
• tailor best practice in response to different project circumstances
• navigate the PRINCE2 Official Book efficiently.
• analyse scenarios and assess what courses of action would be effective and
appropriate in accordance with PRINCE2 best practice
• begin to apply PRINCE2 methodology to your own projects
• complete the PRINCE2 Practitioner examination.
The course is exam-focused and draws heavily on practical activities to prepare you for
the examination.

Here are the aims for the next module.

Notes Notes

16 PRINCE2® Practitioner | Copyright© PeopleCert International Ltd. PRINCE2® Practitioner | Copyright© PeopleCert International Ltd. 17
Module 2
Preparing for success
Preparing for success
Welcome to Module 2: Preparing for success.
Preparing for success

Preparing for success


Module 2

Module 2
The workshop portion of this course presents scenarios and asks you to apply your This course is designed to adapt to your exam preparation needs, to improve your
knowledge of PRINCE2 to choose effective and appropriate actions for those scenarios confidence and help ensure you are well-equipped to take the Practitioner exam. To
that are in accordance with PRINCE2 best practices. This module is about taking the time support this, the course includes interactive discussions and activities that feature group
to assess your knowledge of PRINCE2 Foundation concepts, as a basis for planning your work. This course builds upon the progress made during the Foundation course.
exam preparation. We will cover the format, terms, and conditions of the Practitioner
Note: Managing Successful Projects with PRINCE2, 6th edition is a required resource for the
exam, to give you a clear idea of what the exam will be like and allow you to plan
course and will be featured throughout. Please ensure that you have access to the text
accordingly. We will also have a look at the Scenario Booklet, so that you will learn how
during the course, for revision, and in the exam itself.
you will be applying your PRINCE2 knowledge to scenarios.
Suggestions for annotations that are permitted to remain in the Official Book during the
exam include:
• adding tabs or small bookmarks for key sections
• using highlighter pens to emphasize important paragraphs
• writing notes directly onto the pages of the Official Book.
Useful for exam preparation and study, but must be removed prior to the exam as this
is not permitted material to bring into the exam include:
• creating a cheat sheet of important chapter/page numbers
• sticking additional notes within the pages at relevant points
• creating an overview ‘one-pager’ for key sections and include this at the start of the
relevant chapter.

Notes Notes

20 PRINCE2® Practitioner | Copyright© PeopleCert International Ltd. PRINCE2® Practitioner | Copyright© PeopleCert International Ltd. 21
Preparing for success

Preparing for success


Module 2

Module 2
This is an informal self-evaluation. How confident are you in your understanding of Your trainer should take this opportunity to discuss the results of the class self-evaluation,
PRINCE2 principles, themes, and processes? Rate your confidence for each topic out of and outline the review sections of the course which will give you the opportunity to
5, with 1 as the lowest and 5 as the highest. You should provide the rating to your trainer refresh your memory and consolidate information.
confidentially and also make note of it in your Learner Workbook.

How confident are you in your current ability to take the PRINCE2 Practitioner exam? Rate
your confidence for each topic out of 5, with 1 as the lowest and 5 as the highest. You
should provide the rating to your trainer confidentially and also make note of it in your
Learner Workbook.

Notes Notes

22 PRINCE2® Practitioner | Copyright© PeopleCert International Ltd. PRINCE2® Practitioner | Copyright© PeopleCert International Ltd. 23
PRINCE2 Foundation Knowledge Check Quiz
This section of the module will test your PRINCE2 Foundation knowledge.

Questions:
1. PRINCE2 mandates that the project board represents the primary stakeholder interests.
Which principle does this follow?
Preparing for success

Preparing for success


A. Manage by stages
Module 2

Module 2
B. Focus on products
C. Defined roles and responsibilities
D. Learn from experience

2. Which theme establishes mechanisms to monitor and compare actual achievements


against those planned?
A. Progress
B. Plans
Here are the aims for the PRINCE2 Foundation Knowledge Check. You should keep a
note of your score in your Learner Workbook. C. Change
D. Quality

3. Which is a purpose of the business case theme?


A. To establish mechanisms for managing issues that may impact the baseline
B. To establish methods to judge whether the ongoing project is justified
C. To assess and control uncertain events or situations
D. To describe how products will be delivered that are fit for purpose

4. Which is a minimum requirement for applying the organization theme?


A. To ensure that the senior user verifies user requirements
B. To ensure that the change authority is delegated
C. To ensure that the supplier resources are available
D. To ensure that all the responsibilities of the project board are fulfilled

5. Which is a purpose of the quality theme?


A. To establish the mechanisms to judge whether a project is desirable and achievable
B. To look for ways to improve the effectiveness of the management of the project
C. To control uncertainty to improve the ability of the project to succeed
D. To establish mechanisms to control any unacceptable deviation
Notes

24 PRINCE2® Practitioner | Copyright© PeopleCert International Ltd. PRINCE2® Practitioner | Copyright© PeopleCert International Ltd. 25
6. Which is a purpose of the plans theme? 12. Which is a purpose of the starting up a project process?
A. To identify, assess, and control uncertainty within the project A. To understand the resources and costs to deliver the project’s products
B. To establish a coding system for all components of the project’s products B. To ensure that there is authority to deliver the project’s products
C. To define the means of delivering the products C. To do the minimum to decide whether it is worthwhile initiating the project
D. To produce a benefits management approach D. To create the management products required to control the project
Preparing for success

Preparing for success


7. Which is a minimum requirement for applying the risk theme? 13. Which is a purpose of the controlling a stage process?
Module 2

Module 2
A. To assign risk actioners to undertake planned risk responses A. To agree, perform, and deliver project work
B. To escalate identified threats to the project board B. To draft a plan for the next stage
C. To record identified threats in a risk register C. To agree tolerances for the stage
D. To establish a risk budget to fund responses to risks D. To take action so that the stage remains within tolerance

8. Is this statement about the minimum requirements for applying the change theme true or 14. Is the following statement, true or false?
false?
The managing product delivery process provides accurate progress information to the project
To be following PRINCE2, a project must, as a minimum, assess whether identified issues manager at an agreed frequency to ensure that expectations are managed.
might have a material impact on the business justification of the project (PRINCE2’s continued
A. True
business justification principle).
B. False
A. True
B. False
15. Which of the following is a step in PRINCE2’s recommended approach to planning?
A. Identify activities and dependencies
9. Which statement describes the directing a project process?
B. Describe the means of communication for the project
A. It enables the project board to assure that there is continued business justification
C. Implement responses for opportunities
B. It covers the day-to-day activities of the project manager
D. Baseline the project deliverables
C. It ensures that there are regular progress meetings
D. It begins on completion of the initiating a project process

10. What takes place during the closing a project process?


A. The post-project benefits reviews are performed
B. Ownership of the project’s products is transferred to the customer
C. An end stage report is prepared for the final stage
D. The project closure notification is reviewed and approved

11. Which is an objective of the managing a stage boundary process?


A. To request authorization to start the next stage
B. To ensure that all threats and opportunities for the current stage have been closed
C. To ensure that work on products allocated to the team for the next stage is authorized
D. To implement actions to resolve tolerance deviations from the stage plan

26 PRINCE2® Practitioner | Copyright© PeopleCert International Ltd. PRINCE2® Practitioner | Copyright© PeopleCert International Ltd. 27
Answer

1. PRINCE2 mandates that the project board represents the primary stakeholder interests. 6. Which is a purpose of the plans theme?
Which principle does this follow? A. To identify, assess, and control uncertainty within the project
A. Manage by stages B. To establish a coding system for all components of the project’s products
B. Focus on products C. To define the means of delivering the products
Preparing for success

Preparing for success


Module 2

Module 2
C. Defined roles and responsibilities D. To produce a benefits management approach
D. Learn from experience
7. Which is a minimum requirement for applying the risk theme?
2. Which theme establishes mechanisms to monitor and compare actual achievements A. To assign risk actioners to undertake planned risk responses
against those planned?
B. To escalate identified threats to the project board
A. Progress
C. To record identified threats in a risk register
B. Plans
D. To establish a risk budget to fund responses to risks
C. Change
D. Quality
8. Is this statement about the minimum requirements for applying the change theme true or
false?
3. Which is a purpose of the business case theme? To be following PRINCE2, a project must, as a minimum, assess whether identified issues
might have a material impact on the business justification of the project (PRINCE2’s continued
A. To establish mechanisms for managing issues that may impact the baseline
business justification principle).
B. To establish methods to judge whether the ongoing project is justified
A. True
C. To assess and control uncertain events or situations
B. False
D. To describe how products will be delivered that are fit for purpose

9. Which statement describes the directing a project process?


4. Which is a minimum requirement for applying the organization theme?
A. It enables the project board to assure that there is continued business
A. To ensure that the senior user verifies user requirements justification
B. To ensure that the change authority is delegated B. It covers the day-to-day activities of the project manager
C. To ensure that the supplier resources are available C. It ensures that there are regular progress meetings
D. To ensure that all the responsibilities of the project board are fulfilled D. It begins on completion of the initiating a project process

5. Which is a purpose of the quality theme? 10. What takes place during the closing a project process?
A. To establish the mechanisms to judge whether a project is desirable and achievable A. The post-project benefits reviews are performed
B. To look for ways to improve the effectiveness of the management of the project B. Ownership of the project’s products is transferred to the customer
C. To control uncertainty to improve the ability of the project to succeed C. An end stage report is prepared for the final stage
D. To establish mechanisms to control any unacceptable deviation D. The project closure notification is reviewed and approved

28 PRINCE2® Practitioner | Copyright© PeopleCert International Ltd. PRINCE2® Practitioner | Copyright© PeopleCert International Ltd. 29
11. Which is an objective of the managing a stage boundary process?
A. To request authorization to start the next stage
B. To ensure that all threats and opportunities for the current stage have been closed
C. To ensure that work on products allocated to the team for the next stage is
authorized
D. To implement actions to resolve tolerance deviations from the stage plan
Preparing for success

Preparing for success


Module 2

Module 2
12. Which is a purpose of the starting up a project process?
A. To understand the resources and costs to deliver the project’s products
B. To ensure that there is authority to deliver the project’s products
C. To do the minimum to decide whether it is worthwhile initiating the project
D. To create the management products required to control the project

Reflect on how you felt during the quiz. Are there any areas you need to focus on more
13. Which is a purpose of the controlling a stage process? than others?

A. To agree, perform, and deliver project work


B. To draft a plan for the next stage
C. To agree tolerances for the stage
D. To take action so that the stage remains within tolerance

14. Is the following statement, true or false?


The managing product delivery process provides accurate progress information to the project
manager at an agreed frequency to ensure that expectations are managed.
A. True
B. False

15. Which of the following is a step in PRINCE2’s recommended approach to planning?


A. Identify activities and dependencies
B. Describe the means of communication for the project
C. Implement responses for opportunities
D. Baseline the project deliverables
You should consider what the module has revealed about your confidence, understanding
of key concepts, and the areas of the course that can help strengthen your knowledge
and skills.

Notes

30 PRINCE2® Practitioner | Copyright© PeopleCert International Ltd. PRINCE2® Practitioner | Copyright© PeopleCert International Ltd. 31
Preparing for success

Preparing for success


Module 2

Module 2
The syllabus states: ‘The PRINCE2 Practitioner examination is intended to assess whether The PRINCE2 Practitioner exam is ‘open book’, so you can and should consult the
a candidate can apply and tailor the PRINCE2 project management method’. This course PRINCE2 Official Book, ‘Managing Successful Projects with PRINCE2’ during the exam.
will teach you to apply the method to a real project. Your project management expertise, You are allowed to make notes inside the book. However, no other material is permitted
the complexity of the project, and the support provided for the use of PRINCE2 in your other than the PRINCE2 Official Book.
work environment will all be factors that impact what you can achieve. It is also relevant
You have up to 2 hours 30 minutes to complete the exam. If you are taking the exam in
for other key staff involved in the design, development, and delivery of projects, including:
a language that is not your native or working language, you may be awarded 25% extra
Project Board Members (for example, Senior Responsible Owners), Team Managers (for
time, that is, 188 minutes in total.
example, Product Delivery Managers), Product Assurance (for example, Business Change
Analysts), Project Support (for example, Project and Programme Office Personnel), and The exam consists of 68 questions, each worth 1 mark. There is no negative marking. The
Operational Line Managers/staff. course design factors in the total number of marks available in the exam into activities
to help you to practise the timekeeping you will need to do during the exam. You must
answer at least 38 of the 68 questions correctly to pass the exam.
The questions ask you to apply your PRINCE2 knowledge to a situation, analyse the
information provided, and reason whether a course of action is effective, appropriate,
and in accordance with PRINCE2 best practices.
The exam format is scenario-based. You will be asked to read a realistic scenario, which
gives background information about a project that the questions explore. For at least
one question, you may need to use more information than will be provided about people
working on the project.
All questions start with a short scenario followed by a question and multiple choice.
Classic questions (worth one mark) present four options, each with a response and a
justification. Matching questions (worth three marks) present three pieces of information
and ask you to choose an answer for each from a list of five or six options.

Notes Notes

32 PRINCE2® Practitioner | Copyright© PeopleCert International Ltd. PRINCE2® Practitioner | Copyright© PeopleCert International Ltd. 33
Preparing for success

Preparing for success


Module 2

Module 2
Learning outcome 1 is to analyse the application of PRINCE2 principles in context. The first part of learning outcome 2 is to apply PRINCE2 requirements for applying each
theme, showing a knowledge of each theme’s elements.
A total of 8 marks are included for this outcome.
Business case Benefits management approach and business case, roles and
A summary of the principles is included in the slide for your reference.
responsibilities, outputs, outcomes, benefits, and dis-benefits.
Organization Communication management approach, project management team
structure.
Quality Product and project product description, quality management approach, quality
register, quality planning, control, and assurance.
Plans Project plan, stage plan, exception plan, team plan, and PRINCE2’s recommended
approach to planning, defining, and analysing the products.
Risk Risk management approach, risk register, recommended roles and responsibilities,
and risk management procedure.
Change Control approach, configuration item record, issue register and report, product
status account, issue and change control procedure.
Progress Checkpoint report, daily log, end project report, end stage report, exception
report, highlight report, lessons log, work package, tolerances, and raising exceptions.

Notes Notes

34 PRINCE2® Practitioner | Copyright© PeopleCert International Ltd. PRINCE2® Practitioner | Copyright© PeopleCert International Ltd. 35
Preparing for success

Preparing for success


Module 2

Module 2
Learning outcome 3 is to apply and tailor relevant aspects of PRINCE2 processes in
context. Learning outcome 3 is divided into two parts. The first part demonstrates an
understanding of the following points, in relation to each PRINCE2 process:
• the recommended associated actions
• recommended roles and responsibilities
• how the themes may be applied.

Notes Notes

36 PRINCE2® Practitioner | Copyright© PeopleCert International Ltd. PRINCE2® Practitioner | Copyright© PeopleCert International Ltd. 37
PRINCE2 Core Concepts Refresh Extension
Activity
Welcome to this general overview of the core concepts of PRINCE2.
Preparing for success

Preparing for success


Module 2

Module 2
For the final part of each module, you will look at a typical exam scenario. Refer to The PRINCE2®
Practitioner exam: Sample Paper 1 and 2 Scenario Booklet in the Learner Workbook.
• Instructions Note that Practitioner exam scenarios are not based on actual
organizations, and any similarities to known organizations are coincidental.
• Project scenario You should read through the scenario carefully.
• Stages You should refer to the stages table in the Scenario Booklet. (Note that This module provides an overview of the main topics featured in the Practitioner exam.
the Scenario Booklet for each exam will differ slightly in the format of information It is a good opportunity for you to begin to annotate your Official Book. Use small tabs
provided. In this case, a stage table has been included but it may not be consistent to mark locations in the Official Book, or another approach that you prefer to do this as
across all Practitioner exam scenarios. You should engage with the Scenario Booklet you work through the overview content. Do not use an approach that relies on loose-leaf
at the start of the exam, no matter the format.) paper notes, as you must remove notes on loose-leaf paper from the Official Book prior
• Additional information This section will be important for sections of the to the exam.
examination so you should take time to review it.

Notes Notes

38 PRINCE2® Practitioner | Copyright© PeopleCert International Ltd. PRINCE2® Practitioner | Copyright© PeopleCert International Ltd. 39
Preparing for success

Preparing for success


Module 2

Module 2
PRINCE2 defines a project as a temporary organization that is created for the purpose of A PRINCE2 project follows seven principles that are core obligations and good practices.
delivering one or more business products according to an agreed business case. Several The seven themes describe aspects of project management that must be addressed
characteristics distinguish projects from ‘business as usual’. throughout a project. The seven processes describe the stages of a project lifecycle. The
environment for a PRINCE2 project can be tailored to any type of project, no matter its
size, environment, complexity, importance, capability, or risk.

Change projects are the means by which change is introduced. Temporary projects
may be short or long in duration, but always have a defined start and a defined end.
Cross-functional projects involve a multi-skilled team of people working together
across functions. Each project will be unique in some way (different teams, customers,
locations, and so on). Projects are uncertain and always introduce risks and threats that
are not encountered during an organization’s usual business.

Notes Notes

40 PRINCE2® Practitioner | Copyright© PeopleCert International Ltd. PRINCE2® Practitioner | Copyright© PeopleCert International Ltd. 41
Preparing for success

Preparing for success


Module 2

Module 2
A project manager is responsible for planning the sequence of activities that are needed When it comes to customer and supplier roles, PRINCE2 assumes that there will be a
to complete a project. commercial relationship between the customer and the supplier. The customer will
specify the desired result and pay for the project, and the supplier will supply the project’s
Being a project manager implies that you will delegate some of the work to others. The
products. Projects may be wholly managed within the commissioning organization, or be
role is managing, not doing.
part of a commercial relationship.
When the project is underway, you must monitor how well the work in progress matches
the plan.
If the work is not going as planned, you must exert control to put things right. Control
also means spotting opportunities to (for example) speed things up or reduce costs. In
controlling the project, you must make a new plan then delegate, monitor, and so on.

Notes Notes

42 PRINCE2® Practitioner | Copyright© PeopleCert International Ltd. PRINCE2® Practitioner | Copyright© PeopleCert International Ltd. 43
Preparing for success

Preparing for success


Module 2

Module 2
Projects happen in the context of a portfolio of programmes, projects, and activities The seven core principles underpinning a PRINCE2 project:
that represent an organization’s total investment in the work to achieve its strategic
• Continued business justification There must be a justifiable reason for starting
objectives. A programme is an organizational structure created to direct and oversee
the project. The justification must remain valid throughout the project. The business
the implementation of a set of related projects and activities to achieve the strategic
justification is usually recorded in a business plan.
objectives. A project may be commissioned to deliver outputs that contribute to some of
• Learn from experience Lessons must be sought, recorded, and acted upon
the programme’s expected outcomes. Each is impacted by the programme’s approach
throughout the project. Everyone involved with the project must seek lessons learned
to governance and its reporting requirements. Projects can be standalone entities in an
rather than wait for someone else to provide them.
organization and lie outside the governance structures introduced by programmes or
portfolios, or an organization set up solely for the purpose of undertaking the project. • Defined roles and responsibilities Everyone must have agreed roles and
responsibilities, know what to do, and what to expect from others.
• Manage by stages A PRINCE2 project is planned, monitored, and controlled on a
stage-by-stage basis. These management stages are separated by decision points
made by the project board.
• Manage by exception A project defines tolerances for each objective. PRINCE2
lists six tolerances that can be set: time, cost, quality, scope, risk, and benefit. An issue
outside an agreed tolerance is called an exception. When an exception occurs, the
project manager sends an exception report to the project board who decide what to
do.
• Focus on products A PRINCE2 project focuses on the definition and delivery of
products; in particular, their quality requirements. Product descriptions provide the
means to determine effort, estimates, resource requirements, dependencies, and
activity schedules.
• Tailor to suit the project A PRINCE2 project must be tailored to suit the project’s
size, environment, complexity, importance, capability, and risk.

Notes Notes

44 PRINCE2® Practitioner | Copyright© PeopleCert International Ltd. PRINCE2® Practitioner | Copyright© PeopleCert International Ltd. 45
Preparing for success

Preparing for success


Module 2

Module 2
The PRINCE2 themes as they appear in the next layer of the diagram: This slide presents Figure 13.1 from Chapter 13 of the Official Book. It shows how each
process is applied during a project lifecycle. There are three management levels, which
• Business case The purpose of the business case is to enable the project manager
are delivering, managing, and directing. Each level has specific responsibilities and
and the project board to judge whether the project remains desirable and achievable.
activities to perform at the different stages.
• Organization This theme describes the roles and responsibilities that the PRINCE2
team requires to manage the project effectively. • Starting up a project The purpose is to gather sufficient information to decide if it
is worth investing in the detailed planning of the project.
• Quality The purpose is to enable each team member to understand the quality
attributes of the products and how these requirements will be delivered. • Initiating a project This involves establishing whether there is justification to
proceed with the project. At this stage, a business case is developed, and resources
• Plans PRINCE2 proceeds based on a series of plans. Each plan matches the needs
are committed by the project board.
of people at different levels. They are the focus of communication and control
throughout the project. • Controlling a stage This involves the monitoring and control activities performed
by the project manager. The project manager must ensure that each stage stays on
• Risk This theme addresses how the project manager deals with uncertainty.
course, and must react to unexpected issues. This process describes the day-to-day
• Change This theme helps you to identify, assess, and control change requests or management of the project.
issues that arise during the project.
• Managing a stage boundary This provides the project board with key decision
• Progress The progress theme demonstrates how to control the project and know points where they can decide whether to continue with the project or close it down.
where you are against the current plan, and determines whether and how the project
• Managing product delivery This ensures that the planned products are created
should proceed.
and delivered. Here, the project manager assesses work progress, ensures that
quality criteria are met, and obtains approval for completed products.
• Closing a project This covers the project manager’s work in completing the project
either at its end or at premature close. Here the project manager prepares information
for the project board so they can confirm that the project may close.
• Directing a project This runs from the project start-up until its closure. The project
board directs the project by means of reports at various decision points.
You should refer to the Slide Deck to see the full PRINCE2 journey animated.

Notes Notes

46 PRINCE2® Practitioner | Copyright© PeopleCert International Ltd. PRINCE2® Practitioner | Copyright© PeopleCert International Ltd. 47
Preparing for success

Preparing for success


Module 2

Module 2
This slide is a high-level view of the PRINCE2 journey. Observe how the processes work
together at the various stages to deliver a successful project.
You should refer to the Slide Deck to see the full PRINCE2 journey animated.

Consider each objective. You can also review your notes.

Notes Notes

48 PRINCE2® Practitioner | Copyright© PeopleCert International Ltd. PRINCE2® Practitioner | Copyright© PeopleCert International Ltd. 49
Module 3
The PRINCE2 Practitioner
workshop: Part 1
The PRINCE2 Practitioner workshop: Part 1
The PRINCE2 Practitioner

The PRINCE2 Practitioner


workshop: Part 1

workshop: Part 1
Module 3

Module 3
This module introduces you to the objectives for the workshop. You will also get to answer Tools for the workshop include the Scenario Booklet featured in the Learner Workbook.
some sample questions to familiarize yourself with the question formation and get some The Scenario Booklet will be relevant throughout the workshop modules as the activities
advice on how to choose your response. The focus will be on questions regarding the and exercises are sourced from it. Each exercise will be included in the relevant section
PRINCE2 principles and the organization theme. of the Learner Workbook, which has space for you to take notes. Collaboration, working
together, sharing ideas, being respectful of one another’s suggestions and contributions,
and feeling confident to share your own thoughts, will also be key.

Notes Notes

52 PRINCE2® Practitioner | Copyright© PeopleCert International Ltd. PRINCE2® Practitioner | Copyright© PeopleCert International Ltd. 53
The PRINCE2 Practitioner

The PRINCE2 Practitioner


Refer to Sample Paper 1 and 2 Scenario Booklet in your Learner Workbook to answer the Here are the aims for the next session.
workshop: Part 1

workshop: Part 1
Module 3

Module 3
questions on the slide.

This PRINCE2 Practitioner workshop menu will indicate which topics each workshop will
cover. You can navigate where you are in the workshop by seeing which titles are greyed
out. The PRINCE2 Practitioner workshop: Part 1 will cover the PRINCE2 principles and the
organization theme.

Notes Notes

54 PRINCE2® Practitioner | Copyright© PeopleCert International Ltd. PRINCE2® Practitioner | Copyright© PeopleCert International Ltd. 55
PRINCE2 Principles
Welcome to this session on the PRINCE2 principles.
The PRINCE2 Practitioner

The PRINCE2 Practitioner


workshop: Part 1

workshop: Part 1
Module 3

Module 3
This slide outlines the key syllabus topics that will be covered in this session. This is the first exercise of the workshop. Refer to Question 1 in Sample Paper 1 in the
Learner Workbook to view how the question is formatted in the paper.

Recall the seven principles and consider the diagram. Read the information on the slide and answer the question. You can review the relevant
sections in the PRINCE2 Official Book and use your findings to support your rationale.

Notes Notes

56 PRINCE2® Practitioner | Copyright© PeopleCert International Ltd. PRINCE2® Practitioner | Copyright© PeopleCert International Ltd. 57
Principles review
This is a quick refresher section on the PRINCE2 principles. Consider using this as an opportunity
to annotate the relevant part of the Official Book.
The PRINCE2 Practitioner

The PRINCE2 Practitioner


workshop: Part 1

workshop: Part 1
Module 3

Module 3
This is a quick refresher section on the PRINCE2 principles. Consider using this as an
opportunity to annotate the relevant part of the Official Book.

Notes Notes

58 PRINCE2® Practitioner | Copyright© PeopleCert International Ltd. PRINCE2® Practitioner | Copyright© PeopleCert International Ltd. 59
Organization
Welcome to this session on the organization theme. This topic is the subject of the second exercise in
the workshop.
The PRINCE2 Practitioner

The PRINCE2 Practitioner


workshop: Part 1

workshop: Part 1
Module 3

Module 3
This slide outlines the key syllabus topics that will be covered in this session. This is the second exercise in the workshop. It features a different approach where you
must ‘match’ a series of answers from a selection of options.

This is a recap of the organization theme. Consider each question. Refer to the Official Read the information on the slide and answer the question. Consider the rationale
Book for help. behind each answer.

Notes Notes

60 PRINCE2® Practitioner | Copyright© PeopleCert International Ltd. PRINCE2® Practitioner | Copyright© PeopleCert International Ltd. 61
The answer is D. Consider the rationale behind this answer. The answer is B. Consider the rationale behind this answer.
The PRINCE2 Practitioner

The PRINCE2 Practitioner


workshop: Part 1

workshop: Part 1
Module 3

Module 3
Read the information on the slide and answer the question. Consider the rationale Read the information on the slide and answer the question. Consider the rationale
behind each answer. behind each answer.

Notes Notes

62 PRINCE2® Practitioner | Copyright© PeopleCert International Ltd. PRINCE2® Practitioner | Copyright© PeopleCert International Ltd. 63
Organization review
This is a quick refresher section on the organization theme. Consider using this as an opportunity to
annotate the relevant part of the Official Book.
The PRINCE2 Practitioner

The PRINCE2 Practitioner


workshop: Part 1

workshop: Part 1
Module 3

Module 3
The answer is C. Consider the rationale behind this answer. All stakeholders must be considered and consulted when defining roles and
responsibilities.

Can you think of another way to say ‘stakeholder’?


Note: An initiative could be a programme, project, activity, or risk.

Notes Notes

64 PRINCE2® Practitioner | Copyright© PeopleCert International Ltd. PRINCE2® Practitioner | Copyright© PeopleCert International Ltd. 65
The PRINCE2 Practitioner

The PRINCE2 Practitioner


The main project interests are the business, user, and supplier. In PRINCE2, business, user, and supplier interests are brought together on the project
workshop: Part 1

workshop: Part 1
Module 3

Module 3
board, which is accountable for the project success. Project managers at the level
These interests must work together to make a project successful.
required to make decisions and commitments may be too busy to be involved with the
project on a daily basis. But projects need day-to-day management to succeed, so some
of this work can be delegated to the team manager. The project management structure
has four levels, three of which represent the project management team and a fourth that
sits outside the project. Figure 7.2 (page 60) illustrates these four levels of management.

Notes Notes

66 PRINCE2® Practitioner | Copyright© PeopleCert International Ltd. PRINCE2® Practitioner | Copyright© PeopleCert International Ltd. 67
The communication management approach will need to be reviewed and may need
The PRINCE2 Practitioner

to be adapted during the project. It should align with the programme’s approach to
stakeholder communication and engagement, although not all projects are part of a
workshop: Part 1

Module 3

programme.
The approach should be reviewed and updated regularly.

Notes

68 PRINCE2® Practitioner | Copyright© PeopleCert International Ltd.


Module 4
The PRINCE2 Practitioner
workshop: Part 2
The PRINCE2 Practitioner workshop: Part 2

In this module, you will work on timekeeping during the exam. We will review the PRINCE2 The main priority of the workshop is to introduce and apply time constraints, as working
processes and explore sample exam questions, this time focusing on questions about within time constraints will be a crucial skill for the exam.
starting up and directing a project, initiating project processes, and the business case
Duration You will have 2 hours and 30 minutes to complete the exam; however, those
theme. Lastly, you will practise responding to sample questions within a specific time
taking the exam in a language that is not their native or working language may be awarded
limit, and practise responding to a set of questions at once.
25% extra time, that is, 188 minutes in total.
Average time per question There are 68 questions in the exam, with one mark
The PRINCE2 Practitioner

The PRINCE2 Practitioner


for each. This equates to approximately two minutes per question with time to read
the scenario and review for those completing the exam in 2 hours and 30 minutes.
workshop: Part 2

workshop: Part 2
Module 4

Module 4
Candidates who are allowed extra time have approximately two and a half minutes per
question. However, within that, you must read the Scenario Booklet, engage with the
questions, and refer to the PRINCE2 Official Book. You must be familiar with the Official
Book and the nature of the questions.
Best practice The best way to prepare for the exam is by practising! Time constraints
will be introduced into the exercises from now on. Do not panic; the challenge will
increase gradually as the aim of these sessions is ensure you are prepared for the exam.

Notes Notes

72 PRINCE2® Practitioner | Copyright© PeopleCert International Ltd. PRINCE2® Practitioner | Copyright© PeopleCert International Ltd. 73
Starting up a project
Welcome to the ‘starting up a project’ session.

You have looked at the PRINCE2 principles and the organization theme in the first part of This slide outlines the key syllabus topics that will be covered in this session.
the workshop, so this module (Module 4: The PRINCE2 Practitioner workshop: Part 2) will
focus on some of the processes.
The PRINCE2 Practitioner

The PRINCE2 Practitioner


workshop: Part 2

workshop: Part 2
Module 4

Module 4
Here are the aims for the next session on PRINCE2 principles. This is a recap on the ‘starting up a project’ process. You may want to refer to the Official
Book to find the correct answers.

Notes Notes

74 PRINCE2® Practitioner | Copyright© PeopleCert International Ltd. PRINCE2® Practitioner | Copyright© PeopleCert International Ltd. 75
This is the first exercise in the workshop. You will need to respond to a matching question The answer is A. Consider the rationale behind this answer.
on starting up a project, and provide an explanation for your answer.
The PRINCE2 Practitioner

The PRINCE2 Practitioner


workshop: Part 2

workshop: Part 2
Module 4

Module 4
Read the information on the slide and answer the question. Consider the rationale Read the information on the slide and answer the question. Consider the rationale
behind each answer. behind each answer.

Notes Notes

76 PRINCE2® Practitioner | Copyright© PeopleCert International Ltd. PRINCE2® Practitioner | Copyright© PeopleCert International Ltd. 77
The answer is A. Consider the rationale behind this answer. The answer is F. Consider the rationale behind this answer.
The PRINCE2 Practitioner

The PRINCE2 Practitioner


workshop: Part 2

workshop: Part 2
Module 4

Module 4
Read the information on the slide and answer the question. Consider the rationale How did you feel about the previous exercise? Did you feel like you had enough time?
behind each answer. Did you feel under pressure? Did the time constraint change the way you approached
the exercise?

Notes Notes

78 PRINCE2® Practitioner | Copyright© PeopleCert International Ltd. PRINCE2® Practitioner | Copyright© PeopleCert International Ltd. 79
Directing a project and initiating a project
Welcome to the ‘directing a project’ and ‘initiating a project’ session.

This slide outlines the key syllabus topics that will be covered in this session. This is the second exercise in the workshop. You should spend 5 minutes answering the
two questions on the next slide.
The PRINCE2 Practitioner

The PRINCE2 Practitioner


workshop: Part 2

workshop: Part 2
Module 4

Module 4
This is a chance to recap your knowledge of the processes’ purpose and the main Read the information on the slide and answer the questions. You should locate key
activities. You should look up the processes in the Official Book. pieces of information in the Official Book to support your answers.

Notes Notes

80 PRINCE2® Practitioner | Copyright© PeopleCert International Ltd. PRINCE2® Practitioner | Copyright© PeopleCert International Ltd. 81
Business case
Welcome to the session on the business case theme.

This slide outlines the key syllabus topics that will be covered in this session.
The PRINCE2 Practitioner

The PRINCE2 Practitioner


workshop: Part 2

workshop: Part 2
Module 4

Module 4
This is a chance to recap your knowledge of the business case theme. You should consult
the Official Book.

Notes Notes

82 PRINCE2® Practitioner | Copyright© PeopleCert International Ltd. PRINCE2® Practitioner | Copyright© PeopleCert International Ltd. 83
This is the third exercise in the workshop, and where the challenge increases! You will Read the information on the slide and answer the question. Consider the rationale
need to answer 4 questions within 8 minutes, which is similar to the time you would behind each answer.
have in the exam. The types of questions are matching questions and multiple-choice
questions. You should refer to the Official Book to check key terms and information in
the question.
The PRINCE2 Practitioner

The PRINCE2 Practitioner


workshop: Part 2

workshop: Part 2
Module 4

Module 4
The answer is C. Consider the rationale behind this answer.

Notes Notes

84 PRINCE2® Practitioner | Copyright© PeopleCert International Ltd. PRINCE2® Practitioner | Copyright© PeopleCert International Ltd. 85
Read the information on the slide and answer the question. Consider the rationale Read the information on the slide and answer the question. Consider the rationale
behind each answer. behind each answer.
The PRINCE2 Practitioner

The PRINCE2 Practitioner


workshop: Part 2

workshop: Part 2
Module 4

Module 4
The answer is B. Consider the rationale behind this answer. The answer is F. Consider the rationale behind this answer.

Notes Notes

86 PRINCE2® Practitioner | Copyright© PeopleCert International Ltd. PRINCE2® Practitioner | Copyright© PeopleCert International Ltd. 87
Activity sheet 1: Questions and Rationales

Questions:
1. During the initiation stage, the vice president stated that attendance at launch events held
on Monday evenings is low, and on previous projects, this had resulted in lower album sales.
As a result, the launch event for this album will be held later in the week.
Which principle is being applied, and why?
A. ‘Continued business justification’, because there is sufficient reason to start this project.
B. ‘Continued business justification’, because the project’s justification should remain
unchanged
C. ‘Learn from experience’, because project teams should learn from what occurred on
similar projects
D. ‘Learn from experience’, because the project should continue to learn from its own
experiences

1. When starting a project, previous or similar projects should be


Read the information on the slide and answer the question. Consider the rationale reviewed to see if lessons can be applied. It is the responsibility of
behind each answer. everyone involved with the project to look for lessons rather than
wait for someone else to provide them. The project manager should
communicate with the events coordinator to find out more about the
timing of the previous event.

2. Towards the end of stage 2, the project manager realized that not all of the ‘recorded
The PRINCE2 Practitioner

The PRINCE2 Practitioner


sample songs’ would be completed before the end of the stage. The project manager decided
to move the remaining work to stage 3. This enabled the project manager to report that stage
workshop: Part 2

workshop: Part 2
Module 4

Module 4
2 was completed within time tolerance?
Is this an appropriate application of the ‘manage by stages’ principle, and why?
A. Yes, because moving the work to stage 3 avoided an exception situation in stage 2
B. Yes, because stage 3 is not the final stage, so work can be moved from stage 2
C. No, because the project board should assess project viability on completion of work
planned for stage 2
D. No, because work in stage 3 should start while work planned for stage 2 is being
completed

2. Application of the manage by stages principle provides review and


decision points for the project board at defined intervals, rather than
letting the project run on in an uncontrolled way. This is why the
planned work that remains in stage 2 cannot be moved to stage 3,
without the approval of an exception.

Notes

88 PRINCE2® Practitioner | Copyright© PeopleCert International Ltd. PRINCE2® Practitioner | Copyright© PeopleCert International Ltd. 89
3. As a result of previous lessons, this recommendation has been made: “If an executive is
appointed who does not have an understanding of PRINCE2, someone with experience of
Answers:
applying PRINCE2 should undertake business assurance”. The recording studio manager has 1. During the initiation stage, the vice president stated that attendance at launch events held
therefore been appointed as business assurance for stage 2. on Monday evenings is low, and on previous projects, this had resulted in lower album sales.
Is this appropriate, and why? As a result, the launch event for this album will be held later in the week.

A. Yes, because the recording studio manager is responsible for the delivery of the ‘recorded Which principle is being applied, and why?
album’ A. ‘Continued business justification’, because there is sufficient reason to start this project.
B. Yes, because the recording studio manager has the experience required to deliver the B. ‘Continued business justification’, because the project’s justification should remain
‘recorded album’ unchanged
C. No, because the recording studio manager’s business justification may conflict with the C. ‘Learn from experience’, because project teams should learn from what occurred
interests of the executive on similar projects
D. No, because the recording studio manager will not be available throughout the project D. ‘Learn from experience’, because the project should continue to learn from its own
lifecycle experiences

3. There may be conflict between customer and supplier business


2. Towards the end of stage 2, the project manager realized that not all of the ‘recorded
justification. The business assurance role should be undertaken by
someone from the customer organization as the recording studio sample songs’ would be completed before the end of the stage. The project manager decided
manager is a supplier. to move the remaining work to stage 3. This enabled the project manager to report that stage
2 was completed within time tolerance?
Is this an appropriate application of the ‘manage by stages’ principle, and why?
A. Yes, because moving the work to stage 3 avoided an exception situation in stage 2
The PRINCE2 Practitioner

The PRINCE2 Practitioner


B. Yes, because stage 3 is not the final stage, so work can be moved from stage 2
workshop: Part 2

workshop: Part 2
Module 4

Module 4
C. No, because the project board should assess project viability on completion of
work planned for stage 2
D. No, because work in stage 3 should start while work planned for stage 2 is being
completed

3. As a result of previous lessons, this recommendation has been made: “If an executive is
appointed who does not have an understanding of PRINCE2, someone with experience of
applying PRINCE2 should undertake business assurance”. The recording studio manager has
therefore been appointed as business assurance for stage 2.
Is this appropriate, and why?
A. Yes, because the recording studio manager is responsible for the delivery of the ‘recorded
album’
B. Yes, because the recording studio manager has the experience required to deliver the
‘recorded album’
C. No, because the recording studio manager’s business justification may conflict
with the interests of the executive
D. No, because the recording studio manager will not be available throughout the project
lifecycle

90 PRINCE2® Practitioner | Copyright© PeopleCert International Ltd. PRINCE2® Practitioner | Copyright© PeopleCert International Ltd. 91
PRINCE2 processes review Starting up a project process review
This section is useful if you require a more detailed review of the PRINCE2 processes. This section will refresh your memory on the ‘starting up a project’ process.

Figure 13.1 from Chapter 13 of the PRINCE2 Official Book shows how each process is This is an opportunity to consider the main points of starting up a project.
applied during a project lifecycle.
The lifecycle has three stages: initiation stage, subsequent stage(s), and final stage.
The three management levels are: delivering, managing, and directing. Each level has
specific responsibilities and activities to perform at the different stages:
• Starting up a project The purpose is to gather sufficient information to decide if it
is worth investing in the detailed planning of the project.
The PRINCE2 Practitioner

The PRINCE2 Practitioner


• Initiating a project This involves establishing whether there is justification to
workshop: Part 2

workshop: Part 2
Module 4

Module 4
proceed with the project. At this stage, a business case is developed, and resources
are committed by the project board.
• Controlling a stage This involves the monitoring and control activities performed
by the project manager. The project manager must ensure that each stage stays on
course and must react to unexpected issues. This process describes the day-to-day
management of the project.
• Managing a stage boundary This provides the project board with key decision
points where they can decide whether to continue with the project or close it down.
• Managing product delivery This ensures that the planned products are created
and delivered. Here, the project manager assesses work progress, ensures that
quality criteria are met, and obtains approval for completed products.
• Closing a project This covers the project manager’s work in completing the project
either at its end or at premature close. Here the project manager prepares information
for the project board so they can confirm that the project may close.
• Directing a project This runs from the project start-up until its closure. The project
board directs the project by means of reports at various decision points.
Simple projects have fewer stages than longer projects.

Notes Notes

92 PRINCE2® Practitioner | Copyright© PeopleCert International Ltd. PRINCE2® Practitioner | Copyright© PeopleCert International Ltd. 93
Project mandate This is the information that triggers the project. It could be from
market research or a customer request, for example. The project mandate should
provide the terms of reference for the project and should contain sufficient information
to identify, at least, the prospective executive of the project board. This will be developed
to produce the project brief.
Appoint the executive and project manager If the project is part of a programme,
the programme itself should provide the project brief and will appoint some members
of the project board. The responsibilities and time commitment of the role need to be
identified.
Prepare the outline business case The development of this requires regular and
The PRINCE2 Practitioner

The PRINCE2 Practitioner


frequent interaction and consultation between the project manager, the project board
members, and other stakeholders. It provides an agreed foundation for a more detailed
workshop: Part 2

workshop: Part 2
Module 4

Module 4
business case developed in the ‘initiating a project’ process.
Select the project approach and assemble the project brief The project brief is
produced in parallel to the business case. It also involves interaction with all stakeholders.
Previous lessons need to be considered and the project team need to be appointed.
Plan the initiation stage The project brief is refined and extended, then becomes the
project initiation document at this stage.

Notes Notes

94 PRINCE2® Practitioner | Copyright© PeopleCert International Ltd. PRINCE2® Practitioner | Copyright© PeopleCert International Ltd. 95
Directing a project and Initiating a project
review

The purpose of the ‘initiating a project’ process is to establish solid foundations for the
project, enabling the organization to understand the work that needs to be done to deliver
the project product before committing to a significant spend. The ‘initiating a project’
process helps ensure that everyone involved understands the cost and time commitment
Note the keywords shown here in bold. The project board is accountable for the project’s involved in a project. This allows stakeholders to assess the validity of the project.
success. Although they are accountable, they delegate the responsibility of the day-to-day
management of the project to the project manager. So, the project board is accountable,
and the project manager is responsible.
The PRINCE2 Practitioner

The PRINCE2 Practitioner


workshop: Part 2

workshop: Part 2
Module 4

Module 4
Here is an overview of directing a project. As you can see, directing a project sits on top
of all other processes.
This is an overview of initiating a project.

Notes Notes

96 PRINCE2® Practitioner | Copyright© PeopleCert International Ltd. PRINCE2® Practitioner | Copyright© PeopleCert International Ltd. 97
Business case review
This section will refresh your memory on the business case theme.

• Agree the tailoring requirements Define the tailoring as part of the project
initiation document and create initial project controls.
• Prepare the risk management approach Seek project board approval for the risk
management approach, although they may prefer to review it later as part of the product Note the keywords shown here in bold: a project must be desirable, viable, and achievable.
initiation document. Prepare the change control approach. Change control is essential Remember that project initiation is also mentioned in the viability of a project. The
for the project to maintain control over its management and specialist products. business case is at the centre of any impact assessment of risks, issues, and changes.
• Prepare the quality management approach A key success factor of any project Consider this: how will this risk, issue, or change affect the viability of the business case
is that it delivers what the user expects and finds acceptable. This will only happen and the business objectives and benefits being sought?
if these expectations are both stated and agreed at the beginning of the project,
together with the standards to be used and the means of assessing their achievement.
• Prepare the communication management approach The communication
The PRINCE2 Practitioner

The PRINCE2 Practitioner


management approach addresses both internal and external communications. It should
workshop: Part 2

workshop: Part 2
Module 4

Module 4
contain details of how the project management team will send information to, and receive
information from, the wider organization(s) involved with, or affected by, the project.
• Set up the project controls The level of control required by the project board
after initiation needs to be agreed and the mechanism for such controls needs to be
established, as does the level of control required by the project manager of the work
to be undertaken by team managers.
• Create the project plan The timescale and resource requirements must be
established and held in the project plan.
• Prepare the benefits management approach The outline business case
produced during starting up a project needs to be updated to reflect the estimated
time and costs.
• Assemble the project initiation documentation Tailoring should be included as
part of the project initiation document to create initial project controls.

Notes Notes

98 PRINCE2® Practitioner | Copyright© PeopleCert International Ltd. PRINCE2® Practitioner | Copyright© PeopleCert International Ltd. 99
The business case development path includes: Project outputs enable business changes, which can create desired outcomes, which
may also cause side effects and consequences which can result in dis-benefits. Desired
• developing the business case, getting appropriate decision-making information
outcomes can also cause side effects, consequences and dis-benefits. Desired outcomes
• verifying, or assessing if the project is (still) worthwhile
can be measured in benefits to help achieve one or more strategic objectives.
• maintaining and keeping the business justification updated with actual costs and
benefits, and with current forecasts for costs and benefits
• confirming and assessing whether the intended benefits have been (or will be)
realized. This will mostly take place post-project, although benefits may be realized
during the project.
The PRINCE2 Practitioner

The PRINCE2 Practitioner


workshop: Part 2

workshop: Part 2
Module 4

Module 4
Notes Notes

100 PRINCE2® Practitioner | Copyright© PeopleCert International Ltd. PRINCE2® Practitioner | Copyright© PeopleCert International Ltd. 101
There are minimum requirements for all PRINCE2 themes.
Below you can find the minimum requirements for applying the business case theme:
• create and maintain a business justification for the project, usually a business case
(PRINCE2’s continued business justification principle)
• review and update the business justification in response to decisions and events that
might impact desirability, viability, or achievability of the project (PRINCE2’s continued
business justification)
• define the management actions that will be put in place to ensure that the project’s
outcomes are achieved, and confirm that the project’s benefits are realized (PRINCE2’s
continued business justification principle)
The PRINCE2 Practitioner

• define and document the roles and responsibilities for the business case and benefits
workshop: Part 2

management (PRINCE2’s defined roles and responsibilities principle).


Module 4

Notes

102 PRINCE2® Practitioner | Copyright© PeopleCert International Ltd.


Module 5
Let’s review: Part 1
Let’s review: Part 1
Welcome to this review module of the progress you have made on the course so far.

The main aim of this review module is to reflect on the progress you have made so far This diagram reviews the topics you have completed so far and shows the progress you
and assess your understanding of PRINCE2. have made.
• Introduction The introductory module was a chance for you to introduce yourself
and become familiar with the course objectives.
• Preparing for success The first part of this module featured a self- evaluation activity
and a PRINCE2 Foundation Knowledge Check quiz. The second part of the ‘Preparing
for success’ module outlined the duration and nature of the exam, described the
learning objectives, and explained how the objectives are measured in the exam.
• Workshop parts 1 and 2 So far, the workshop sessions have been highly structured,
and you should have completed the exercises. The workshops introduce a new
approach where you are given more flexibility to focus on topics that you find more
complex or are of particular interest.
Let’s review: Part 1

Let’s review: Part 1


Module 5

Module 5
Notes Notes

106 PRINCE2® Practitioner | Copyright© PeopleCert International Ltd. PRINCE2® Practitioner | Copyright© PeopleCert International Ltd. 107
This slide recaps the exercises you have completed so far in the course. You have reached the halfway point of the course. Take this opportunity to reflect on
your understanding of the PRINCE2 principles, themes, and processes.
Workshop 1:
• Principles exercise A group discussion introducing the format of the exam question,
explaining the importance of the answer rationale, and analysing the explanations
provided in the answer booklet.
• Organization theme exercise You were asked to approach a matching question
and asked to explain the rationales for the answers you reached.
Workshop 2:
• Starting up a project exercise Time constraints were introduced. You were asked
to respond to a question within a set time, working alone.
• Directing a project and initiating a project exercise For this activity, you
responded to questions on two different processes within a 5-minute time frame.
• Business case exercise You worked independently to answer four questions within
an 8-minute time frame, which is the requisite pace for the exam itself. This is the time
constraint that you need to work to.
Let’s review: Part 1

Let’s review: Part 1


Module 5

Module 5
Take this opportunity to reflect on your understanding of the PRINCE2 principles, themes,
and processes.

Notes Notes

108 PRINCE2® Practitioner | Copyright© PeopleCert International Ltd. PRINCE2® Practitioner | Copyright© PeopleCert International Ltd. 109
Here are the aims for the next module.
Let’s review: Part 1

Let’s review: Part 1


Module 5

Module 5
Take some time to reflect.
Which topic area are you most interested in exploring in the next part of the workshop?
Is there a concept included within that topic area which you find particularly difficult? Was
the topic highlighted during the quiz or self-evaluations as requiring more focus?
Is that area more relevant to your interests/line of work?

Notes Notes

110 PRINCE2® Practitioner | Copyright© PeopleCert International Ltd. PRINCE2® Practitioner | Copyright© PeopleCert International Ltd. 111
Module 6
The PRINCE2®
Practitioner
workshop: Part 3
The PRINCE2® Practitioner workshop: Part 3

This module is about preparing your plan of action and priorities for the upcoming This part of the workshop is controlled by you. Think about the topics you would benefit
workshop. In it, we will explore the PRINCE2 syllabus topics and complete exercises and from revisiting this module. The topics that are in grey have already been visited during
activities that relate to sample questions. The aim of this module is to help you boost your parts 1 and 2 of the workshop.
understanding of PRINCE2 and your confidence in your preparation for the examination.
The PRINCE2 Practitioner

The PRINCE2 Practitioner


Notes Notes
workshop: Part 3

workshop: Part 3
Module 6

Module 6
114 PRINCE2® Practitioner | Copyright© PeopleCert International Ltd. PRINCE2® Practitioner | Copyright© PeopleCert International Ltd. 115
Managing a stage boundary
Welcome to the session on managing a stage boundary.

Take a moment to recall information about the managing a stage boundary process.
Locate and consider the overview diagram included as Figure 19.1, page 247 of the
Official Book.

This slide outlines the key syllabus topics that will be covered in this module.
The PRINCE2 Practitioner

The PRINCE2 Practitioner


Notes Notes
workshop: Part 3

workshop: Part 3
Module 6

Module 6
116 PRINCE2® Practitioner | Copyright© PeopleCert International Ltd. PRINCE2® Practitioner | Copyright© PeopleCert International Ltd. 117
Sample Paper 1, Question 63. Read the information on the slide and answer the question. Sample Paper 1, Question 64. Read the information on the slide and answer the question.
The PRINCE2 Practitioner

The PRINCE2 Practitioner


Notes Notes
workshop: Part 3

workshop: Part 3
Module 6

Module 6
118 PRINCE2® Practitioner | Copyright© PeopleCert International Ltd. PRINCE2® Practitioner | Copyright© PeopleCert International Ltd. 119
Managing a stage boundary process review
This is short review on the ‘managing a stage boundary’ process where you will be reminded of
the key points.

This is a Foundation-level course extension activity for creating a checklist. This slide outlines the purpose of managing a stage boundary. Note the keywords
(in bold), as this will help you to remember the purpose: to provide information to the
project board so they can review success, approve the next stage, review the updated
plan, and confirm that business justification still exists and that risks are still acceptable.

This is what your checklist should include.


The PRINCE2 Practitioner

The PRINCE2 Practitioner


Notes Notes
workshop: Part 3

workshop: Part 3
Module 6

Module 6
120 PRINCE2® Practitioner | Copyright© PeopleCert International Ltd. PRINCE2® Practitioner | Copyright© PeopleCert International Ltd. 121
The diagram on the slide provides further information on the objectives of managing a This diagram demonstrates three management stages and the connection between the
stage boundary: stage boundary, the stage end, and the exception plan.
• Assure project board that all products in the stage plan have been completed and There are two different entry points for a stage boundary:
approved.
• initiating a project
• Prepare stage plan for the next management stage. Review and update the project
• controlling a stage.
initiation documentation.
Managing a stage boundary At the end of each management stage, a decision needs
• Provide information for the project board to assess the continuing viability of the
to be made whether to continue the project. If necessary, the project can be redirected
project.
or stopped to avoid wasting time and money. This would lead to the implementation of
• Record information or lessons that can help later management stages of this an exception process.
project and/or other projects.
Stage end The results of a management stage are reported to the project board so that
• Request authorization to start the next management stage.
progress is clearly visible to the project management team. This activity should happen
as closely as possible to the actual end of a management stage.
For exceptions, you would also need to do the following: Exception plans Exception plans must be produced to show the actions required
Prepare an exception plan as directed by the project board. to recover from or avoid a deviation from the agreed tolerances in the project plan
or a stage plan. The exception plan usually follows an exception report. A stage plan
Seek approval to replace the project or stage plan for the current management stage exception covers the period from the present to the end of the current management
with the exception plan. stage. Exception plans are prepared to the same level of detail as the plan they replace.
If approved, the exception plan replaces the stage plan that is in exception and becomes
the new baselined plan.
The PRINCE2 Practitioner

The PRINCE2 Practitioner


Notes Notes
workshop: Part 3

workshop: Part 3
Module 6

Module 6
122 PRINCE2® Practitioner | Copyright© PeopleCert International Ltd. PRINCE2® Practitioner | Copyright© PeopleCert International Ltd. 123
Closing a project
Welcome to this session on closing a project.

Purpose To check that the project has reached its objectives and that the products This slide outlines the key syllabus topics that will be covered in this session.
have been accepted.
Reasons for closing:
• acceptance confirmed
• objectives achieved.
Objectives:
• verify user acceptance
• all open issues and risks addressed
• ensure host site can support the products
• assess any benefits
• review the performance of the project.
Consider the ‘closing a project process’ diagram (Figure 20.1, page 261 of the Official
Book).
The PRINCE2 Practitioner

The PRINCE2 Practitioner


Notes Notes
workshop: Part 3

workshop: Part 3
Module 6

Module 6
124 PRINCE2® Practitioner | Copyright© PeopleCert International Ltd. PRINCE2® Practitioner | Copyright© PeopleCert International Ltd. 125
Sample Paper 1, Question 66. Read the information on the slide and answer the question. This is a Foundation-level course extension activity for how to prepare for planned
closures.

Sample Paper 1, Question 67. Read the information on the slide and answer the question.
The PRINCE2 Practitioner

The PRINCE2 Practitioner


Notes Notes
workshop: Part 3

workshop: Part 3
Module 6

Module 6
126 PRINCE2® Practitioner | Copyright© PeopleCert International Ltd. PRINCE2® Practitioner | Copyright© PeopleCert International Ltd. 127
Closing a project process review
This is a short review on the ‘closing a project’ process to remind you of the key points.

This slide outlines the purpose of the ‘closing a project’ process. Note the keywords
(in bold) as they will help you recall the purpose.
The fixed points for closing could be:
• Acceptance confirmed by, for instance, referring to the acceptance criteria or
gaining acceptance directly from the customer
• Objectives achieved from the business case. Note: The acceptance criteria is a
prioritized list of criteria that the project product must meet before the customer will
accept it.

This is what your answer should include.


The PRINCE2 Practitioner

The PRINCE2 Practitioner


Notes Notes
workshop: Part 3

workshop: Part 3
Module 6

Module 6
128 PRINCE2® Practitioner | Copyright© PeopleCert International Ltd. PRINCE2® Practitioner | Copyright© PeopleCert International Ltd. 129
The objectives of closing a project should: The diagram is an overview of the ‘closing a project’ process, showing the two uses (and
two entry points).
• verify user acceptance of the project product
• ensure that the host site is able to support the products when the project is One of the defining features of a PRINCE2 project is that it is finite: it has a start and an end.
disbanded Closure activities should be planned as part of the stage plan for the final management
stage. A clear end to a project is always more successful.
• review the performance of the project against its baselines
• assess any benefits that have been realized and update the benefits management Planned close When closing a project (if the project has achieved its objectives or
approach to include any post-project benefit reviews acceptance has been confirmed), work is required to prepare input to the project board
in order to obtain its authorization to close the project. Afterwards, the executive should
• Ensure that provision has been made to address all open issues and risks, with
also notify corporate, programme management, or the customer that the project has
follow-on action recommendations.
closed (see Section 15.4.5 of the PRINCE2 Official Book).
Premature close It is also possible that the project board may initiate a premature
closure of the project under some circumstances (for example if the business case is no
longer valid). If the project is being brought to a premature close, this process will still
need to be executed but may have to be tailored to the actual project situation.
Note: In both, approval must be sought before releasing resources.

This figure provides an overview of closing a project.


The PRINCE2 Practitioner

The PRINCE2 Practitioner


Notes Notes
workshop: Part 3

workshop: Part 3
Module 6

Module 6
130 PRINCE2® Practitioner | Copyright© PeopleCert International Ltd. PRINCE2® Practitioner | Copyright© PeopleCert International Ltd. 131
Managing product delivery
Welcome to this session on managing product delivery.

Purpose To manage and control the work between the project manager and the This slide outlines the key syllabus topics that will be covered in this module.
team manager by placing certain formal requirements on the accepting, executing, and
delivery of products.
Objectives:
• Work on products allocated to the team is authorized and agreed.
• Team managers, team members, and suppliers are clear as to what is to be produced
and what is the expected effort, cost, or timescales.
• Planned products are delivered to expectations and within tolerance.
• Accurate progress information is provided to the project manager at an agreed
frequency to ensure that expectations are managed.
Define each step of the process:
• Accepting a work package from the project manager and creating a team plan to
manage product development. This is an agreement between the project and team
manager as to what is to be delivered.
• Executing a work package. The team produces products, requests quality checks,
and monitors it to the requirements defined in the authorized work package. Also,
the team obtains approval, and reports to the project manager using the checkpoint
report.
• Delivering a work package. This is proof that the products are complete. Encourage
learners to recap this information in the Official Book, in Section 18.4, page 237 (and
the overview diagram Figure 18.1, page 237).
The PRINCE2 Practitioner

The PRINCE2 Practitioner


Notes Notes
workshop: Part 3

workshop: Part 3
Module 6

Module 6
132 PRINCE2® Practitioner | Copyright© PeopleCert International Ltd. PRINCE2® Practitioner | Copyright© PeopleCert International Ltd. 133
Sample Paper 1, Question 60. Read the information on the slide and answer the question.

Sample Paper 1, Question 60. Read the information on the slide and answer the question.
The PRINCE2 Practitioner

The PRINCE2 Practitioner


Notes Notes
workshop: Part 3

workshop: Part 3
Module 6

Module 6
134 PRINCE2® Practitioner | Copyright© PeopleCert International Ltd. PRINCE2® Practitioner | Copyright© PeopleCert International Ltd. 135
Managing product delivery process review
Welcome to the review for the managing product delivery process.

Take careful note of the key terms highlighted here in bold as they will help you recall this The objective of the ‘managing product delivery’ process is to ensure that all the actions
definition. Managing product delivery includes managing the acceptance of products, shown here happen. The work done must be work that is authorized and agreed. The
the execution of tasks associated with product delivery, and finally, the management deliverable to be produced, and the expected effort, cost, and timescales must be clear
of the delivery of products. This session will cover accepting, executing, and delivery of to everyone involved. These things take management to ensure they happen. And so
products. does ensuring products are delivered to expectations and within agreed tolerances, and
ensuring that progress information is delivered and is accurate, and that expectations are
managed.
The PRINCE2 Practitioner

The PRINCE2 Practitioner


Notes Notes
workshop: Part 3

workshop: Part 3
Module 6

Module 6
136 PRINCE2® Practitioner | Copyright© PeopleCert International Ltd. PRINCE2® Practitioner | Copyright© PeopleCert International Ltd. 137
Change
Welcome to this session on change.

Managing product delivery activities: Purpose To identify, assess, and control any potential and approved changes (issues).
Accept a work package The team manager accepts a work package from the project Three types of issues Request for change, off-specification, problem/concern.
manager and creates a team plan to manage development of the products. This is an
Minimum requirements:
agreement between the project manager and the team manager as to what is to be
delivered. • define an approach

Execute a work package The team produces products, requests quality checks • define baselines
(and monitors it to the requirements defined in the authorized work package), obtains • maintain some form of issue register to record identified issues and decisions
approval, and reports to the project manager using the checkpoint report. • use lessons to identify and manage issues.
Deliver a work package This is delivering proof that the products are complete.
The PRINCE2 Practitioner

The PRINCE2 Practitioner


Notes Notes
workshop: Part 3

workshop: Part 3
Module 6

Module 6
138 PRINCE2® Practitioner | Copyright© PeopleCert International Ltd. PRINCE2® Practitioner | Copyright© PeopleCert International Ltd. 139
This slide outlines the key syllabus topics that will be covered in this module. Sample Paper 1, Question 34, 35, and 36. Read the information on the slide and answer
the question. For this exercise, you will need to review information about the change
theme roles and responsibilities (Section 11.2.1, page 141 of the Official Book).

The answer is C. Consider the rationale behind this answer.


The PRINCE2 Practitioner

The PRINCE2 Practitioner


Notes Notes
workshop: Part 3

workshop: Part 3
Module 6

Module 6
140 PRINCE2® Practitioner | Copyright© PeopleCert International Ltd. PRINCE2® Practitioner | Copyright© PeopleCert International Ltd. 141
Sample Paper 1, Question 34, 35, and 36. Read the information on the slide and answer Sample Paper 1, Question 34, 35, and 36. Read the information on the slide and answer
the question. For this exercise, you will need to review information about the change the question. For this exercise, you will need to review information about the change
theme roles and responsibilities (Section 11.2.1, page 141 of the Official Book). theme roles and responsibilities (Section 11.2.1, page 141 of the Official Book).

The answer is B. Consider the rationale behind this answer. The answer is D. Consider the rationale behind this answer.
The PRINCE2 Practitioner

The PRINCE2 Practitioner


Notes Notes
workshop: Part 3

workshop: Part 3
Module 6

Module 6
142 PRINCE2® Practitioner | Copyright© PeopleCert International Ltd. PRINCE2® Practitioner | Copyright© PeopleCert International Ltd. 143
Sample Paper 1, Question 37. Read the information on the slide and answer the This is a Foundation-level course extension activity for how to identify types of issues.
question. You will need to review the Section of the Official Book called ‘Deciding on
corrective actions’ (Ref. 11.4.4, page 145) to respond correctly to this question. Check the
answer information included in Table 11.3 on page 146.

The answer is C. Consider the rationale behind this answer. This is what your answer should include.
The PRINCE2 Practitioner

The PRINCE2 Practitioner


Notes Notes
workshop: Part 3

workshop: Part 3
Module 6

Module 6
144 PRINCE2® Practitioner | Copyright© PeopleCert International Ltd. PRINCE2® Practitioner | Copyright© PeopleCert International Ltd. 145
Change review
This is a short review on the change theme to remind you of the key points.

Take note of the keywords (in bold) which will help you to remember the purpose of the Types of issues include:
theme.
Request for change This is a proposal for a change to a baseline. Example: The senior
user would like to increase the capacity of a product from 100 to 150 users.
Off-specification Something that should be provided by the project, but currently is
not (or is forecast not to be). It might be a missing product or a product not meeting its
specifications. Example: Advice from a supplier that they can no longer deliver one of the
products specified by the customer.
Problem/concern Any other issue that the project manager needs to resolve or
escalate. Example: Advice from a team manager that one of the suppliers has gone
bankrupt, resulting in the need to identify and engage a new supplier.
The PRINCE2 Practitioner

The PRINCE2 Practitioner


Notes Notes
workshop: Part 3

workshop: Part 3
Module 6

Module 6
146 PRINCE2® Practitioner | Copyright© PeopleCert International Ltd. PRINCE2® Practitioner | Copyright© PeopleCert International Ltd. 147
Plans
Welcome to this session on plans.

Define an approach This approach must minimally cover: Recall the key information regarding the plans theme. You can consult the Official Book
to refresh your memory.
• how issues are identified and managed
• an assessment of whether identified issues might have a material impact on the
business justification of the project (PRINCE2’s continued business justification
principle)
• a justified reason for starting the project
• a documented and approved reason
• a justification that is assessed and remains valid throughout the project
• roles and responsibilities for change control (PRINCE2’s defined roles and
responsibilities principle), include a defined change authority.
Define baselines How baselines are created, maintained, and controlled. A baseline is
a reference level against which an entity is monitored and controlled.
Use an issue register Maintain some form of issue register to record identified issues
and decisions relating to their analysis, management, and review.
Ensure that issues are captured, examined, managed, and reviewed throughout the
project lifecycle.
Refer to lessons Use lessons to inform issue identification and management (PRINCE2’s
‘learn from experience’ principle).

This slide outlines the key syllabus topics that will be covered in this module.
The PRINCE2 Practitioner

The PRINCE2 Practitioner


Notes Notes
workshop: Part 3

workshop: Part 3
Module 6

Module 6
148 PRINCE2® Practitioner | Copyright© PeopleCert International Ltd. PRINCE2® Practitioner | Copyright© PeopleCert International Ltd. 149
Sample Paper 1, Question 27. Read the information on the slide and answer the Sample Paper 1, Question 28. This question relates to preparing estimates, which is
question. For this exercise, you will need to review the information regarding the way featured in Section 9.3.1.4, page 110 of the Official Book.
PRINCE2’s plans relate to each other, which is included as a diagram on page 99 of the
Official Book (Figure 9.1).
The PRINCE2 Practitioner

The PRINCE2 Practitioner


Notes Notes
workshop: Part 3

workshop: Part 3
Module 6

Module 6
150 PRINCE2® Practitioner | Copyright© PeopleCert International Ltd. PRINCE2® Practitioner | Copyright© PeopleCert International Ltd. 151
Your trainer should list the criteria to include in the project product description to refer
to. You should refer to the Official Book throughout the exercises to add notes, identify
the correct answer, and validate your rationales.

This is a Foundation-level course extension activity for how to create a project product
description. You have 20 minutes to complete this exercise. Refer to the example of a
product description included in the Official Book on page 351 (Table D.1). You should
complete a similar breakdown project product description for the music album project
(refer to the Scenario Booklet in your Learner Workbook for more information). You
should refer to the Official Book throughout the exercises to add notes, identify the
correct answer, and validate your rationales.
The PRINCE2 Practitioner

The PRINCE2 Practitioner


Notes Notes
workshop: Part 3

workshop: Part 3
Module 6

Module 6
152 PRINCE2® Practitioner | Copyright© PeopleCert International Ltd. PRINCE2® Practitioner | Copyright© PeopleCert International Ltd. 153
Plans review
Welcome to a short review on the plans theme.

Project plan Created during the ‘initiating a project’ process and updated during the
‘managing a stage boundary’ process.
Initiation stage plan Created during the ‘starting up a project’ process. All subsequent
stage plans are produced near the end of the current management stage when preparing
for the next management stage.
Team plan An optional level of plan used as the basis for team management control
when executing work packages.
If approved, the exception plan replaces the plan that is in exception and becomes the
new baselined plan.
The PRINCE2 Practitioner

The PRINCE2 Practitioner


Notes Notes
workshop: Part 3

workshop: Part 3
Module 6

Module 6
154 PRINCE2® Practitioner | Copyright© PeopleCert International Ltd. PRINCE2® Practitioner | Copyright© PeopleCert International Ltd. 155
Progress
Welcome to the session on the progress theme. This will serve as a refresher on this topic.

How much can you remember? Feel free to refer to the Official Book here. This slide outlines the key syllabus topics that will be covered in this module.
The PRINCE2 Practitioner

The PRINCE2 Practitioner


Notes Notes
workshop: Part 3

workshop: Part 3
Module 6

Module 6
156 PRINCE2® Practitioner | Copyright© PeopleCert International Ltd. PRINCE2® Practitioner | Copyright© PeopleCert International Ltd. 157
Sample Paper 1, Question 39, 40, and 41. Read the information on the slide and answer Sample Paper 1, Question 39, 40, and 41. Read the information on the slide and answer
the question. This question is a matching exercise and draws upon information about the question. This question is a matching exercise and draws upon information about
the management products, which is included within Sections 12.2.2.1 and 12.2.2.4 the management products, which is included within Sections 12.2.2.1 and 12.2.2.4 pages
pages150–153 of the Official Book. 150–153 of the Official Book.

The answer is A. Consider the rationale behind this answer. The answer is B. Consider the rationale behind this answer.
The PRINCE2 Practitioner

The PRINCE2 Practitioner


Notes Notes
workshop: Part 3

workshop: Part 3
Module 6

Module 6
158 PRINCE2® Practitioner | Copyright© PeopleCert International Ltd. PRINCE2® Practitioner | Copyright© PeopleCert International Ltd. 159
Sample Paper 1, Question 39, 40, and 41. Read the information on the slide and answer Sample Paper 1, Question 42. Read the information on the slide and answer the
the question. This question is a matching exercise and draws upon information about question. This question requires you to apply information about checkpoint reports,
the management products, which is included within Sections 12.2.2.1 and 12.2.2.4 which can be found in Appendix A, A.4, page 298 of the Official Book.
pages150–153 of the Official Book.

The answer is D. Consider the rationale behind this answer.


The PRINCE2 Practitioner

The PRINCE2 Practitioner


Notes Notes
workshop: Part 3

workshop: Part 3
Module 6

Module 6
160 PRINCE2® Practitioner | Copyright© PeopleCert International Ltd. PRINCE2® Practitioner | Copyright© PeopleCert International Ltd. 161
Progress review
This is an opportunity to review the key points of the progress theme.

This is a Foundation-level course extension activity revolving around documents. This To monitor and compare actual achievements against those planned Establish
activity should take 30 minutes. You should prepare an explanation of the document you mechanisms to monitor and compare actual achievements against planned achievements.
are assigned by your trainer.
To provide a forecast This is for the project’s objectives and continued viability. Of
PRINCE2’s seven principles, ‘manage by exception’ is particularly important to the progress
theme. An exception is a situation where a deviation is beyond agreed tolerance levels.
To control unacceptable deviations Tolerances are the permissible deviation above
and below a plan’s target for cost and time without escalating the deviation to the next
level of management. There may also be tolerances for quality, scope, benefits, and risk.
The PRINCE2 Practitioner

The PRINCE2 Practitioner


Notes Notes
workshop: Part 3

workshop: Part 3
Module 6

Module 6
162 PRINCE2® Practitioner | Copyright© PeopleCert International Ltd. PRINCE2® Practitioner | Copyright© PeopleCert International Ltd. 163
Quality
Welcome to the session for the quality theme.

Define an approach for controlling progress in the PID (project initiation document).
Manage by stages PRINCE2’s ‘manage by stages’ principle.
Set tolerances and be managed by exception against them PRINCE2’s ‘manage by
exception’ principle.
Review the business justification when exceptions are raised PRINCE2’s
‘continued business justification’ principle.
Learn lessons PRINCE2’s ‘learn from experience’ principle.

This slide outlines the key syllabus topics that will be covered in this module.
The PRINCE2 Practitioner

The PRINCE2 Practitioner


Notes Notes
workshop: Part 3

workshop: Part 3
Module 6

Module 6
164 PRINCE2® Practitioner | Copyright© PeopleCert International Ltd. PRINCE2® Practitioner | Copyright© PeopleCert International Ltd. 165
The answer is B. Consider the rationale behind this answer.

Sample Paper 1, Question 19, 20, 21. Read the information on the slide and answer the Sample Paper 1, Question 19, 20, 21. Read the information on the slide and answer the
question. This question is a matching exercise. question. This question is a matching exercise.
The PRINCE2 Practitioner

The PRINCE2 Practitioner


Notes Notes
workshop: Part 3

workshop: Part 3
Module 6

Module 6
166 PRINCE2® Practitioner | Copyright© PeopleCert International Ltd. PRINCE2® Practitioner | Copyright© PeopleCert International Ltd. 167
The answer is F. Consider the rationale behind this answer. The answer is D. Consider the rationale behind this answer.

Sample Paper 1, Question 19, 20, 21. Read the information on the slide and answer the Sample Paper 1, Question 22. Read the information on the slide and answer the
question. This question is a matching exercise. question. This question requires you to engage with information about independent
quality assurance, included in Section 8.3.5, page 84 of the Official Book.
The PRINCE2 Practitioner

The PRINCE2 Practitioner


Notes Notes
workshop: Part 3

workshop: Part 3
Module 6

Module 6
168 PRINCE2® Practitioner | Copyright© PeopleCert International Ltd. PRINCE2® Practitioner | Copyright© PeopleCert International Ltd. 169
Quality theme review
Welcome to a short review on the quality theme. You may refer to the Official Book to refresh
your memory on key points.

Here we see the purpose of the quality theme. Keywords are highlighted to help you
remember purpose. The means to verify that products are fit for purpose must be
defined and implemented, and that is what the quality theme is all about getting done.

This terminology is integral to the quality theme. Consider your answers to this exercise.
The PRINCE2 Practitioner

The PRINCE2 Practitioner


Notes Notes
workshop: Part 3

workshop: Part 3
Module 6

Module 6
170 PRINCE2® Practitioner | Copyright© PeopleCert International Ltd. PRINCE2® Practitioner | Copyright© PeopleCert International Ltd. 171
Here you can find the answers to the exercise on the previous slide. • Specify explicit quality criteria for products in their product descriptions as part of
PRINCE2’s focus on products principle.
• Maintain records to provide evidence that the planned quality activities have been
performed and summarize those activities that are planned or have taken place in
some form of quality register. Explain that a quality register is used to summarize
all of the quality management activities that are planned or have taken place, and
provides information for the end stage reports and end project report.
• Specify the customer’s quality expectations and prioritized acceptance criteria for the
project in the project product description.
• Use lessons to inform quality planning, the definition of quality expectations, and
quality criteria.

Do you understand the difference between quality assurance and quality control?
Quality assurance is a medium- to long-term process within the product design period. It
is a planned and systematic process that is maintained throughout the project.
Quality control is a shorter-term activity, usually at the end of a project cycle.
The roles and responsibilities for quality management is part of PRINCE2’s defined roles
and responsibilities principle.
The PRINCE2 Practitioner

The PRINCE2 Practitioner


Notes Notes
workshop: Part 3

workshop: Part 3
Module 6

Module 6
172 PRINCE2® Practitioner | Copyright© PeopleCert International Ltd. PRINCE2® Practitioner | Copyright© PeopleCert International Ltd. 173
Risk
Welcome to the risk theme.

Take a moment to recall key information regarding the risk theme. Remember you can This slide outlines the key syllabus topics that will be covered in this module.
review the Official Book and add notes wherever necessary.
Purpose of the risk theme To identify, assess, and control uncertainty, and as a result,
improve the ability of the project to succeed.
Definition of a risk An uncertain event that, should it occur, will affect the achievement
of objectives.
The six recommended risk response types:
1. Avoid a threat/exploit an opportunity removing the risk or taking the
opportunity.
2. Reduce a threat/enhance an opportunity taking definitive action to change the
probability/impact of the risk.
3. Transfer the risk Pass part of the risk on to a third party.
4. Share the risk This seeks multiple parties, typically within a supply chain, to share
the risk on a pain/gain share basis.
5. Accept the risk Where the organization ‘takes the chance’ that the risk will occur,
with its full impact if it does.
6. Prepare contingent plans These are a planned set of actions that will be taken
for each risk.
The PRINCE2 Practitioner

The PRINCE2 Practitioner


Notes Notes
workshop: Part 3

workshop: Part 3
Module 6

Module 6
174 PRINCE2® Practitioner | Copyright© PeopleCert International Ltd. PRINCE2® Practitioner | Copyright© PeopleCert International Ltd. 175
Sample Paper 1, Question 29, 30, and 31. Read the information on the slide and answer Sample Paper 1, Question 29, 30, and 31. Read the information on the slide and answer
the question. This question is a matching exercise and an example of reading the question the question. This question is a matching exercise and an example of reading the question
closely to eliminate certain options and match others. You can review information closely to eliminate certain options and match others. You can review information
regarding the composition of the risk register in Section A.25.2, page 329 of the Official regarding the composition of the risk register in Section A.25.2, page 329 of the Official
Book. Book.

The answer is F. Consider the rationale behind this answer. The answer is A. Consider the rationale behind this answer.
The PRINCE2 Practitioner

The PRINCE2 Practitioner


Notes Notes
workshop: Part 3

workshop: Part 3
Module 6

Module 6
176 PRINCE2® Practitioner | Copyright© PeopleCert International Ltd. PRINCE2® Practitioner | Copyright© PeopleCert International Ltd. 177
Sample Paper 1, Question 29, 30, and 31. Read the information on the slide and answer Sample Paper 1, Question 29, 30, and 31. Read the information on the slide and answer
the question. This question is a matching exercise and an example of reading the question the question. Knowledge about the risk budget should be drawn on for this question,
closely to eliminate certain options and match others. You can review information which is included in Section 10.3.7, page 125 of the Official Book.
regarding the composition of the risk register in Section A.25.2, page 329 of the Official
Book.

The answer is B. Consider the rationale behind this answer.


The PRINCE2 Practitioner

The PRINCE2 Practitioner


Notes Notes
workshop: Part 3

workshop: Part 3
Module 6

Module 6
178 PRINCE2® Practitioner | Copyright© PeopleCert International Ltd. PRINCE2® Practitioner | Copyright© PeopleCert International Ltd. 179
Risk review
This is a short review on the risk theme. It is an opportunity to remember the key points.

This is a Foundation-level course extension activity for identifying risks. You have 15
minutes to complete both tasks in this exercise. Refer to the music album project case in
the Scenario Booklet in your Learner Workbook.

This is a Foundation-level course extension activity for identifying risks. Consider how you Here we see the purpose of the PRINCE2 risk theme. Pay particular attention to the
would apply structure to the risks you identified in the first part of the exercise. keywords (in bold), as they will help you to remember the purpose of the risk theme,
which is to identify, assess, and control uncertainty.
The PRINCE2 Practitioner

The PRINCE2 Practitioner


Notes Notes
workshop: Part 3

workshop: Part 3
Module 6

Module 6
180 PRINCE2® Practitioner | Copyright© PeopleCert International Ltd. PRINCE2® Practitioner | Copyright© PeopleCert International Ltd. 181
Controlling a stage
Welcome to the session on the ‘controlling a stage’ process.

A risk can be: Purpose:


• A threat For uncertain events that would have a negative impact on objectives. • assign work to be done
• An opportunity For uncertain events that would have a positive impact on objectives. • monitor the work
Risk response types Solutions to a predicted risk. • deal with issues
Risk owner A named individual who is responsible for the management, monitoring, • report progress to the project board
and control of all aspects of a particular risk assigned to them. • take corrective actions to ensure that the management stage remains within tolerance.
Risk actioner The person who takes action to address the risk. Objectives:

Risk exposure The extent of risk tolerated by the organization at the time. • Attention is focused on delivery of the management stage’s products
• Risks and issues are kept under control.
Risk probability How likely the risk is to happen.
• The business case is kept under review.
Risk impact The effect the risk can have on the project/organization. Risk impact
example: even a small, simple project on replacing core IT network infrastructure can • The agreed products for the management stage are delivered to stated quality
stop an organization working if it goes wrong. standards.
• The project management team is focused on delivery within the tolerances laid down.
Risk proximity The predicted timing of the risk. How quickly the risk is likely to
materialize if no action was taken.
Risk appetite An organization’s unique attitude towards risk-taking. It dictates the
amount of risk that the organization considers acceptable.
Risk tolerance A threshold that if exceeded would lead to a response.
The PRINCE2 Practitioner

The PRINCE2 Practitioner


Notes Notes
workshop: Part 3

workshop: Part 3
Module 6

Module 6
182 PRINCE2® Practitioner | Copyright© PeopleCert International Ltd. PRINCE2® Practitioner | Copyright© PeopleCert International Ltd. 183
This slide outlines the key syllabus topics that will be covered in this module. Sample Paper 1, Question 56, 57, and 58. Read the information on the slide and answer
the question. This is a matching activity. You need to apply information of the activities
within the ‘controlling a stage’ process that you should have checked during the recap
section.

The answer is F. Consider the rationale behind this answer.


The PRINCE2 Practitioner

The PRINCE2 Practitioner


Notes Notes
workshop: Part 3

workshop: Part 3
Module 6

Module 6
184 PRINCE2® Practitioner | Copyright© PeopleCert International Ltd. PRINCE2® Practitioner | Copyright© PeopleCert International Ltd. 185
Sample Paper 1, Question 56, 57, and 58. Read the information on the slide and answer Sample Paper 1, Question 56, 57, and 58. Read the information on the slide and answer
the question. This is a matching activity. You need to apply information of the activities the question. This is a matching activity. You need to apply information of the activities
within the ‘controlling a stage’ process that you should have checked during the recap within the ‘controlling a stage’ process that you should have checked during the recap
section. section.

The answer is B. Consider the rationale behind this answer. The answer is D. Consider the rationale behind this answer.
The PRINCE2 Practitioner

The PRINCE2 Practitioner


Notes Notes
workshop: Part 3

workshop: Part 3
Module 6

Module 6
186 PRINCE2® Practitioner | Copyright© PeopleCert International Ltd. PRINCE2® Practitioner | Copyright© PeopleCert International Ltd. 187
Controlling a stage process review
Welcome to the review on the ‘controlling a stage’ process. It is an opportunity to review and
remember the key points.

This is a Foundation-level course extension activity for typical day-to-day activities carried Here we see the purpose of the ‘controlling a stage’ process. The stage is about taking
out during the ‘controlling a stage’ process. You have 25 minutes to complete this activity. corrective actions to ensure that the management stage remains within tolerance.
This activity is connected to the project manager’s role in controlling a stage. You do not These tolerances can include time, cost, quality, scope, benefits, and risk. Control is
have to produce a lot of text (a list is fine). Refer to your notes and the Official Book to accomplished through the keywords highlighted here: through assigning and monitoring
support your responses. of work; through dealing with issues; reporting progress; and taking corrective actions to
ensure the management stage remains within tolerance.

Here are some examples of daily tasks that could be included in the diary while the
project manager is controlling a stage. Compare it to your list.
The PRINCE2 Practitioner

The PRINCE2 Practitioner


Notes Notes
workshop: Part 3

workshop: Part 3
Module 6

Module 6
188 PRINCE2® Practitioner | Copyright© PeopleCert International Ltd. PRINCE2® Practitioner | Copyright© PeopleCert International Ltd. 189
In the ‘controlling a stage’ process, attention is focused on delivery of the management Remember that the stage plan is produced close to the time when the planned events
stage’s products. Any movement away from the direction and products agreed at the will take place. Central to the ultimate success of the project is the day-to-day control of
start of the management stage is monitored to avoid uncontrolled change and loss of the work that is being conducted. Throughout a management stage, this will consist of
focus. The project management team is focused on delivery within the tolerances laid a cycle of:
down.
• authorizing work to be done
• monitoring progress information about that work, including signing off completed
work packages
• reviewing the situation (including that for product quality) and triggering new work
packages
• reporting highlights
• watching for, assessing, and dealing with issues and risks
• taking any necessary corrective action.

This figure considers the project from the point of view of the project manager. It shows
the kinds of activities involved in the ‘controlling a stage’ process.
The PRINCE2 Practitioner

The PRINCE2 Practitioner


Notes Notes
workshop: Part 3

workshop: Part 3
Module 6

Module 6
190 PRINCE2® Practitioner | Copyright© PeopleCert International Ltd. PRINCE2® Practitioner | Copyright© PeopleCert International Ltd. 191
Module 7
Let’s review: Part 2
Let’s review: Part 2

Let’s review: Part 2


Module 7

Module 7
Let’s review: Part 2
Welcome to this review module of the progress you have made on the course so far.

The main aim of this review module is to reflect on the progress you have made so far This slide recaps the topics you have completed in the course. Your trainer will recap the
and assess your understanding of PRINCE2. exercises you completed over the third part of the workshop.

This diagram reviews the topics you have completed so far. It depicts the structure of the It is time to review your level of confidence and understanding ahead of the examination.
course. This module will confirm the progress made so far and clarify other topics which Reflect on your understanding of key PRINCE2 concepts.
you may wish to review, before moving on to the sample paper.

Notes Notes

194 PRINCE2® Practitioner | Copyright© PeopleCert International Ltd. PRINCE2® Practitioner | Copyright© PeopleCert International Ltd. 195
Let’s review: Part 2

Let’s review: Part 2


Module 7

Module 7
Reflect on how confident you feel in your ability to take the PRINCE2 Practitioner
exam. Refer to Activity sheet 2 in your Learner Workbook which is an extended confidence
evaluation. You can complete this as an extension activity to track your progress and help
you recognize which areas you would benefit from revisiting.

If there are any areas where you feel you lack confidence, it is important prioritize these
topics for your private study.

Notes Notes

196 PRINCE2® Practitioner | Copyright© PeopleCert International Ltd. PRINCE2® Practitioner | Copyright© PeopleCert International Ltd. 197
Module 8
The PRINCE2®
Practitioner
workshop: Part 4
Other areas to explore
The PRINCE2® Practitioner workshop: Part 4
Other areas to explore
The PRINCE2 Practitioner

The PRINCE2 Practitioner


workshop: Part 4

workshop: Part 4
Module 8

Module 8
On this slide, all the areas of the course are in colour which demonstrates that they are
options for you to revisit.

Notes Notes

200 PRINCE2® Foundation | Copyright© PeopleCert International Ltd. PRINCE2® Foundation | Copyright© PeopleCert International Ltd. 201
Activity sheet 3: Questions and Rationales

Questions: Answers:
1. During stage 3, the music album project board consists of the vice president as the 1. During stage 3, the music album project board consists of the vice president as the
executive and senior user, with no senior supplier. executive and senior user, with no senior supplier.
The PRINCE2 Practitioner

The PRINCE2 Practitioner


Is this an appropriate application of the ‘defined roles and responsibilities’ principle, and why? Is this an appropriate application of the ‘defined roles and responsibilities’ principle, and why?
workshop: Part 4

workshop: Part 4
Module 8

Module 8
A. Yes, because the vice president can represent all three primary stakeholder interests. A. Yes, because the vice president can represent all three primary stakeholder interests.
B. Yes, because appointing the vice president provides an explicit project management team B. Yes, because appointing the vice president provides an explicit project management team
structure. structure.
C. No, because the vice president cannot judge if the project can be feasibly delivered by all C. No, because the vice president cannot judge if the project can be feasibly
supply streams. delivered by all supply streams.
D. No, because having defined roles should help each person to answer ‘what is expected of D. No, because having defined roles should help each person to answer ‘what is expected of
me? me?

1. The vice president cannot represent all suppliers, especially as


2. The work package to create the ‘artwork’ will commence in stage 3. The graphic design
some are from third-party organizations. All three stakeholder
company uses their own development method to deliver products. They have appointed their
interests must be represented: two out of three stakeholder interests
are not enough. graphic designer as the team manager to complete this work package. The project manager
will review the appointment at the end of the stage.
Which principle is being applied by the review, and why?
2. The work package to create the ‘artwork’ will commence in stage 3. The graphic design
company uses their own development method to deliver products. They have appointed their A. ‘Manage by stages’, because the project manager reviews the contents of the
graphic designer as the team manager to complete this work package. The project manager project initiation documentation when planning stage 4.
will review the appointment at the end of the stage. B. ‘Manage by stages’, because the project manager reviews the status of stage 3 products at
Which principle is being applied by the review, and why? the end of the stage.

A. ‘Manage by stages’, because the project manager reviews the contents of the project C. ‘Manage by exception’, because the project board delegates the authority to the project
initiation documentation when planning stage 4. manager on a stage-by-stage basis.

B. ‘Manage by stages’, because the project manager reviews the status of stage 3 products at D. Manage by exception’, because the team manager’s performance should be reviewed
the end of the stage. before authorizing the next work package.

C. ‘Manage by exception’, because the project board delegates the authority to the project
manager on a stage-by-stage basis.
D. Manage by exception’, because the team manager’s performance should be reviewed
before authorizing the next work package.

2. Applying the manage by stages principle ensures that key decisions


are made prior to the detailed work needed to implement them. For
that reason, the project manager reviews the components of the
project initiation documentation as part of planning the next stage.

202 PRINCE2® Foundation | Copyright© PeopleCert International Ltd. PRINCE2® Foundation | Copyright© PeopleCert International Ltd. 203
Module 9
The PRINCE2 Practitioner
sample paper
The PRINCE2 Practitioner sample paper
Welcome to the PRINCE2 Practitioner sample paper. This module will replicate exam conditions
and will give you an opportunity to review your strengths and weaknesses.
The PRINCE2® Practitioner

The PRINCE2® Practitioner


sample paper

sample paper
Module 9

Module 9
Here are some instructions if you are attempting the mini sample paper:
Duration You have 45 minutes to complete 21 sample exam questions (21 marks
available).
Materials The sample paper will be distributed according to the format of the training
delivery. For face-to-face sessions, a printed sample paper will be provided. For online
training, you can complete the sample exam session online. You can write notes during
the exam (for instance, in a notepad/loose paper) if you prefer, but you must submit
answers on the exam answer sheet. You should also refer to the Scenario Booklet
included in the Learner Workbook, as well as your PRINCE2 Official Book. Remember this
is an open book exam.
Conditions You need to be silent during the sample paper exam. If the session is taking
place virtually, use the mute feature for the duration of the process. Phones should be
turned off and other distractions kept to a minimum. If you finish early, use the remaining
time to revise from the Official Book.

Notes Notes

206 PRINCE2® Practitioner | Copyright© PeopleCert International Ltd. PRINCE2® Practitioner | Copyright© PeopleCert International Ltd. 207
The sample paper exam should help you find out which areas to focus on for your
The PRINCE2® Practitioner

The PRINCE2® Practitioner


revision.
sample paper

sample paper
Module 9

Module 9
You may wish to review the answers and rationales booklet for the sample paper between
this module and the next.

Notes Notes

208 PRINCE2® Practitioner | Copyright© PeopleCert International Ltd. PRINCE2® Practitioner | Copyright© PeopleCert International Ltd. 209
Independent study
The PRINCE2® Practitioner

The PRINCE2® Practitioner


sample paper

sample paper
Module 9

Module 9
Notes Notes

210 PRINCE2® Practitioner | Copyright© PeopleCert International Ltd. PRINCE2® Practitioner | Copyright© PeopleCert International Ltd. 211
Module 10
Exam preparaton
hints and tips
Exam preparaton hints and tips
Welcome to this module on exam preparation.

In this module, you will get some tips and tricks for successfully completing the PRINCE2 You will have up to 2 hours and 30 minutes to complete the exam. If you take the exam
Practitioner examination. You will also learn how to avoid common mistakes during the in a language that is not your native or working language, you may be awarded 25% extra
exam and be able to share ideas for how to prepare. time, or 188 minutes in total. There are 68 questions on the exam, each worth one mark.
There is no negative marking.
The exam includes a scenario in a Scenario Booklet which is not the same as the scenario
used for the workshop. Be sure to read the scenario in the Scenario Booklet thoroughly,
as you will be unfamiliar with this version. Take time to make notes or highlight relevant
sections before responding to the questions.
Exam preparation

Exam preparation
Module 10

Module 10
hints and tips

hints and tips


The PRINCE2 Practitioner exam is open book. You can consult the Official Book, Managing
Successful Projects with PRINCE2, during the exam. You can also make notes inside the
book. No material is permitted in the exam other than the Official Book.
Is your exam booked already? If it is, are you sure of the date and time? If it is not, find
out how to and book it. Do you know where your exam will take place? Be sure you know
the location of the exam.
You must score at least 38 marks to pass the exam and will receive your results as soon as
they are released. If you do not pass, you can schedule a retake of the exam immediately.

Notes Notes

214 PRINCE2® Practitioner | Copyright© PeopleCert International Ltd. PRINCE2® Practitioner | Copyright© PeopleCert International Ltd. 215
Time management One key issue is running out of time in an exam. On average, you Here are some useful independent study tips:
have about two minutes to respond to each question. If a particular question is taking
Retrieval Practise asking a question and having to provide a response. This is
longer to answer, move on and come back to it after you have attempted the rest of the
consistently found to be one of the most effective ways to prepare for an exam. The
questions. Remember to keep checking the time as you progress through the exam.
most obvious format for this is completing a sample paper, so a complete sample paper
Attention to detail Make sure to read each question thoroughly, more than once. (Sample Paper 2) is included in the Learner Workbook. You could also ask someone to
Sometimes a question may seem daunting at first, but after a few readings it starts to ask you questions about PRINCE2 principles, themes, and processes.
make more sense. Better still, pay attention to initial thoughts about the answer to a
Spacing Do not overdo study sessions. Little and often is better than all at once.
question. Often, we can double-back and change our initial response, when in fact that
Essentially, short study sessions over a longer period is better than trying to complete all
first response was correct. Make notes during the exam, including highlighting keywords
exam preparation in one go.
in the questions and eliminating answer options which seem incorrect.
Interleaving Mix up the topics to review, to encourage thinking deeply about the
Pressure Stress is a perfectly natural reaction to a difficult situation such as an exam.
material. This may also help you to become familiar with the Official Book, add further
Exam preparation

Exam preparation
Stress can challenge us to achieve great things, but it can also get out of control and
annotations and tabs to access information quickly, and highlight connections between
Module 10

Module 10
hints and tips

hints and tips


cause problems. If you feel stressed or panicky at any point before or during the exam,
topics where relevant.
take deep breaths in and out. Simple breathing exercises will get more oxygen to the
brain, which allows us to think more clearly. Elaborative interrogation This approach requires you to ask ‘why is this true?’ and
try to find your own rationale for responses before checking them in the Answers and
rationales booklet. This will help to further deepen knowledge and may highlight any
misconceptions.
Reading out loud This has been shown to be far more effective than reading alone.
Distraction free Listening to music or having phones nearby has been found to
compete for attention and usually reduces learning. Try to reduce distractions during
study sessions.
Teaching others Also known as ‘the protégé effect’. Having to teach someone else
makes us learn the material more thoroughly and clearly. If you have a partner, friend,
or colleague who can help you prepare for the exam, this approach can be really helpful.

Notes Notes

216 PRINCE2® Practitioner | Copyright© PeopleCert International Ltd. PRINCE2® Practitioner | Copyright© PeopleCert International Ltd. 217
Exam preparation
Module 10
hints and tips

Notes

218 PRINCE2® Practitioner | Copyright© PeopleCert International Ltd.


Module 11
Course review
Course review What’s Next?
Welcome to the final module of the PRINCE2 practitioner course.
Course review

Course review
Module 11

Module 11
You have reached the end of the course. This shows the progress you have made.

Notes Notes

222 PRINCE2® Practitioner | Copyright© PeopleCert International Ltd. PRINCE2® Practitioner | Copyright© PeopleCert International Ltd. 223
If you want more practice and would like to bolster your confidence before take the live For greater peace of mind, PeopleCert offers an exam re-sit option.
exam, PeopleCert’s Official Mock Exams offer you the opportunity to test your readiness
Take2:
for the official exam with fully timed and marked mock exams and a chance to you
familiarize yourself with the exam interface. • gives you the chance to retake a demanding exam, future-proofing your chances
against any unexpected setbacks
Ask your training provider for more information or visit: https://www.peoplecert.org/
• gives you up to six months from the date of the initial exam, ensuring that you have
help-and-support/FAQ#28C88E91C5E64BD59D3D0DE4EEBE5768 for more details.
all the time you need to get ready
• allows you to retake an unsuccessful exam at a fixed, attractive rate, at a fraction of
the cost, without having to purchase it again at full price
• allows you to schedule the re-sit exam when it best suits your schedule and take it
from the comfort of your home or office.
Ask your training provider for more information or log in to your PeopleCert account to
purchase.
Course review

Course review
Module 11

Module 11
Notes Notes

224 PRINCE2® Practitioner | Copyright© PeopleCert International Ltd. PRINCE2® Practitioner | Copyright© PeopleCert International Ltd. 225
This course will not be of value to you unless you put a plan of action to use what you Register for your exam if you have not already done so.
have learned. You are probably, and rightly so, concerned with passing the exam right
When you have successfully passed your exam, do not forget to claim your digital badge
now if you have signed up for it. But do take a moment to reflect and plan some quick
from MyAxelos. Just log in to your PeopleCert account to claim your free MyAxelos
wins for when you take this back to work. Ask yourself what you have learned that you
subscription. Your digital badge is your way of demonstrating achievement and
would like to share with others in your organization. Take the time to write down your
certification in the digital age. You can use your digital badge across online platforms
personal commitments as to how you will apply what you have learned when you go back
including your social media and your email signature to showcase your certifications.
to work. You will be glad you did.
Also, this training qualifies for MyAxelos Continuing Professional Development points,
which are needed to maintain your digital badge. Be sure to log in to claim your points
as you progress through your PRINCE2 journey.
Finally, thanks for spending time learning with us!
Course review

Course review
Module 11

Module 11
Notes Notes

226 PRINCE2® Practitioner | Copyright© PeopleCert International Ltd. PRINCE2® Practitioner | Copyright© PeopleCert International Ltd. 227
PRINCE2 Agile Foundation PRINCE2 Agile Foundation is designed to give you an Because of your enrolment on the PRINCE2 practitioner course, you now have three
overview of PRINCE2 and Agile concepts as well as the techniques and knowledge needed months of free access to MyAxelos.
to apply the two together.
On MyAxelos, it is possible to:
PRINCE2 Agile Practitioner PRINCE2 Agile Practitioner is designed to help you to apply
• access a wide range of PRINCE2 and ProPath content
the PRINCE2 Agile method in practice through real-world project management examples.
• explore content from the wider Axelos best practice portfolio, including ITIL 4®
• preview excerpts from popular publications
• support your professional development and take advantage of the platform’s
continuing professional development to boost your future career.
Course review

Course review
Module 11

Module 11
If you are considering diversifying your skill set, PeopleCert’s portfolio of highly sought-
after certifications may be of interest to you. Listed above are certifications that are
perfect for the PRINCE2 practitioner.
To view PeopleCert’s entire portfolio of certifications, please visit: https://www.peoplecert.
org/browse-certifications

Notes Notes

228 PRINCE2® Practitioner | Copyright© PeopleCert International Ltd. PRINCE2® Practitioner | Copyright© PeopleCert International Ltd. 229
The survey is an opportunity to provide feedback to PeopleCert on your course. You
can use the QR code on the slide to access the survey. It is also included in the Learner
Workbook.
Course review
Module 11

Notes

230 PRINCE2® Practitioner | Copyright© PeopleCert International Ltd.


Discover more at my.axelos.com

Appendix 1
Handouts/
activity sheets

Activity sheet 1

Activity sheet 2

Activity sheet 3
Discover more at my.axelos.com
Handouts / Activity sheets

Handouts / Activity sheets


Appendix 1

Appendix 1
Activity sheet 1 Principles: Question 2

During the initiation stage, the vice president stated that attendance at launch events
held on Monday evenings is low, and on previous projects, this had resulted in lower
album sales. As a result, the launch event for this album will be held later in the week.
Independent study
Which principle is being applied, and why?

This short activity sheet is available for you A. 'Continued business justification', because there is sufficient reason to start this
to practise PRINCE2 Practitioner sample project.
exam questions in your own time. You
will need to refer to the Sample Paper 1 B. 'Continued business justification', because the project's justification should remain
Scenario Booklet and may also refer to unchanged.
the PRINCE2 Official Book. The questions C. 'Learn from experience', because project teams should learn from what occurred on
relate to PRINCE2 principles and the similar projects.
organization theme.
D. 'Learn from experience', because the project should continue to learn from its own
experiences.

Do something now

Notes

234 PRINCE2® Practitioner | Copyright© PeopleCert International Ltd. PRINCE2® Practitioner | Copyright© PeopleCert International Ltd. 235
Handouts / Activity sheets

Handouts / Activity sheets


Appendix 1

Appendix 1
Principles: Question 3 Organization: Question 17

Towards the end of stage 2, the project manager realized that not all of the ‘recorded As a result of previous lessons, this recommendation has been made: “If an executive is
sample songs’ would be completed before the end of the stage. The project manager appointed who does not have an understanding of PRINCE2, someone with experience
decided to move the remaining work to stage 3. This enabled the project manager to of applying PRINCE2 should undertake business assurance.”. The recording studio
report that stage 2 was completed within time tolerance. manager has therefore been appointed as business assurance for stage 2.

Is this an appropriate application of the ‘manage by stages’ principle, and why? Is this appropriate, and why?

A. Yes, because moving the work to stage 3 avoided an exception situation in stage 2. A. Yes, because the recording studio manager is responsible for the delivery of the
‘recorded album’.
B. Yes, because stage 3 is not the final stage, so work can be moved from
stage 2. B. Yes, because the recording studio manager has the experience required to deliver
the ‘recorded album’.
C. No, because the project board should assess project viability on completion of work
planned for stage 2. C. No, because the recording studio manager’s business justification may conflict with
the interests of the executive.
D. No, because work in stage 3 should start while work planned for stage 2 is being
completed. D. No, because the recording studio manager will not be available throughout the
project lifecycle.

Do something now Do something now

Notes Notes

236 PRINCE2® Practitioner | Copyright© PeopleCert International Ltd. PRINCE2® Practitioner | Copyright© PeopleCert International Ltd. 237
Discover more at my.axelos.com
Handouts / Activity sheets

Handouts / Activity sheets


Appendix 1

Appendix 1
Activity sheet 2 Topic Confidence rating (1–5) Notes

Principles

Extended confidence General Low High

evaluation
Themes

Business case Low High

Organization Low High

Plans Low High

Risk Low High

Quality Low High

Change Low High

Progress Low High

238 PRINCE2® Practitioner | Copyright© PeopleCert International Ltd. PRINCE2® Practitioner | Copyright© PeopleCert International Ltd. 239
Discover more at my.axelos.com
Handouts / Activity sheets

Handouts / Activity sheets


Appendix 1

Appendix 1
Topic Confidence rating (1–5) Notes

Processes

Starting up a project Low High

Directing a project Low High

Initiating a project Low High

Controlling a stage Low High

Managing product delivery Low High

Managing a stage boundary Low High

Closing a project Low High

Notes

240 PRINCE2® Practitioner | Copyright© PeopleCert International Ltd. PRINCE2® Practitioner | Copyright© PeopleCert International Ltd. 241
Handouts / Activity sheets

Handouts / Activity sheets


Appendix 1

Appendix 1
Activity sheet 3 Question

During stage 3, the music album project board consists of the vice president as the
executive and senior user, with no senior supplier.
Is this an appropriate application of the ‘defined roles and responsibilities’
Justify your response principle, and why?
A. Yes, because the vice president can represent all three primary stakeholder
interests.
B. Yes, because appointing the Vice President provides an explicit project management
team structure.
C. No, because the Vice President cannot judge if the project can be feasibly delivered
by all supply streams.
D. No, because having defined roles should help each person to answer ‘what is
expected of me?’.

Do something now
Correct answer:

Notes

242 PRINCE2® Practitioner | Copyright© PeopleCert International Ltd. PRINCE2® Practitioner | Copyright© PeopleCert International Ltd. 243
Handouts / Activity sheets

Handouts / Activity sheets


Appendix 1

Appendix 1
Question 7

The work package to create the ‘artwork’ will commence in stage 3. The graphic
design company uses their own development method to deliver products. They have
appointed their graphic designer as the team manager to complete this work package.
The project manager will review the appointment at the end of the stage.
Which principle is being applied by the review, and why?
A. ‘Manage by stages’, because the project manager reviews the contents of the project
initiation documentation when planning stage 4.
B. ‘Manage by stages’, because the project manager reviews the status of stage 3
products at the end of the stage.
C. ‘Manage by exception’, because the project board delegates the authority to the
project manager on a stage-by-stage basis.
D. ‘Manage by exception’, because the team manager’s performance should be
reviewed before authorizing the next work package.

Do something now
Correct answer:

Notes

244 PRINCE2® Practitioner | Copyright© PeopleCert International Ltd. PRINCE2® Practitioner | Copyright© PeopleCert International Ltd. 245
Discover more at my.axelos.com

Appendix 2
Resources

PRINCE2® Practitioner
Exam Specification

Sample papers
Candidate syllabus and index 3. Syllabus
The tables below give a summary of the concepts that are tested in the exam, and the main parts
of the Official Book in which these are described. (Ch = chapter, † = all subsections of that section
of the Official Book.)
1. Introduction
Learning outcome 1 - Apply the PRINCE2 principles in context
The PRINCE2 Practitioner qualification is intended for project managers and aspiring project
managers. It is also relevant to other key staff involved in the design, development and delivery Assessment Criteria Book Bloom’s No. Learner
of projects, including: project board members (e.g. senior responsible owners), team managers references level marks Workbook
(e.g. product delivery managers), project assurance (e.g. business change analysts), project page no.
support (e.g. Project and Programme Office personnel), and operational line managers/staff.
1.1 Analyse the application of PRINCE2 Ch. 3†,4.1, BL4 8 34, 45, 56-
The PRINCE2 Practitioner examination is intended to assess whether a candidate can apply and
principles in context 4.3† 59
tailor the PRINCE2 project management method (as described in the syllabus below). A
successful Practitioner candidate should, with suitable direction, be able to start applying the Learning outcome 2 - Apply and tailor relevant aspects of PRINCE2
method to a real project but may not be sufficiently skilled to do this appropriately for all themes in context
situations. Their individual project management expertise, complexity of the project and the
support provided for the use of PRINCE2 in their work environment will all be factors that impact Assessment Criteria Book Bloom’s No. Learner
what the Practitioner can achieve. references level marks Workbook
page no.
2. Exam Overview
2.1.1 Apply the PRINCE2 requirements for the 6.1, fig 6.1, BL3 3 35, 46, 83,
Material PRINCE2 This is an ‘open book’ exam. The Managing Successful business case theme, demonstrating an 6.2, A.1, A.2 100-102
allowed Official Book Projects with PRINCE2 Official Book, 2017 edition, understanding of:
should be used (and you can make notes inside the • benefits management approach &
book), but no other material is allowed. business case
Appendix 2

Appendix 2
Exam duration 2 hours 30 Candidates taking the exam in a language that is not • the recommended roles and
Resources

Resources
minutes their native or working language may be awarded 25% responsibilities
extra time, i.e. 188 minutes in total. • outputs, outcomes, benefits and dis-
There are 68 (part) questions, each worth 1 mark. benefits
Number of 68 marks
marks There is no negative marking.
2.1.2 Assess whether an approach to applying 6-6.3†, 4.3†, BL4 2 35, 46, 83,
Pass mark 38 marks You will need to get 38 (part) questions correct to pass the business case theme is effective and fit for 5.1, A.1†, A.2†, 100-102
the exam. purpose, taking into consideration: the App.C†,
Level of Bloom’s “Bloom’s level” describes the type of thinking needed context, the PRINCE2 principles, and the 14.5.2, 16.5.2
thinking levels 3&4 to answer the question. For Bloom’s level 3 questions, purpose and requirements of the theme
you need to apply your knowledge to a situation. For
2.2.1 Apply the PRINCE2 requirements for the 7.1, 7.2†, fig BL3 3 35, 46, 60,
Bloom’s 4 questions, you need to analyse the
organization theme, demonstrating an 7.2, 7.3.1, 65-68
information provided and reason whether a course of
understanding of: 7.3.7, A.5, App
action is effective/appropriate.
• communication management C†
Exam format Scenario, You should read the ‘Project Scenario’ which gives approach
additional background information about the project that the • the recommended roles and
information questions apply to. For at least one question, you will responsibilities
& questions also need to use the ‘Additional Information’ which • the recommended project
gives information about people who may be working management team structure
on the project.
Question Classic & The questions are all ‘multiple choice’. There is a short
types matching description of a situation, and then a question. For the
‘classic’ questions (1 mark), you have a question and
four options (A,B,C,D). For the ‘matching’ questions (3
marks) you have 3 pieces of information and you have
to choose an answer for each from a list of 5 or 6
options.
Please see the sample paper for an example of the exam format and content.

248 PRINCE2® Practitioner | Copyright© PeopleCert International Ltd. PRINCE2® Practitioner | Copyright© PeopleCert International Ltd. 249
Learning outcome 2 - Apply and tailor relevant aspects of PRINCE2 Learning outcome 2 - Apply and tailor relevant aspects of PRINCE2
themes in context themes in context

Assessment Criteria Book references Bloom’s No. Learner Assessment Criteria Book references Bloom’s No. Learner
level marks Workbook level marks Workbook
page no. page no.

2.2.2 Assess whether an approach to 7.1-2†, 7.3.1-5, BL4 2 35, 46, 60, 14.5.2, 16.5.2,
applying the organization theme is 7.3.7, 4.3†, 5.1, 65-68 18.5.2, 19.5.2,
effective and fit for purpose, taking into 16.5.2, A.5†, App.C†, A.16†, App.C†
consideration: the context, the PRINCE2 15.5.2, 14.5.3,
2.5.1 Apply the PRINCE2 requirements 10.2, tab 10.1, BL3 3 35, 46, 174,
principles, and the purpose and 16.5.3, 17.5.3,
for applying the risk theme, 10.3.2, 10.4† 181-182
requirements of the theme 18.5.3, 19.5.3,
demonstrating an understanding of: (excluding risk
20.5.3
• risk management approach, estimation and
2.3.1 Apply the PRINCE2 requirements 8.1.1, fig 8.1, 8.2, BL3 3 35, 46, 165, risk register evaluation
for the quality theme, demonstrating an 8.3.5, 8.3.6, 8.3.8- 171-173 • recommended roles and techniques),
understanding of: 9, 8.3.11, A.17, responsibilities A.24†, A.25†
• product description, project A.21, A.22, A.23, the recommended risk management
product description, quality procedure
management approach, quality
2.5.2 Assess whether an approach to Ch.10† BL4 2 35, 46, 174,
register
applying the risk theme is effective and fit (excluding risk 181-182
• recommended roles and
for purpose, taking into consideration: estimation and
responsibilities
the context, the PRINCE2 principles, and evaluation
• quality planning, quality
the purpose and requirements of the techniques), 4.3†,
Appendix 2

Appendix 2
control, quality assurance
Resources

Resources
theme 5.1†, 16.5.2, A.24-
2.3.2 Assess whether an approach to Ch.8† (excluding BL4 2 35, 46, 165, 25†, App.C†
applying the quality theme is effective 8.3.10 & 8.3.12), 171-173
2.6.1 Apply the PRINCE2 requirements 11.1, tab 11.1, BL 3 3 35, 46, 139,
and fit for purpose, taking into 4.3†, 5.1, 16.5.2,
for applying the change theme, 11.2†, tab 11.2, 146-148
consideration: the context, the PRINCE2 19.5.2, A.17†,
demonstrating an understanding of: 11.4†, 11.3.6,
principles, and the purpose and A.21-23†, App.C†
• change control approach, A.3†, A.6†, A.12†,
requirements of the theme
configuration item record, issue A.13†
2.4.1 Apply the PRINCE2 requirements tab 9.1, 9.2.1†, BL3 3 35, 46, 149, register, issue report, product
for applying the plans theme, 9.2†, 9.3.1- 154-155 status account
demonstrating an understanding of: 7(excluding • recommended roles and
• project plan, stage plan, detailed responsibilities
exception plan, team plan estimating and the recommended issue and change
• recommended roles and resource control procedure
responsibilities scheduling
2.6.2 Assess whether an approach to Ch.11†, 4.3†, 5.1, BL 4 2 35, 46, 139,
• the recommended approach to techniques),
applying the change theme is effective 16.5.2, 17.5.2, 146-148
planning, including the A.16†
and fit for purpose, taking into A.3†, A.6†, A.12†,
recommended approach to
consideration: the context, the PRINCE2 A.13†, A.18†,
defining and analysing the
principles, and the purpose and App.C†
products
requirements of the theme
2.4.2 Assess whether an approach to 9.1-3† (excluding BL4 2 35, 46, 149,
2.7.1 Apply the PRINCE2 requirements 12.2†, tab 12.2, BL 3 3 35, 46, 156,
applying the plans theme is effective and detailed 154-155
for controlling progress, demonstrating 12.2.1, 12.2.3, 163-164
fit for purpose, taking into consideration: estimating and
an understanding of: A.4†, A.7†, A.8†,
the context, the PRINCE2 principles, and resource
• checkpoint report, daily log, A.9†, A.10†,
the purpose and requirements of the scheduling
end project report, end stage A.11†, A.14†,
theme techniques),
report, exception report, A.26†
9.4.1, 4.3†, 5.1,
highlight report, lessons log,

250 PRINCE2® Practitioner | Copyright© PeopleCert International Ltd. PRINCE2® Practitioner | Copyright© PeopleCert International Ltd. 251
Learning outcome 2 - Apply and tailor relevant aspects of PRINCE2 Learning outcome 3 - Apply (and tailor) relevant aspects of PRINCE2
themes in context processes in context
Assessment Criteria Book references Bloom’s No. Learner Assessment Criteria Book Bloom’s No. Learner
level marks Workbook references level marks Workbook
page no. page no.

work package 3.1.1. Carry out the starting up a project 14.2-4†, BL 3 3 47, 75, 92,
• recommended roles and process activities, demonstrating an 14.4†, A.19†, 94-95
responsibilities understanding of: 14.1-6, Ch,
tolerances and raising exceptions • the recommended associated 6-12†
actions
2.7.2 Assess whether an approach to 12.1-3†, 4.3†, BL4 2 35, 46, 156,
• recommended roles and
applying the progress theme is effective ,5.1, 14.5.2, 163-164
responsibilities
and fit for purpose, taking into 17.5.2, 18.5.2,
how the themes may be applied
consideration: the context, the PRINCE2 19.5.2, 20.5.2,
principles, and the purpose and A.4†, A.7-11†, 3.1.2 Assess whether starting up a project Ch.14† BL4 1 47, 75, 92,
requirements of the theme A.14†, A.26†, process activities/actions, roles and (excluding 94-95
App.C† responsibilities are effective and fit for 14.5.2), 4.3†,
purpose, taking into consideration: the 13.4, A.19†,
context, the PRINCE2 principles, and the App.C†
purpose and objectives of the process

3.2.1 Carry out the directing a project process 15.2-4†, BL 3 3 47, 80, 92,
activities, demonstrating an understanding of: 15.4†, tab 96
Appendix 2

Appendix 2
• the recommended associated 15.1-6, Ch,
Resources

Resources
actions 6-12†
• recommended roles and
responsibilities
how the themes may be applied

3.2.2 Assess whether directing a project Ch.15†, 4.3†, BL 4 1 47, 80, 92,
process activities/actions, roles and 13.4, A.19†, 96
responsibilities are effective and fit for App.C†
purpose, taking into consideration: the
context, the PRINCE2 principles, and the
purpose and objectives of the process

3.3.1 Carry out the initiating a project process 16.2-4†, BL 3 3 47, 80, 92,
activities, demonstrating an understanding of: 16.4†, A.20†, 97-98
• the recommended associated tab 16.1-9,
actions Ch, 6-12†
• recommended roles and
responsibilities
how the themes may be applied

3.3.2 Assess whether initiating a project Ch.16† BL 4 1 47, 80, 92,


process activities/actions, roles and (excluding 97-98
responsibilities are effective and fit for 16.5.2), 4.3†,
purpose, taking into consideration: the 13.4, A.20†,
context, the PRINCE2 principles, and the App.C†
purpose and objectives of the process

252 PRINCE2® Practitioner | Copyright© PeopleCert International Ltd. PRINCE2® Practitioner | Copyright© PeopleCert International Ltd. 253
Learning outcome 3 - Apply (and tailor) relevant aspects of PRINCE2 Learning outcome 3 - Apply (and tailor) relevant aspects of PRINCE2
processes in context processes in context
Assessment Criteria Book Bloom’s No. Learner Assessment Criteria Book Bloom’s No. Learner
references level marks Workbook references level marks Workbook
page no. page no.

3.4.1 Carry out the controlling a stage process 17.2-4†, BL 3 3 47, 92, 183, 3.7.1 Carry out the closing a project process 20.2-4†, BL 3 2 47, 92, 124,
activities, demonstrating an understanding of: 17.4†, tab 189-191 activities, demonstrating an understanding of: 20.4†, tab 129-131
the recommended associated
• 17.1-8, Ch 6- • the recommended associated 20.1-5, Ch 6-
actions 12† actions 12†
• recommended roles and • recommended roles and
responsibilities responsibilities
how the themes may be applied how the themes may be applied

3.4.2 Assess whether controlling a stage Ch.17† BL 4 1 47, 92, 183, 3.7.2 Assess whether closing a project Ch.20† BL 4 1 47, 92, 124,
process activities/actions, roles and (excluding 189-191 process activities/actions, roles and (excluding 129-131
responsibilities are effective and fit for 17.5.2), 4.3†, responsibilities and actions are effective and 20.5.2), 4.3†,
purpose, taking into consideration: the 13.4, App.C†) fit for purpose, taking into consideration: the 13.4, App.C†
context, the PRINCE2 principles, and the context, the PRINCE2 principles, and the
purpose and objectives of the process purpose and objectives of the process

3.5.1 Carry out the managing product 18.2-4†, BL 3 2 47, 92, 132,
delivery process activities, demonstrating an 18.4†, tab 137-138
understanding of: 18.1-3, Ch 6-
Appendix 2

Appendix 2
the recommended associated
• 12†
Resources

Resources
actions
• recommended roles and
responsibilities
how the themes may be applied

3.5.2 Assess whether managing product Ch.18† BL 4 1 47, 92, 132,


delivery process activities/actions, roles and (excluding 137-138
responsibilities are effective and fit for 18.5.2), 4.3†,
purpose, taking into consideration: the 13.4, App.C†
context, the PRINCE2 principles, and the
purpose and objectives of the process

3.6.1 Carry out the managing a stage 19.2-4†, BL 3 2 47, 92, 116,
boundary process activities, demonstrating an 19.4†, tab 121-123
understanding of: 19.1-5, Ch 6-
the recommended associated
• 12†
actions
• recommended roles and
responsibilities
how the themes may be applied

3.6.2 Assess whether managing a stage Ch.19† BL 4 1 47, 92, 116,


boundary process activities/actions, roles and (excluding 121-123
responsibilities are effective and fit for 19.5.2), 4.3†,
purpose, taking into consideration: the 13.4, App.C†
context, the PRINCE2 principles, and the
purpose and objectives of the process

254 PRINCE2® Practitioner | Copyright© PeopleCert International Ltd. PRINCE2® Practitioner | Copyright© PeopleCert International Ltd. 255
Project Scenario – Music Album Project
Sample Paper 1 & 2: A small independent record company is working with a new singer with the objective of releasing

their first album.

The record company will undertake a project to produce the ‘album ready for launch’. The singer

has already written the songs. Contractual negotiations between the singer and the record

company will be outsourced to a legal firm. The record company has booked studio time with an

external producer and has hired a graphic design company to produce the artwork. The album

will be released through established delivery channels, for example it will be available to

download or buy on CD. They have decided that the promotional video and launch event are

Scenario Booklet outside the scope of the project. However, the plan for the launch event will be produced by an

Multiple Choice external events company as part of this project.

Examination Duration: 2 hours and 30 minutes


Initially, some sample songs will be produced to allow the internal Marketing Manager to check

Instructions with focus groups that the music has a market, because the music industry is a highly

1. Read the ‘Project Scenario’ carefully. You will need this information to answer the questions competitive business.
in the Question Booklet.
Appendix 2

Appendix 2
Resources

Resources
2. For some questions you will also need the ‘Additional Information’. The duration of the project is 10 months and the budget is £100,000.
3. The ‘Project Scenario’ and ‘Additional Information’ are not based on actual organizations.
Any similarities to known organizations are coincidental.
Stage 1 Initiation stage
Stage 2 Signed contract
Recorded sample songs
Focus groups report
Stage 3 Recorded album
Artwork
Stage 4 Registered artwork
Signed contracts for delivery channels
Launch event plan
Project product: Album ready for launch

256 PRINCE2® Practitioner | Copyright© PeopleCert International Ltd. PRINCE2® Practitioner | Copyright© PeopleCert International Ltd. 257
Additional Information
The Chief Executive Officer (CEO) of the record company obtained finance from external
investors to establish the company 10 years ago. He travels around the world to identify singers
and groups to contract and then delegates their management to the Vice President.

The Vice President supervises the production of albums and associated products. Her annual
bonus depends on the success of this project. She has no experience of using PRINCE2.

The Production Manager, who reports to the Vice President, has successfully managed the
delivery of several albums in the past, using a range of project management methods, including
PRINCE2.

The Contracts Manager is a full-time employee of the record company and is responsible for
ensuring that the contracts deliver the project. He is an experienced PRINCE2 project manager.

The Marketing Director is responsible, within the record company, for ensuring that marketing
campaigns will deliver value for money.

The Marketing Manager is responsible, within the record company, for delivering marketing
campaigns that will achieve the required sales of albums and associated products.

The Production Assistant is employed by the record company to assist the Production and
Marketing Managers with documentation and communications.
Appendix 2

Appendix 2
The singer’s agent is negotiating the contract between the singer and the record company. He
Resources

Resources
will have an ongoing interest in the terms of the singer’s contract.

The singer has not previously released an album. He has written a number of songs which may
be recorded for this album.

The music lawyer is a specialist lawyer, contracted by the record company to ensure that the
contracts, copyright, and project material are dealt with appropriately.

The Recording Studio Manager is an employee of the recording studio contracted by the record
company to produce the recordings. He is PRINCE2 qualified.

The Graphic Designer is employed by an external graphic design company. She will produce the
artwork for the album.

The Events Organizer will plan the release event for the album which will include a live
performance of the songs by the singer. He has a lot of experience of planning and monitoring
small projects.

258 PRINCE2® Practitioner | Copyright© PeopleCert International Ltd. PRINCE2® Practitioner | Copyright© PeopleCert International Ltd. 259
Sample Paper 1: PRINCIPLES
1. The Chief Executive Officer (CEO) of the record company requires the cost-benefit analysis of
every project to be recorded in a document called a 'project rationale'. The executive is
preparing the draft 'project rationale' as part of the pre-project phase.

Question Booklet Which principle is being applied, and why?

Multiple Choice A. 'Continued business justification', because the justification for starting the project needs
to be recorded in some form of a business case
Examination Duration: 2 hours and 30 minutes B. 'Continued business justification', because it is the executive who drafts the outline
business case at the beginning of a project
C. 'Learn from experience', because it is important to consider lessons from previous
Instructions projects at the beginning of a new project
D. 'Learn from experience', because the project management team should learn from more
experienced corporate management
1. You should attempt all 68 questions. Each question is worth one mark.
2. There is only one correct answer per question. 2. During the initiation stage, the Vice President stated that attendance at launch events held
on Monday evenings is low, and on previous projects this had resulted in lower album sales.
3. You need to answer 38 questions correctly to pass the exam. As a result, the launch event for this album will be held later in the week.
4. Mark your answers on the answer sheet provided. Use a pencil (NOT pen). Which principle is being applied, and why?
5. You have 2 hours and 30 minutes to complete this exam. A. 'Continued business justification', because there is sufficient reason to start this project
6. open book’ exam. You can use the Managing Successful Projects with PRINCE2
This is an ‘o B. 'Continued business justification', because the project's justification should remain
(2017) Official Book. unchanged
Appendix 2

Appendix 2
C. 'Learn from experience', because project teams should learn from what occurred on
7. No other material is allowed.
Resources

Resources
similar projects
8. Read the ‘P
Project Scenario’ in the Scenario Booklet. D. 'Learn from experience', because the project should continue to learn from its own
experiences
9. Additional Information’ in the Scenario Booklet to answer some
You will also need the ‘A
questions.
10. Each question is separate. Do not use information from one question to answer another 3. Towards the end of stage 2, the project manager realized that not all of the ‘recorded sample
question. songs' would be completed before the end of the stage. The project manager decided to
move the remaining work to stage 3. This enabled the project manager to report that stage 2
was completed within time tolerance.
Is this an appropriate application of the 'manage by stages' principle, and why?
A. Yes, because moving the work to stage 3 avoided an exception situation in stage 2
B. Yes, because stage 3 is not the final stage so work can be moved from stage 2
C. No, because the project board should assess project viability on completion of work
planned for stage 2
D. No, because work in stage 3 should start while work planned for stage 2 is being
completed

260 PRINCE2® Practitioner | Copyright© PeopleCert International Ltd. PRINCE2® Practitioner | Copyright© PeopleCert International Ltd. 261
4. The executive has appointed the company's finance manager to provide business assurance, 7. The work package to create the 'artwork' will commence in stage 3. The graphic design
to monitor whether the album sales will exceed the production costs as the project company uses their own development method to deliver products. They have appointed
progresses through each stage. their Graphic Designer as the team manager to complete this work package. The project
manager will review the appointment at the end of the stage.
How well does this apply the 'manage by exception' principle, and why?
Which principle is being applied by the review, and why?
A. It applies it well, because a PRINCE2 project should structure the project into
management stages to enable approval on a stage-by-stage basis A. 'Manage by stages', because the project manager reviews the contents of the project
B. It applies it well, because the executive needs to be confident that controls and initiation documentation when planning stage 4
tolerances are being implemented effectively B. 'Manage by stages', because the project manager reviews the status of stage 3 products
C. It applies it poorly, because roles should be combined in a small project as long as there at the end of the stage
is no conflict of interest C. 'Manage by exception', because the project board delegates the authority to the project
D. It applies it poorly, because a PRINCE2 project should focus on delivering quality outputs manager on a stage-by-stage basis
rather than on the work required to deliver the products D. 'Manage by exception', because the team manager's performance should be reviewed
before authorizing the next work package

5. During the 'initiating a project' process, it was decided that the sound quality of the album
should be the same, regardless of the delivery channel used. At the beginning of stage 4, the 8. The project is in the initiation stage. The Vice President requests that management products
project manager agrees this requirement with the team manager responsible for delivering be produced in the form of slides, to be presented at project board meetings. This is in line
the album to the different channels. with company policy.
Which principle is being applied, and why? Is this an appropriate application of the 'tailor to suit the project' principle, and why?
A. 'Manage by stages', because an output-oriented project should define the products prior A. Yes, because the controls applied need to be appropriate to the organization's
to producing them governance
B. 'Manage by stages', because users are more likely to be satisfied if the products are B. Yes, because this provides control points during the project for decisions to be made
agreed at the start of the project C. No, because producing slides takes more effort than producing written documents
Appendix 2

Appendix 2
C. 'Focus on products', because the work done will contribute to the products being D. No, because applying the 'manage by exception' principle removes the need for
Resources

Resources
delivered to the required standards meetings
D. 'Focus on products', because the project manager should make key decisions prior to
the start of detailed work
BUSINESS CASE
6. During stage 3, the Music Album Project board consists of the Vice President as the executive
Here are three actions related to managing business justification for the Music Album Project.
and senior user, with no senior supplier.
Which role (A-F) should be responsible for each action?
Is this an appropriate application of the 'defined roles and responsibilities' principle, and
why? Choose only ONE role for each action. Each role can be used once, more than once, or not at all.
A. Yes, because the Vice President can represent all three primary stakeholder interests 9. Ensure that the law firm that is working on the contract A. Executive
B. Yes, because appointing the Vice President provides an explicit project management between the singer and the record company will make
team structure B. Senior user
a profit
C. No, because the Vice President cannot judge if the project can be feasibly delivered by all C. Senior supplier
supply streams
D. No, because having defined roles should help each person to answer 'what is expected 10. At a benefits review of the album launch, state whether D. Project manager
of me?' the album sales met the targets specified in the
E. Team manager
business case
F. Project assurance

11. Monitor monthly spend against the agreed budget on


behalf of the record company

262 PRINCE2® Practitioner | Copyright© PeopleCert International Ltd. PRINCE2® Practitioner | Copyright© PeopleCert International Ltd. 263
12. The Music Album Project is part of a programme to contract new singers. The 'artwork' is 17. As a result of previous lessons, this recommendation has been made: "If an executive is
being produced by an external graphic designer. The graphic designer's profit has been appointed who does not have an understanding of PRINCE2, someone with experience of
documented in the record company's business case. applying PRINCE2 should undertake business assurance." The Recording Studio Manager has
therefore been appointed as business assurance for stage 2.
Is this appropriate, and why?
Is this appropriate, and why?
A. Yes, because project costs should be recorded as part of the project's business case
B. Yes, because the project's business case should record any benefits for the project's A. Yes, because the Recording Studio Manager is responsible for the delivery of the
stakeholder 'recorded album'
C. No, because the project, as part of a programme, should be provided with the approach B. Yes, because the Recording Studio Manager has the experience required to deliver the
to the business case 'recorded album'
D. No, because the graphic designer's business justification should not be part of the C. No, because the Recording Studio Manager's business justification may conflict with the
project's business case interests of the executive
D. No, because the Recording Studio Manager will not be available throughout the project
lifecycle
13. The project is in the initiation stage. When negotiating with the record company on behalf of
the singer, the singer's agent stated that this type of music represents 3% of the total music
market. The singer's agent understands the music industry and has been asked to specify 18. The Contracts Manager has been appointed as the project manager for the Music Album
how much profit the record company should expect to make from the sales of this album. Project. In a previous job, the Contracts Manager worked as a graphic designer and, when
planning for stage 3, decided to also take on the role of team manager for the 'artwork'
Is this appropriate, and why?
production.
A. Yes, because the agent has the skills required to be responsible for forecasting the
Is this an appropriate application of the ‘organization’ theme, and why?
album sales
B. Yes, because the agent identified the size of the market for this type of music during the A. Yes, because the project manager can take on a team manager role if they have the
initiation stage specialist skills
C. No, because the senior user should be responsible for the development of the detailed B. Yes, because in a commercial environment the project manager should understand
Appendix 2

Appendix 2
business case supplier contractual obligations
Resources

Resources
D. No, because the senior user should be accountable for specifying the benefits which C. No, because the team manager should come from the graphic design company to avoid
justify the project conflicts of interest.
D. No, because the project manager should plan roles during the 'starting up a project'
process
ORGANIZATION
Here are three roles relating to the Music Album Project. QUALITY
Which individual (A-F) would be most appropriate for each role?
Here are three items of information that will be included in the project product description for
Choose only ONE individual for each role. Each individual can be used once, more than once, or the 'album ready for launch'.
not at all.
Under which heading (A-F) should they be recorded?
14. Delegated user assurance A. Vice President
Choose only ONE heading for each item of information. Each heading can be used once, more
B. Production Manager than once, or not at all.
15. Project manager
C. Production Assistant 19. 'Recorded album', 'registered artwork' and 'launch A. Purpose
event plan'
D. Marketing Manager B. Composition
16. Project support
E. Music lawyer C. Development skills
20. The singer will give final approval of the 'artwork'
F. Events Organizer required

21. The 'artwork' must comply completely with relevant D. Project-level quality
equality legislation
tolerances

E. Acceptance method

F. Acceptance responsibilities

264 PRINCE2® Practitioner | Copyright© PeopleCert International Ltd. PRINCE2® Practitioner | Copyright© PeopleCert International Ltd. 265
22. The record company must comply with music industry regulations when producing the 27. The project is approaching the end of stage 3. The project manager has invited the team
'artwork'. managers involved in stage 4 to a workshop to draft the stage 4 plan. The project manager
has asked the team managers, some of whom are external suppliers, to draft their team
Which action should the project manager take, and why?
plans beforehand in order to verify that the stage plan is achievable.
A. Record the need to meet this requirement during stage 2, because the 'artwork' will be
Is this appropriate, and why?
delivered to the specified quality criteria during stage 3
B. Record the need to meet this requirement during stage 2, because the product A. Yes, because the team plans for stage 4 should be approved by the project board before
description for the 'artwork' will specify the required quality criteria the stage begins
C. Record the requirement in the quality management approach, because compliance with B. Yes, because team plans can be created in parallel with the project manager creating the
external standards should be addressed when determining the approach to quality stage plan
D. Record the requirement in the quality management approach, because independent C. No, because team plans should be produced as part of the 'managing product delivery'
quality assurance needs to be planned at the beginning of the project process
D. No, because team plans produced by external team managers should comply with
supplier standards
23. During stage 2, the 'recorded sample songs' will be delivered and made available to
members of focus groups. Feedback from the focus groups will be used to improve the
'recorded album'. The Production Manager has asked the singer to assess the recordings of 28. In order to prepare the project plan, the project manager used the record company's
the sample songs. This has been planned as part of the quality management approach. historical data, such as the types and number of human resources who took part in the
previous music album projects. However, for the stage plans, the project manager organized
Why is this an appropriate action?
workshops with the team managers to estimate the resources required.
A. The quality that the singer expects from the 'recorded sample songs' needs to be
Is this appropriate, and why?
documented
B. The quality checking of the 'recorded sample songs' needs to be aligned with the A. Yes, because the resources that will undertake the work may estimate the work
incremental delivery approach differently
C. The acceptance criteria for the 'recorded sample songs' need to be prioritized by the B. Yes, because each workshop participant should understand their role
Appendix 2

Appendix 2
singer C. No, because initial project estimates should be accurate
Resources

Resources
D. The acceptance criteria for the 'recorded album' could change as a result of recording D. No, because one method of estimating should be used throughout the project
the sample songs

RISK
PLANS
The following risk has been recorded in the risk register:
Here are three statements that are considered when planning the Music Album Project.
"As the singer is new, there is a risk that the music album sales will not exceed the production
Which step (A-F) in PRINCE2's recommended approach to planning do they apply to? costs, leading to the benefit no longer being achievable."
Choose only ONE step for each statement. Each step can be used once, more than once, or not at In response to this risk, an alternative graphic design company will be sought to reduce costs.
all.
Here are three items of information to be included in the risk register.
24. The dependency between 'recorded sample songs' and A. Designing a plan Under which heading of the risk register (A-F) should the information be recorded?
'signed contract' should first be identified
B. Defining and analysing the Choose only ONE heading for each item of information. Each heading can be used once, more
products. than once, or not at all.
25. The Music Album Project will have four stages
C. Identifying activities and 29. The project manager has been instructed to search for A. Probability, impact and
26. A workshop will be held to identify the components a graphic design company that can produce the
dependencies expected value
that will make up the 'recorded album' artwork more cheaply
D. Preparing estimates B. Proximity

E. Preparing a schedule 30. If a graphic design company can be found that can C. Risk category
produce the artwork more cheaply, production costs
F. Documenting a plan D. Risk status
are predicted to be significantly reduced
E. Risk owner

31. Sales of the album will occur after the project has F. Risk actionee
closed

266 PRINCE2® Practitioner | Copyright© PeopleCert International Ltd. PRINCE2® Practitioner | Copyright© PeopleCert International Ltd. 267
32. During stage 3, the singer's agent informed the project manager that the singer may be 37. At the end of stage 2, the CEO decided to add a promotional video to the scope of stage 3
invited to perform at an international festival. If there is interest from an international and increase the project budget by £10,000. This amount is sufficient to resolve the issue,
audience, the record company will need extra money to expand their distribution channels. which will be managed using the recommended issue and change control procedure.
The project manager has created a provisional plan to cover the activities required, should However, after noticing that stage 3 has a cost tolerance of £12,000, the CEO decided to use
the singer be invited. this instead.
Is this appropriate, and why?
From which budget should the extension of the distribution channels be funded, and why?
A. Yes, because all types of issue should be funded from stage cost tolerance
B. Yes, because this is a problem and should be funded from stage cost tolerance
A. The change budget, because this includes the provision for unknown risks
C. No, because this is a request for change and should not be funded from stage cost
B. The change budget, because the distribution channels are being changed
tolerance
C. The risk budget, because it should be used to fund planned risk tolerances
D. No, because all requests for change should be funded from the change budget
D. The risk budget, because it should include the funds to cover a contingent plan

33. The following risk has been recorded in the risk register: 38. During stage 3, the 'artwork' has been completed. However, it does not fully meet the quality
"As the singer is new to the market, there is a threat that the music album sales will not criteria requested by the singer and in the documented product description. The cost of
corrective action will be £500 and will delay the project by a week. The team manager has
exceed the production costs, which would result in the project no longer being viable".
discussed this issue with the graphic designer and the singer. It has been agreed that the
'artwork' is good enough and will be used as it is.
The record company plans to find an alternative graphic design company, to lower the
overall production costs. Is this an appropriate approach to controlling change, and why?

Which risk response is being applied, and why? A. Yes, because the ‘artwork’ is of acceptable quality and project delay will be avoided
B. Yes, because the singer has agreed the revisions to his original quality criteria
A. 'Transfer the risk', because using a cheaper company transfers the threat to the third
C. No, because the project board must agree any change to the quality criteria
party
D. No, because corporate, programme management or the customer must approve an off-
B. 'Transfer the risk', because using a cheaper company reduces the financial impact on the
Appendix 2

Appendix 2
specification
project
Resources

Resources
C. 'Reduce a threat', because the threat is being made less likely to occur and would have
less impact PROGRESS
D. 'Reduce a threat', because the threat is being made certain by increasing the probability
of it occurring
Here are three statements relating to the Music Album Project.
In which management product (A-E) should this information be recorded?
CHANGE Choose only ONE product for each statement. Each product can be used once, more than once,
or not at all.
The launch event is planned to be held at a local hotel. A month before the event, the hotel
unexpectedly has to close. Another venue is available, but it is double the cost of the original 39. When producing the 'artwork', the Graphic Designer needs A. Work package
venue. to inform the singer which images have been selected
B. Checkpoint report
Here are three actions being taken in response to the loss of venue.
C. Highlight report
40. An incident at the recording studio caused a delay in
Which role (A-E) should be responsible for carrying them out?
recording the songs. However, this will be completed within D. End stage report
Choose only ONE role for each action. Each role can be used once, more than once, or not at all. time tolerance
E. Exception report
34. Decide whether the impact on the project benefits of A. Corporate, programme
using an alternative venue, is acceptable 41. There were problems with the power supply at the
management or the
recording studio throughout stage 2. It is recommended
customer that the studio use an alternative power supply for future
35. Obtain authority for an increase in the change budget
projects
to cover the cost of re-planning the launch event B. Executive

C. Senior user
36. Manage the issue and, if approved, arrange the
D. Project manager
alternative venue
E. Project support

268 PRINCE2® Practitioner | Copyright© PeopleCert International Ltd. PRINCE2® Practitioner | Copyright© PeopleCert International Ltd. 269
42. In stage 2, the music lawyer is a team manager working on the draft contract for the singer. 47. The project manager is preparing the project brief. A previous project had an issue with a
He usually sends an email to the project manager every two days summarizing the status of focus group member uploading sample songs to the internet without permission. The
the work. No major progress is expected over the next week, so the project manager project manager has asked the record company's cyber security expert to draft a section for
amends the work package to receive reports over the phone. the project brief that identifies measures required to prevent this from happening again.
Is this appropriate, and why? Is this appropriate, and why?
A. Yes, because a checkpoint report can be event-driven A. Yes, because the project brief should record any risks identified during the 'starting up a
B. Yes, because a checkpoint report can be an oral report project' process
C. No, because the reporting format cannot be changed during delivery B. Yes, because potential security issues that apply to the project should be considered
D. No, because only an exception report can be an oral report when developing the project brief
C. No, because it is sufficient to record the issue in the lessons log for the team manager of
the focus groups to consider
43. During the 'starting up a project' process, the project manager was told that the Production
D. No, because it is a serious issue that should be recorded in the issue register and
Assistant will not be available for the first stage. This issue needs to be managed formally.
managed formally
Which management product should be used to record this issue, and why?
A. Daily log, because it should be used to formally manage issues throughout the project
lifecycle
DIRECTING A PROJECT
B. Daily log, because the issue register is not created during the 'starting up a project'
process Here are three actions that are carried out as part of the 'directing a project' process.
C. Issue register, because it should be used to formally manage issues throughout the During which activity (A-E) should the action occur?
project lifecycle
Choose only ONE activity for each action. Each activity can be used once, more than once, or not
D. Issue register, because it should be used by the project manager to monitor issues on a
at all.
regular basis
48. Approve the work completed to record the album and A. Authorize initiation
Appendix 2

Appendix 2
the forecast to complete the 'registered artwork' and
STARTING UP A PROJECT B. Authorize the project
Resources

Resources
'launch event plan'
C. Authorize a stage or
Here ar three actions carried out during the 'starting up a project' process.
49. Ensure that there will be sufficient reviews after the exception plan
Which role (A-F) should carry them out? launch event to monitor that the album sales deliver
D. Give ad hoc direction
Choose only ONE role for each action. Each role can be used once, more than once, or not at all. the expected profit
E. Authorize project closure
44. Decide whether the Production Manager can take on A. Executive
50. Approve the forecast that the expected album sales will
the role of project manager, given the estimated time
B. Senior user exceed the production costs, which was refined when
and effort involved
the project plan was created
C. Senior supplier

45. Seek approval from corporate management that the D. Project manager
51. The project is approaching the end of stage 2. The project manager may need to consult the
investment of time and money in promoting the singer
E. Project support senior user and executive about planning the production of the 'artwork', and the 'recorded
is justified
album'. As a result, the project manager has checked their availability for the following week.
F. Project assurance
Is this appropriate as part of the 'give ad hoc direction' activity, and why?
46. Review the priority of the acceptance criteria for the
'album ready for launch' A. Yes, because the project board should provide advice to the project manager when
preparing exception reports
B. Yes, because the need for the project board to provide informal advice to the project
manager increases at the end of a stage
C. No, because highlight reports should keep the project board informed without the need
for other communications
D. No, because applying the 'manage by exception' principle should allow for the efficient
use of senior managers' time

270 PRINCE2® Practitioner | Copyright© PeopleCert International Ltd. PRINCE2® Practitioner | Copyright© PeopleCert International Ltd. 271
INITIATING A PROJECT CONTROLLING A STAGE
Here are three actions that take place during the 'initiating a project' process. Here are three actions that are carried out during the 'controlling a stage' process.
Which theme (A-F) is being applied? During which activity (A-F) should each action be carried out?
Choose only ONE theme for each action. Each theme can be used once, more than once, or not Choose only ONE activity for each action. Each activity can be used once, more than once, or not
at all. at all.

52. The project manager transfers the following statement: A. Business case 56. The project manager updates the product description A. Authorize a work package
"A similar singer may be working on another album, to for the album cover, following a concession granted by
B. Organization B. Review work package status
be released at the same time." from the daily log the project board
C. Risk C. Review the management

53. The project manager documents the statement: D. Progress 57. The project manager asks for confirmation from status
“Funding was secured from a youth development fund, project support that the quality checks of the draft
E. Plans contract have been carried out, as reported in the D. Report highlights
which must be used to produce the initial sample
recordings” F. Quality checkpoint report E. Escalate issues and risks

F. Take corrective action


58. The project manager asks project support to confirm
54. The singer's agent checks the project initiation
the status of the sample songs, prior to preparing the
documentation to ensure that the singer's needs will
regular progress report
be met

55. The project manager has recommended that a highlight report should be submitted every 59. An external recording studio will be used to record the sample songs from the start of stage
four weeks. However, as the project manager only joined the company recently, the 2. Therefore, the record company's purchasing department needs to carry out the supplier
Appendix 2

Appendix 2
executive wants to receive a highlight report every week while the sample songs are being selection process during the initiation stage. The project manager has recommended that
Resources

Resources
recorded. As a result, the project manager has recorded this requirement in the controls the 'controlling a stage' process is used to control the work of the purchasing department.
section of the project initiation documentation. Is this appropriate, and why?
Is this appropriate, and why? A. Yes, because work packages should be used to manage work during the initiation stage
A. Yes, because the project board uses highlight reports to monitor progress during B. Yes, because using a work package will help to ensure that the output is delivered on
management stages time
B. Yes, because the reporting should be more frequent when a team is inexperienced, to C. No, because the 'controlling a stage' process should be used for work within delivery
build confidence stages
C. No, because the frequency of highlight reports should be set in each stage plan to allow D. No, because the team manager for the sample songs production should select the
a different level of monitoring recording studio
D. No, because the frequency of highlight reporting should be specified in the
communication management approach
MANAGING PRODUCT DELIVERY
60. What should the Recording Studio Manager do as part of the 'accept a work package' activity
for the 'recorded album'?

A. Agree when the 'recorded album' needs to be completed


B. Report the amount spent when producing the 'recorded album'
C. Verify that the required sound quality checks have been completed
D. Report progress on acceptance of the work package using a checkpoint report

272 PRINCE2® Practitioner | Copyright© PeopleCert International Ltd. PRINCE2® Practitioner | Copyright© PeopleCert International Ltd. 273
61. The team manager for the 'signed contracts for delivery channels' is in the process of 65. Late in stage 3, the project manager has reported that the 'artwork' is going to take longer to
accepting the work package. The team manager is concerned that the list of quality produce than planned, and the stage is likely to exceed time tolerance. As a result, the
reviewers included in the product description may not include anyone with the required project board has requested an exception plan and also wants to establish the status of the
specialist knowledge. current stage.
What should the team manager do first? What action should the project manager take, and why?
A. Consult with project assurance A. Prepare an exception report, because it should show the status of the work package
B. Raise a risk with the project manager B. Prepare an exception report, because it should describe the options for dealing with the
C. Request a resource from the senior supplier deviation
D. Revise the product description C. Prepare an end stage report, because the project board have asked what is outstanding
62. The project is in stage 2. The 'recorded sample songs' have been produced and handed over for stage 3
to the team manager for the focus groups. On the day before the focus group meeting, the D. Prepare an end stage report, because the project is nearing the end of stage 3
team manager discovered that the sound quality of one song was not of the required
standard. The team manager spoke to the singer's agent, and the singer will attend the focus
group meeting to perform the songs. CLOSING A PROJECT
Is this appropriate application of the 'managing product delivery' process, and why? 66. Which action should the project manager take during the 'prepare planned closure' activity?
A. Yes, because this will 'exploit' the opportunity for the focus group to hear the singer A. Summarize the final amount spent in producing the 'recorded album', for future
perform the songs reference
B. Yes, because the team manager is taking corrective action to resolve the issue of the B. Check the 'album ready for launch' to confirm that it meets the record company's
poor-quality recordings requirements
C. No, because the team manager should ask the Recording Studio Manager to take C. Report on the number of downloads for the sample songs and the predicted sales
corrective action D. Identify the marketing activities that still need to take place to support the launch
D. No, because an issue should be raised so that the project manager can decide on
corrective action 67. Which action should the project manager take during the 'hand over products' activity?
Appendix 2

Appendix 2
A. Update the project plan with the actual time taken to plan the launch event
Resources

Resources
B. Check whether the graphic designer can be released to work on another project
MANAGING A STAGE BOUNDARY C. Summarize whether the 'album ready for launch' was delivered on time and to cost
D. Review the dates when sales of the new album will be measured
63. The project is approaching the end of stage 3 and the 'artwork' is taking longer to produce
than expected. As a result, an exception report has been sent to the project board. The
68. The 'launch event plan' has been completed on time and within budget. A quality review has
project board has decided to follow the recommendation of the project manager to increase
been carried out and there are no outstanding issues. The group running the launch event
the time tolerance for the stage.
has confirmed that the plan meets their needs and that they will be able to run the launch
When should the 'managing a stage boundary' process be used next? event.
A. When preparing the stage 4 plan for approval by the project board Who will use this information, and when?
B. When reporting that stage 3 is now progressing according to plan
A. Project manager, when updating the end project report with lessons
C. When re-planning stage 3 in response to the increased time tolerance
B. Project manager, when identifying follow-on action recommendations
D. When the performance of the whole Music Album Project is reviewed
C. Project support, when creating the product status account
D. Project support, when transferring responsibility for the launch event plan
64. This project is part of a larger programme. An exception plan has been produced during the
final stage because there has been a delay in the registering of the 'artwork'. The stage will
be delayed, however the project will be completed within the time tolerance set.
Which role, or combination of roles, should approve this exception plan?
A. Programme board
B. Project board
C. Project manager with project assurance
D. Both project board and programme board

274 PRINCE2® Practitioner | Copyright© PeopleCert International Ltd. PRINCE2® Practitioner | Copyright© PeopleCert International Ltd. 275
Sample Paper 1: Answers and Rationales Q A Syllabus Ref Rationale
3 C 1.1.1d A. Incorrect. Application of the 'manage by stages' principle
Q A Syllabus Ref Rationale
provides review and decision points for the project board at
1 A 1.1.1a A. Correct. For all projects, PRINCE2 requires a justifiable defined intervals. The project manager cannot decide to move
reason for starting the project, which must be recorded. work from stage 2 to stage 3, as this is outside the project
The format and formality of documentation may vary, manager's delegated authority. This is also against the
depending on organizational standards, needs and principle of 'manage by exception', as exceeding time
circumstances. Ref 3.1 tolerance for the stage must be reported. Ref 3.4, 3.5
B. Incorrect. The principle being applied is 'continued business B. Incorrect. Application of the ‘manage by stages’ principle
justification'. It is true that the executive may draft the outline provides review and decision points for the project board at
business case, however this does not explain why 'continued defined intervals, rather than letting the project run on in an
business justification' is being applied in this situation. Ref 3.1, uncontrolled way. The project board authorizes one stage of
14.4.4 the project at a time against a stage plan. Although the project
C. Incorrect. The principle being applied is not 'learn from manager has discretion to make adjustments, this would not
experience'. In the situation given, there is no description of a include amending a stage baseline, such as moving work from
lesson having been learnt from the current project or outside. one stage to another. Ref 3.4
Ref 3.1, 3.2 C. Correct. Application of the ‘manage by stages’ principle
D. Incorrect. The project should be aligned with the provides review and decision points for the project board
commissioning organization’s strategy, and the project at defined intervals, rather than letting the project run on
management team members are expected to follow the in an uncontrolled way. This is why the planned work that
guidelines set by their corporate management. However, the remains in stage 2 cannot be moved to stage 3, without
'learn from experience' principle is not being applied in this the approval of an exception. Ref 3.4
situation as there is no description of a lesson having been D. Incorrect. Application of the ‘manage by stages’ principle
learnt from the current project, or outside. Ref 3.1 provides review and decision points for the project board at
defined intervals. The project board authorizes one stage of
2 C 1.1.1b A. Incorrect. The situation describes learning from the
Appendix 2

Appendix 2
the project at a time against a stage plan. Although the project
experience of a similar launch event, and is not related to the
Resources

Resources
manager has discretion to make adjustments, this would not
'continued business justification' principle. The 'continued
include amending a stage baseline, such as moving work from
business justification' principle aims to ensure that the project
one stage to another. Delivery steps often overlap but stages
remains aligned to the benefits being sought that contribute to
cannot. Ref 3.4, 9.3.1.1
the business objectives. Ref 3.2, 3.1
B. Incorrect. The situation describes learning from the 4 B 1.1.1e A. Incorrect. PRINCE2 breaks the project down into discrete,
experience of a similar launch event, and is not related to the sequential sections, called management stages, as part of the
'continued business justification' principle. The 'continued ‘manage by stages’ principle. However, this does not explain
business justification' principle aims to ensure that the project why the project assurance role is required as part of the
remains aligned to the benefits being sought that contribute to ‘manage by exception’ principle. Ref 3.4
the business objectives. Ref 3.2, 3.1 B. Correct. As part of the ‘manage by exception’ principle,
C. Correct. When starting a project, previous or similar an assurance mechanism should be put in place so that
projects should be reviewed to see if lessons can be each management layer can be confident that controls are
applied. It is the responsibility of everyone involved with effective. Ref 3.5
the project to look for lessons rather than wait for C. Incorrect. As part of the ‘tailor to suit the project’ principle,
someone else to provide them. The project manager roles may be combined or split, provided that accountability is
should communicate with the events coordinator to find maintained and there are no conflicts of interest, however this
out more about the timing of the previous event. Ref 3.2 is not mandatory. Ref 4.3.1
D. Incorrect. The project is in the initiation stage and so this is D. Incorrect. As part of the ‘focus on products’ principle,
learning from experience at the start of a project, not learning PRINCE2 requires projects to be output-oriented rather than
as the project progresses. As the project moves into the work-oriented. However, this does not explain why the project
management stages after initiation the project should assurance role is required as part of the ‘manage by exception’
continue to learn. Lessons should be included in relevant principle. Ref 3.6
reports and reviews. The goal is to seek opportunities to
implement improvements during the life of the project. Ref 3.2

276 PRINCE2® Practitioner | Copyright© PeopleCert International Ltd. PRINCE2® Practitioner | Copyright© PeopleCert International Ltd. 277
Q A Syllabus Ref Rationale Q A Syllabus Ref Rationale
5 C 1.1.1f A. Incorrect. The 'focus on products' principle requires projects 7 A 1.1.1d A. Correct. Applying the 'manage by stages' principle
to be output-oriented, not the principle 'manage by stages'. Ref ensures that key decisions are made prior to the detailed
3.6 work needed to implement them. For that reason, the
B. Incorrect. Application of the 'focus on products' principle, project manager reviews the components of the project
not 'manage by stages', reduces the risk of user dissatisfaction initiation documentation as part of planning the next
by ensuring that there is agreement about what will be stage. Ref 3.4, 19.4.1, A.20.2
produced. Ref 3.6 B. Incorrect. The principle being applied is ‘manage by stages’.
C. Correct. Ensuring that the work package includes the However, when reviewing the status of stage 3, the project
requirements for the product(s) to be delivered is an manager is not required to wait until the end of it. Using the
application of the 'focus on products' principle. This helps 'controlling a stage' process, the project manager reviews the
to ensure that the team only carries out work that directly stage status throughout the stage. Ref 3.4, 17.4.4
contributes to the delivery of the desired product(s). Ref C. Incorrect. The principle being applied is ‘manage by stages’,
3.6 not ‘manage by exception’. Although it is true that the project
D. Incorrect. Application of the 'manage by stages' principle, board delegates the authority to the project manager on a
not 'focus on products', ensures that key decisions are made stage-by-stage basis, the situation described is not concerned
prior to commencing detailed work. Ref 3.4, 3.6 with delegation or deviation from any project objective. Ref
3.4, 3.5
6 C 1.1.1c A. Incorrect. The Vice President cannot represent all suppliers,
D. Incorrect. The principle being applied is ‘manage by stages’,
especially as some are from third-party organizations. All three
not ‘manage by exception’. The project manager reviews the
stakeholder interests must be represented - two out of three
team’s performance as part of the ‘report management stage
stakeholder interests are not enough. Ref 3.3
end’ activity and is expected to assess the team manager’s
B. Incorrect. To be successful, projects must have an explicit
performance before delegating the authority to work on the
project management team structure. However, this requires all
next relevant work package, as part of applying the ‘learn from
three stakeholder interests (business, user and supplier) to be
experience' principle. However, the situation described is not
represented, which would not be the case if the project board
Appendix 2

Appendix 2
concerned with delegation or deviation of any project
has no supplier representation. Ref 3.3
Resources

Resources
objective. Ref 3.2, 3.4, 3.5, 19.4.4
C. Correct. The Vice President cannot represent all
suppliers, especially as some are from third-party 8 A 1.1.1g A. Correct. This is a correct application of the ‘tailor to suit
organizations. All three stakeholder interests must be the project’ principle. The purpose of tailoring is to ensure
represented - two out of three stakeholder interests are that the project controls are appropriate to the project’s
not enough. Ref 3.3 scale, complexity, importance, team capability and risk.
D. Incorrect. Having roles and responsibilities defined helps Ref 3.7
each person to know what is expected of them. However, this B. Incorrect. This is a correct application of the ‘tailor to suit the
does not explain why having no senior supplier on the project project’ principle. However, it is the application of the ‘manage
board is not appropriate. Ref 3.3 by stages’ principle that provides review and decision points,
giving the project board the opportunity to assess the project’s
viability at defined intervals, rather than let it run on in an
uncontrolled manner. Ref 3.4
C. Incorrect. This is a correct application of the ‘tailor to suit the
project’ principle. Tailoring requires the project board and the
project manager to make pro-active choices and decisions on
how PRINCE2 will be applied. When tailoring PRINCE2, it is
important to remember that effective project management
requires information (not necessarily documents). Therefore, it
is appropriate to produce slides, irrespective of the effort
involved. Ref 3.7
D. Incorrect. This is a correct application of the ‘tailor to suit
the project’ principle as management products should be
tailored to the requirements and environment of each project
and can be in the form of slides. Also, the 'manage by
exception' principle does not remove the need for meetings,
although it does provide for efficient use of senior
management time. Ref 3.5, App A

278 PRINCE2® Practitioner | Copyright© PeopleCert International Ltd. PRINCE2® Practitioner | Copyright© PeopleCert International Ltd. 279
Q A Syllabus Ref Rationale Q A Syllabus Ref Rationale
9 C 2.1.1b C. Correct. The law firm is an external supplier and 14 D 2.2.1c D. Correct. The Marketing Department will use the product by
therefore the senior supplier should be accountable for marketing the album. The Marketing Manager is the right
the supplier’s business case. Ref tab 6.1 person to undertake the user assurance role by assessing
A, B, D, E, F. Incorrect. Ref tab 6.1 whether the album meets the user's needs. Ref C.7.1, 7.2.1.5
A, B, C, E, F. Incorrect. Ref 7.1, C.5, C.7.1, C.9.1
10 B 2.1.1b B. Correct. At benefit reviews the senior user should
provide statements of actual benefit achievements 15 B 2.2.1c B. Correct. The Production Manager has experience of
compared to forecast benefits. Ref tab 6.1 managing album projects and, of the options that are
A, C, D, E, F. Incorrect. Ref tab 6.1 provided, would be most appropriate for this role. Ref C.5
A, C, D, E, F. Incorrect. Ref 7.1, C.5, C.7.1, C.9.1
11 F 2.1.1b F. Correct. Project assurance should monitor project
finance on behalf of corporate, programme management 16 C 2.2.1c C. Correct. Provision of administrative support is one
or the customer. Ref tab 6.1 responsibility of project support. Ref C.9.1
A, B, C, D, E. Incorrect. Ref tab 6.1 A, B, D, E, F. Incorrect. Ref 7.1, C.5, C.7.1, C.9.1
12 D 2.1.2 A. Incorrect. The customer's business case should include all 17 C 2.2.2 A. Incorrect. The Recording Studio Manager is responsible for
costs. However, the graphic designer's business justification the delivery of the recorded album. However, business
should not be part of the project's business case. Ref A.2.2, assurance should be undertaken by someone from the
6.3.3 customer organization, to avoid a conflict of interest. Ref
B. Incorrect. The customer's business case should include the 7.2.1.10
benefits to the customer, but the graphic designer's business B. Incorrect. Although the Recording Studio Manager has the
justification should not be part of the project's business case. required knowledge and experience, the business assurance
Ref A.2.2, 6.3.3 role should be undertaken by someone from the customer
C. Incorrect. It is true that where the project is part of the organization. The Recording Studio Manager is a supplier and
programme, the programme will typically define both the therefore this would be a conflict of interest. Ref 7.2.1.10
approach to business case development and provide an C. Correct. There may be conflict between customer and
Appendix 2

Appendix 2
outline business case for the project. However, this does not supplier business justification. The business assurance
Resources

Resources
explain why the supplier should have a separate business role should be undertaken by someone from the customer
case. Ref 6.3.4 organization as the Recording Studio Manager is a
D. Correct. The project’s business case for a customer’s supplier. Ref 7.2.1.10
project is separate from a supplier’s business justification D. Incorrect. Ideally, those with project assurance
for bidding for, and working on that customer’s project. responsibilities should be able to carry out the role throughout
The customer needs to ensure that their project is viable the project, although it is possible for someone to provide
and risks are acceptable, bearing in mind the suppliers project assurance during a specific stage. This is not the reason
chosen. A supplier would have to ensure that they will why it is not appropriate for the Recording Studio Manager to
benefit from the work they undertake on the project. Ref undertake the role. Business assurance should be undertaken
6.3.3 by someone from the customer organization to avoid a conflict
of interest. Ref 7.2.1.10, C.7.2
13 D 2.1.2 A. Incorrect. Even though the singer's agent has experience of
the market and skills to forecast sales, it is the senior user who
specifies the benefits for the project. Ref tab 6.1
B. Incorrect. Even though the singer's agent understands the
size of the market, it is the senior user who specifies the
benefits for the project. Ref tab 6.1
C. Incorrect. The project manager is responsible for the
development of the business case, but the benefits are
specified by the senior user. Ref tab 6.1, C.5
D. Correct. The singer's agent is an external supplier who
cannot be responsible for the benefits in the record company's
business case. The senior user is accountable for specifying the
benefits. Ref tab 6.1, C.3

280 PRINCE2® Practitioner | Copyright© PeopleCert International Ltd. PRINCE2® Practitioner | Copyright© PeopleCert International Ltd. 281
Q A Syllabus Ref Rationale Q A Syllabus Ref Rationale
18 A 2.2.2 A. Correct. The project manager can always choose to be 22 C 2.3.2 A. Incorrect. The project may be subject to external quality
the team manager. In a commercial environment, the standards, for example when the project is within a regulated
supplier’s staff may fulfil a team manager role on the environment. These various circumstances must be addressed
project - this is not mandatory and can introduce a conflict when determining the project’s approach to quality. It would
of interest. Ref 7.2.1.8, 7.3.4 be too late to consider this requirement during stage 2. Ref
B. Incorrect. It is important that the project manager has a 8.3.2, A.20.2
good understanding of the supplier contractual obligations in a B. Incorrect. The project may be subject to external quality
commercial environment. However, this does not explain why standards, for example when the project is within a regulated
the project manager and team manager roles may be environment. These various circumstances must be addressed
combined in the situation described. Ref. 7.3.4 when determining the project’s approach to quality. It would
C. Incorrect. The graphic design company is a third-party be too late to consider this requirement during stage 2. Ref
supplier and there could be a reporting line between the team 8.3.2, A.20.2
manager and the senior supplier. This link would need to be C. Correct. The project may be subject to external quality
understood in order to avoid conflicts of interest. However, standards, for example when the project is within a
appointing the team manager from the third-party supplier regulated environment. These various circumstances must
would introduce the conflict of interest, not avoid it. Ref 7.2.1.8 be addressed when determining the project’s approach to
D. Incorrect. The project manager should plan roles during the quality. Ref 8.3.2, A.20.2
'starting up a project' process. However, this does not explain D. Incorrect. Quality assurance is defined in the quality
why the project manager and team manager roles may be management approach. However, this is not why the quality
combined in the situation described. Ref 7.2.1.8, 14.4.3 standards need to be identified in the quality management
approach. Ref A.20.2
19 B 2.3.1a B. Correct. The 'composition' heading includes a
description of the major products and/or outcomes to be 23 B 2.3.2 A. Incorrect. The customer’s quality expectations should be
delivered by the project. Ref A.21.2 agreed early in the 'starting up a project' process. The
A, C, D, E, F. Incorrect. Ref A.21.2 expectations are captured in discussions with the customer
Appendix 2

Appendix 2
and then refined for inclusion in the project product
Resources

Resources
20 F 2.3.1a F. Correct. The 'acceptance responsibilities' heading
description, rather than the quality management approach.
defines who will be responsible for confirming acceptance.
The singer is not the customer, even though he is representing
PRINCE2 uses the term ‘acceptance’ to describe the
the customer during the quality checking. Ref 8.3.6
ultimate approval of the project’s product. Ref A.21.2,
B. Correct. It is important that the approach to managing
8.3.17
quality works with, and supports, the chosen delivery
A, B, C, D, E. Incorrect. Ref A.21.2
approach, and not against it. For example, when using an
21 D 2.3.1a D. Correct. The 'project-level quality tolerances' heading agile approach, the high frequency of quality checking (in
specifies any tolerances that may apply for the acceptance the form of reviews, demos or tests) may have a
criteria. In this case the tolerance is zero. Ref A.21.2 significant impact on how a project is planned. This will
A, B, C, E, F. Incorrect. Ref A.21.2 affect the incremental delivery of the project’s products
and how they are released. Ref 8.3.3
C. Incorrect. It is true that the acceptance criteria need to be
prioritized by the customer. However, the singer is not the
customer and this also does not explain why the quality
management approach needs to take into account the
incremental delivery approach. Ref 8.3.8, 8.3.3
D. Incorrect. It is true that acceptance criteria could evolve as a
result of the initial sample recordings. However, this does not
explain why the approach to quality needs to work with and
support the chosen delivery approach. Ref 8.3.9, 8.3.3
24 B 2.4.1c B. Correct. As part of PRINCE2's recommended approach to
defining and analysing the products, a product flow
diagram is created to identify the sequence in which the
products of the plan will be developed. Ref 9.3.1.2
A, C, D, E, F. Incorrect. Ref 9.3.1.1-7

282 PRINCE2® Practitioner | Copyright© PeopleCert International Ltd. PRINCE2® Practitioner | Copyright© PeopleCert International Ltd. 283
Q A Syllabus Ref Rationale Q A Syllabus Ref Rationale
25 A 2.4.1c A. Correct. One of the decisions made during the 29 F 2.5.1a F. Correct. The risk actionee is the person(s) who will
'designing a plan' activity is about the number of implement the action(s) described in the risk response.
management stages in the project. Ref 9.3.1.1 This may or may not be the same person as the risk
B, C, D, E, F. Incorrect. Ref 9.3.1.1-7 owner. Ref A.25.2
A, B, C, D, E. Incorrect. Ref A.25.2
26 B 2.4.1c B. Correct. As part of PRINCE2’s recommended approach to
defining and analysing the products, a product breakdown 30 A 2.5.1a A. Correct. Probability, impact and expected value. It is
structure is created. This is when an approach such as helpful to estimate the inherent values (pre-response
brainstorming is chosen to identify products. Ref 9.3.1.2 action) and residual values (post-response action). These
A, C, D, E, F. Incorrect. Ref 9.3.1.1-7 should be recorded in accordance with the project’s
chosen scales. Ref A.25.2
27 B 2.4.2 A. Incorrect. Team plans can be produced in parallel with the
B, C, D, E, F. Incorrect. Ref A.25.2
project manager producing the stage plan. However, team
plans are not approved by the project board. The project 31 B 2.5.1a B. Correct. Proximity typically states how close to the
manager authorizes a work package. Ref 9.2.1.4, tab 12.2 present time the risk event is anticipated to happen (e.g.
B. Correct. Team plans can be produced in parallel with imminent, within the management stage, within the
the project manager producing the stage plan. This can be project, beyond the project). Proximity should be recorded
especially helpful where the project manager has little in accordance with the project’s chosen scales. Ref A.25.2
knowledge of the development area, however this does A, C, D, E, F. Incorrect. Ref A.25.2
not mean the team plans have been approved. Ref 9.2.1.4
32 D 2.5.2 A. Incorrect. Provisions for unknown risks should be funded
C. Incorrect. Team plans can be produced in parallel with the
from the risk budget, not the change budget. Ref 10.3.7
project manager producing the stage plan, as part of the
B. Incorrect. The distribution channels will only be changed if
'managing a stage boundary' process. However, it is true that
the singer is invited to the international festival and if there is
team plans are typically produced during the 'managing
an increase in international interest. Therefore, this is a
product delivery' process. Ref 9.2.1.4
contingent plan which should be funded from the risk budget,
Appendix 2

Appendix 2
D. Incorrect. Teams may be from separate organizations that
not the change budget. Ref tab 10.3, 10.3.7
Resources

Resources
may follow different project management methods. However,
C. Incorrect. Risk tolerances should be recorded in the risk
this does not mean that the team plans cannot be produced in
management approach, the stage plan or even in the work
parallel with the stage plan. Ref 9.2.1.4
package, not in the risk budget. Ref tab 12.1
28 A 2.4.2 A. Correct. Estimates are usually based on consultation D. Correct. The change to the distribution channels is a
with the resources, who will undertake the work, and/or contingent plan to be carried out, if the opportunity
historical data. Estimates should be challenged, as work materializes that the singer is invited to the festival and
can be estimated differently by various estimators or by there is an increase in international interest. Management
the same estimator at different times. Therefore, it is responses to project threats and opportunities should be
appropriate to consult with the resources who will funded from the risk budget, if this has been established
undertake the work, in order to also take into account within the project's budget. Ref tab 10.3, 10.3.7
their estimates. Ref 9.3.1.4, A.16.5
B. Incorrect. The purpose of the workshops described is to
estimate the work to be done in each stage in consultation
with the resources who will undertake the work. The purpose
is not to ensure that those undertaking the project understand
their role. Ref 9.3.1.4, A.16.5
C. Incorrect. The project manager’s course of action is
appropriate. No estimating can guarantee accuracy. Estimates
will inevitably change as more is discovered about the project.
Ref 9.3.1.4, A.16.5
D. Incorrect. The project manager’s course of action is
appropriate. Estimates are usually based on consultation with
the resources, who will undertake the work, and/or historical
data. There is no need to use the same estimation methods
throughout the project. Ref 9.3.1.4, A.16.5

284 PRINCE2® Practitioner | Copyright© PeopleCert International Ltd. PRINCE2® Practitioner | Copyright© PeopleCert International Ltd. 285
Q A Syllabus Ref Rationale Q A Syllabus Ref Rationale
33 C 2.5.2 A. Incorrect. 'Transfer the risk' is an option that aims to pass 38 C 2.6.2 A. Incorrect. Accepting an off-specification product without
part of the risk to a third party. Transferring a risk is not amendment is known as a concession and must be approved
automatic and the cost of transference must be justified in by the project board, or its delegated change authority. Ref tab
terms of the change to residual risk - is the premium to be paid 11.3
worth it? However, this response does not transfer the risk to B. Incorrect. Accepting an off-specification product without
the third party, it merely reduces it. Ref tab 10.3 amendment is known as a concession and must be approved
B. Incorrect. 'Transfer the risk' is an option that aims to pass by the project board, or its delegated change authority.
part of the risk to a third party. Transferring a risk is not Although the singer set the quality criteria, the request for a
automatic and the cost of transference must be justified in concession must be referred to the project board as this is a
terms of the change to residual risk - is the premium to be paid request for change. Ref tab 11.3
worth it? However, this response does not transfer the risk to C. Correct. Accepting an off-specification product without
the third party, it merely reduces it. Ref tab 10.3 amendment is known as a concession and must be
C. Correct. 'Reduce a threat' is when a definite action is approved by the project board or its delegated change
taken to change the probability and/or the impact of the authority. Ref tab 11.3
risk. Reducing production costs by finding a cheaper D. Incorrect. Accepting an off-specification product without
supplier would make it less likely that the costs exceed amendment is known as a concession. Concessions may be
the sales, and should reduce the amount by which they granted by the project board, or its delegated change authority
exceed sales, if the threat materializes. Ref tab 10.3 without requesting the approval of corporate, programme
D. Incorrect. 'Avoid a threat', not 'reduce a threat', is a management or the customer. Ref tab 11.3
response that makes the uncertain situation certain by
39 A 2.7.1a A. Correct. The requirement for the graphic designer to
removing the threat. This can often be achieved by removing
inform the singer of the selected images represents the
the cause. Ref tab 10.3
development interface in the work package to produce the
34 C 2.6.1b C. Correct. The senior user makes decisions on escalated 'artwork'. Ref A.26.2
issues with particular focus on safeguarding the expected B, C, D, E. Incorrect. Ref 12.2.2.4, 12.2.2.3
Appendix 2

Appendix 2
benefits. Ref tab 11.2
Resources

Resources
40 B 2.7.1a B. Correct. Reporting of the delay in recording one song to
A, B, D, E. Incorrect. Ref tab 11.2
the next week (reporting period) will be included in the
35 B 2.6.1b B. Correct. The executive is responsible for securing the checkpoint report. The delay did not cause a tolerance
funding for the project and determining the change breach. A.4.2
budget. Ref tab 11.2, C.2.1 A, C, D, E. Incorrect. Ref 12.2.2.4, 12.2.2.3
A, C, D, E. Incorrect. Ref tab 11.2
41 D 2.7.1a D. Correct. The recommendation to use an independent
36 D 2.6.1b D. Correct. The project manager manages the issues and power supply following the interruptions during a stage
may implement corrective actions. Ref tab 11.2 will be included as a lesson learned in the end stage report
A, B, C, E. Incorrect. Ref tab 11.2 for stage 2. Ref A.9.2, 12.2.2.3
A, B, C, E. Incorrect. Ref 12.2.2.4, 12.2.2.3
37 C 2.6.2 A. Incorrect. The action taken is not appropriate because cost
tolerance should not be used to fund requests for change. Ref 42 B 2.7.2 A. Incorrect. A checkpoint report can be in oral format and is a
11.1, 11.3.6 time-driven control (not an event-driven control). It takes place
B. Incorrect. The action taken is not appropriate. Also, the issue at predefined periodic intervals, showing the progress of a
described is a request for change, not a problem. A request for work package. Ref 12.2.2
change can be funded by increasing project budget or by de- B. Correct. A checkpoint report is used to report to the
scoping other elements of the project as well, if required. Ref project manager on the status of the work package. A
11.1, 11.3.6 checkpoint report can take a number of formats, including
C. Correct. A request for a change is an issue that proposes an oral report in person or over the phone. Ref A.4.4
a change to a baseline. A change of project scope is, C. Incorrect. A checkpoint report can be in oral format.
therefore, a request for change and tolerances should not However, a change in the reporting format and frequency can
be used to fund requests for change. Ref tab 11.1, 11.3.6 be agreed by the relevant authority (project manager/project
D. Incorrect. The action taken is not appropriate. However, a board) at any time, if there is an appropriate reason. Ref A.4.4,
request for change is not always funded from the change 17.4.1
budget. It can be funded by increasing project budget or by de- D. Incorrect. A checkpoint report can be in oral format. For
scoping other elements of the project as well, if required. Ref urgent exceptions, the exception report can be in oral format
11.1, 11.3.6 in the first instance, followed up in the agreed format. Ref
A.4.4, A.10.4

286 PRINCE2® Practitioner | Copyright© PeopleCert International Ltd. PRINCE2® Practitioner | Copyright© PeopleCert International Ltd. 287
Q A Syllabus Ref Rationale Q A Syllabus Ref Rationale
43 B 2.7.2 A. Incorrect. The daily log is used to record issues until the 47 B 3.1.2 A. Incorrect. During the 'starting up a project' process, any
issue register becomes available. However, it is not used to security constraints that apply to the project or the operation
formally manage issues once the issue register has been of its products should be considered. However, it is the daily
created during the initiation stage. Ref A.7.1, A.12.1, 16.4.3, tab log, not the project brief that should be used to record any
16.3 new issues or risks. Ref 14.4.5
B. Correct. The issue register is not created in the 'starting B. Correct. During the 'starting up a project' process, any
up a project' process. The daily log is used to record issues security constraints that apply to the project or the
until the issue register becomes available. Ref A.7.1, 14.4.1, operation of its products should be considered. Ref 14.4.5
tab 16.3 C. Incorrect. During the 'starting up a project' process, any
C. Incorrect. The issue register is not created in the 'starting up security constraints that apply to the project or the operation
a project' process. The daily log is used to record issues until of its products should be considered. Although recording this
the issue register becomes available. However, the issue in the lessons log might be sufficient, it is not inappropriate to
register is used to formally manage issues, once it has been consult the cyber security expert and include information in
created. Ref A.7.1, 14.4.1, 16.4.3, tab 16.3 the brief. Ref 14.4.5
D. Incorrect. The issue register is not created in the 'starting up D. Incorrect. During the 'starting up a project' process, any
a project' process. The daily log is used to record issues until security constraints that apply to the project or the operation
the issue register becomes available. However, the issue of its products should be considered. It cannot be recorded in
register should be used by the project manager to monitor the issue register as the issue register is not created in the
issues on a regular basis. Ref A.7.1, A.12.1, 14.4.1, 16.4.3, tab ‘starting up a project’ process. Ref 14.4.5, tab 16.3
16.3
48 C 3.2.1a C. Correct. As part of the 'authorize a stage or exception
44 A 3.1.1b A. Correct. As part of the 'appoint the executive and plan' activity, the end stage report should be reviewed and
project manager' activity in the 'starting up a project' approved. This is to ascertain the performance of the
process, the executive appoints the project manager and project to date, asking the project manager to explain any
estimates the time and effort required for the project deviations from the approved plans, and to provide a
Appendix 2

Appendix 2
manager role. Ref 14.4.1, tab 14.1 forecast of project performance for the remainder of the
Resources

Resources
B, C, D, E, F. Incorrect. Ref 14.4.1, tab 14.1, tab 14.4 project. Ref 15.4.3
A, B, D, E. Incorrect. Ref 15.4.1-5
45 A 3.1.1b A. Correct. As part of the 'prepare the outline business
case' activity in the 'starting up a project' process, the 49 E 3.2.1a E. Correct. As part of the 'authorize project closure'
executive prepares the outline business case that goes activity, the project board should ensure that post-project
into the project brief and seeks approval from corporate benefits reviews defined by the updated benefits
management, if required. Ref 14.4.4, tab 14.4 management approach cover the performance of the
B, C, D, E, F. Incorrect. Ref 14.4.1, tab 14.1, tab 14.4 project’s products in operational use, in order to identify
whether there have been any side-effects (beneficial or
46 F 3.1.1b F. Correct. As part of the 'prepare the outline business
adverse). Ref 15.4.5
case' activity in the 'starting up a project' process, project
A, B, C, D. Incorrect. Ref 15.4.1-5
assurance reviews the project product description. The
project product description will include prioritized 50 B 3.2.1a B. Correct. The outline business case produced during the
acceptance criteria. Ref 14.4.4, tab 14.4, 8.1 'starting up a project' process needs to be updated to
A, B, C, D, E. Incorrect. Ref 14.4.1, tab 14.1, tab 14.4 reflect the estimated time and costs, as determined by the
project plan. The objective of the 'authorize the project'
activity is to decide whether to proceed with the rest of
the project. The project board has to confirm that an
adequate and suitable business case exists and that it
shows a viable project. Ref 16.4.8, 15.4.2
A, C, D, E. Incorrect. Ref 15.4.1-5

288 PRINCE2® Practitioner | Copyright© PeopleCert International Ltd. PRINCE2® Practitioner | Copyright© PeopleCert International Ltd. 289
Q A Syllabus Ref Rationale Q A Syllabus Ref Rationale
51 B 3.2.2 A. Incorrect. In response to informal requests for advice and 55 B 3.3.2 A. Incorrect. The project board does use highlight reports to
guidance as part of the ‘give ad hoc direction’ activity, the monitor and control management stages and project progress,
project board should assist the project manager as required however this is not the reason why it is appropriate to set the
(this may include asking the project manager to produce an frequency of the reports to weekly in this case, and record this
issue report and/or an exception report). However, this does information in the project initiation documentation (PID). Ref
not explain why the project board should 'give ad hoc advice' A.20.2, 16.4.6, 12.2.2.4, 19.4.1
as required, at the end of a stage. Ref 15.4.4 B. Correct. It is in the PID that the frequency of the
B. Correct. Project board members may offer informal highlight reports for the duration of the project would be
guidance or respond to requests for advice at any time stated, with an understanding that for each stage in the
during a project. The need for consultation between the stage plan the frequency for highlight reports would be
project manager and project board is likely to be agreed. Each stage may need a different level of control
particularly frequent during the initiation stage, and when and more or less frequent reports. As the confidence in
approaching stage boundaries. Ref 15.4.4 the project manager and team increases in future stage
C. Incorrect. As part of the ‘give ad hoc advice’ activity, the plans the board may agree to less frequent highlight
project board should review the highlight report to understand reports. The PID will be updated to show this. Ref A.20.2,
the status of the project. However, this does not explain why 16.4.6, 12.2.2.4, 19.4.1
the project board should give ad hoc advice as required, at the C. Incorrect. It is appropriate to record the frequency of
end of a stage. Ref 15.4.4 highlight reporting in the PID. It is true that each stage may
D. Incorrect. The application of the ‘manage by exception’ need a different level of control and more or less frequent
principle provides for very efficient use of senior management reports, but if the frequency of reporting changes, this would
time as it reduces senior managers’ time burden, without be reflected in both the stage plan for that change and in an
removing their control by ensuring decisions are made at the update to the PID. Ref A.20.2, 16.4.6, 12.2.2.4, 19.4.1
right level in the organization. However, this does not explain D. Incorrect. The action described is appropriate. Additionally,
why the project board should give ad hoc advice as required, although it is true that the communication management
Appendix 2

Appendix 2
at the end of a stage. Ref 3.5 approach states when formal communication activities are to
Resources

Resources
be undertaken (for example, at the end of a management
52 C 3.3.1c C. Correct. According to the ‘risk’ theme, the project
stage) including performance audits of the communication
manager should review the daily log for any risks and
methods, this does not explain why the frequency of reporting
populate the risk register. Ref 16.4.2, 10.2
may be varied to take into account the inexperience of team
A, B, D, E, F. Incorrect. Ref 16.4.2, 16.4.9, A.2.2
members. Ref A.5.2, 12.2.2.4
53 A 3.3.1c A. Correct. According to the ‘business case’ theme, the
56 F 3.4.1a F. Correct. The product description will need to be updated
project manager should summarize the project costs and
to reflect the concession granted by the project board.
funding arrangements in the business case. Ref 6.2, A.2.2
This is a corrective action that has been decided on, e.g. to
B, C, D, E, F. Incorrect. Ref 16.4.2, 16.4.9, A.2.2
resolve an issue with the 'artwork'. Ref. 17.4.8
54 F 3.3.1c F. Correct. According to the ‘quality’ theme, project A, B, C, D, E. Incorrect. Ref 17.4.1-8
assurance should be consulted to check that the
57 B 3.4.1a B. Correct. Reviewing whether quality checks have been
assembled project initiation documentation meets the
carried out, in the quality register, is part of the 'review
needs of the customer. Ref 16.4.9, 8.2
the work package status' activity. Ref 17.4.2
A, B, C, D, E. Incorrect. Ref 16.4.2, 16.4.9, A.2.2
A, C, D, E, F. Incorrect. Ref 17.4.1-8
58 D 3.4.1a D. Correct. A product status account can be requested to
check the status of the products before reporting on
progress, as part of the 'report highlights' activity. Ref.
17.4.5
A, B, C, E, F. Incorrect. Ref 17.4.1-8

290 PRINCE2® Practitioner | Copyright© PeopleCert International Ltd. PRINCE2® Practitioner | Copyright© PeopleCert International Ltd. 291
Q A Syllabus Ref Rationale Q A Syllabus Ref Rationale
59 B 3.4.2 A. Incorrect. Work packages may be used during the initiation 62 D 3.5.2 A. Incorrect. The team manager can only proceed with the
stage, but do not need to be used. Ref 17.3 work package, or take corrective action while the work package
B. Correct. The 'controlling a stage' process is normally is forecast to be complete within the tolerances set by the
first used after the project board authorizes the project, project manager. Performing the songs live is outside the
but it may also be used during the initiation stage, if scope of the work package, so must be raised with the project
necessary. Ref 17.3 manager to decide on corrective action. Deciding to 'exploit'
C. Incorrect. The 'controlling a stage' process should be used the opportunity is not a decision the team manager should
for work within delivery stages. However, this is not the reason take. Ref 18.4.2, tab 10.3
why it is appropriate to use the 'controlling a stage' process in B. Incorrect. The team manager can only proceed with the
the initiation stage in this context. Ref 17.3 work package or take corrective action while the work package
D. Incorrect. The purchasing department is selecting the is forecast to complete within the tolerances set by the project
recording studio in this scenario. Team managers should be manager. Therefore, it is not appropriate that the team
selected according to their capability to manage the work on manager decides to take this corrective action. Ref 18.4.2
the product. The team manager will be selected once the C. Incorrect. The team manager can only proceed with the
recording studio is selected. This product is not the work package or take corrective action while the work package
responsibility of the sample song production team manager. is forecast to complete within the tolerances set by the project
Ref 7.2.1.8 manager. The recordings are off-specification and should be
reported to the project manager via the issue process. Ref
60 A 3.5.1a A. Correct. The fundamental principle is that before a
18.4.2
work package is allocated to a team, there should be
D. Correct. The team manager can only proceed with the
agreement between the project manager and the team
work package, or take corrective action while the work
manager as to what is to be delivered. Ref 18.4.1
package is forecast to be complete within the tolerances
B. Incorrect. As part of the 'execute a work package' activity,
set by the project manager. Performing the songs live is
the team manager should review and report the status of the
outside the scope of the work package. Team managers
work package to the project manager. Ref 18.4.2
Appendix 2

Appendix 2
must raise issues with the project manager before a
C. Incorrect. As part of the 'deliver a work package' activity, the
Resources

Resources
decision is made to take corrective action. Ref 18.4.2
team manager should review the approval records to verify
63 A 3.6.1a A. Correct. As the tolerance has been increased for the
that all the products to be delivered by the work package are
current stage, the next action to take would be corrective
approved. Ref 18.4.3
action. Therefore, the next use of the 'managing a stage
D. Incorrect. Checkpoint reports are used to report progress in
boundary' process will be when planning the next stage.
the ‘execute a work package’ activity, not as part of the ‘accept
Ref 17.4.8, 19.4.1
a work package’ activity. Ref 18.4.1, 18.4.2
B. Incorrect. As the tolerance has been increased for the
61 A 3.5.1a A. Correct. When accepting a work package, the team current stage, the next action to take would be corrective
manager should review the work package and consult action. Therefore, the next use of the 'managing a stage
with project assurance as to whether any extra reviewers boundary' process will be when planning the next stage. Ref
are required. Ref 18.4.1 17.4.8, 19.4.1
B. Incorrect. When accepting a work package, the team C. Incorrect. As part of the 'give ad hoc direction' activity, the
manager should review the work package and consult with project board can increase the tolerances that are forecast to
project assurance as to whether any extra reviewers are be breached within their delegated limits of authority without
required. If the concern cannot be resolved it may then be the need for an exception plan. Therefore, the next use of the
appropriate to raise a risk. Ref 18.4.1 'managing a stage boundary' process will be when planning
C. Incorrect. When accepting a work package, the team the next stage. Ref 15.4.4, 19.4.1
manager should review the work package and consult with D. Incorrect. Reviewing the performance of the whole project is
project assurance as to whether any extra reviewers are done as part of the 'closing a project' process, not the
required. If it is agreed that a specialist reviewer is required, it 'managing a stage boundary' process. Ref 20.4.4
may then be appropriate to ask the senior supplier for the
resource. Ref 18.4.1
D. Incorrect. When accepting a work package, the team
manager should review the work package and consult with
project assurance as to whether any extra reviewers are
required before any corrective action is taken. Ref 18.4.1

292 PRINCE2® Practitioner | Copyright© PeopleCert International Ltd. PRINCE2® Practitioner | Copyright© PeopleCert International Ltd. 293
Q A Syllabus Ref Rationale Q A Syllabus Ref Rationale
64 B 3.6.1b B. Correct. Exception plans must be produced to show the 67 D 3.7.1a A. Incorrect. As part of the ‘prepare planned closure’ activity,
actions required to recover from or avoid a deviation from the project plan is updated with actuals from the final
agreed tolerances in the project plan or a stage plan. If a management stage. Ref 20.4.1
stage plan is being replaced, this needs the approval of the B. Incorrect. As part of the ‘prepare planned closure’ activity,
project board. The exception plan has been produced for approval is requested to give notice to corporate, programme
the final stage, so it must be submitted to the project management or the customer that resources can be (or are
board. The stage-level exception does not threaten project about to be) released. Ref 20.4.1
tolerance, so does not need to be referred to the C. Incorrect. As part of the ‘prepare planned closure’ activity, it
programme level. Ref 9.2.1.3, tab 19.5 is confirmed that the project has delivered what is defined in
A, C, D. Incorrect. Ref 9.2.1.3, tab 19.5 the project product description, and that the acceptance
65 C 3.6.2 A. Incorrect. An exception report has already been produced. It criteria have been met. Ref 20.4.1
is a checkpoint report that shows the status of work packages, D. Correct. As part of the ‘hand over products’ activity, the
not an exception report. The checkpoint report is produced by benefits management approach is checked to ensure that
the team manager for the project manager during the it includes post-project activities to confirm the benefits
'managing product delivery' process. The project manager that cannot be measured until after the project’s products
does not produce checkpoint reports. Ref tab 18.2, A.4.2, have been in operational use for some time. Ref 20.4.3
19.4.4 68 C 3.7.2 A. Incorrect. There are no lessons to be learned from the
B. Incorrect. An exception report has already been produced. context given, so the information is not relevant when
The project board has requested an end stage report. Ref updating a lessons report or the end project report. Ref A.8.1,
19.4.4 A.15.1
C. Correct. If requested by the project board, an end stage B. Incorrect. The follow-on action recommendations cover the
report should be produced in response to the exception project's products and include any uncompleted work, issues
report and to accompany the exception plan. The end and risks. There are no follow-on actions identified in the
stage report will include the activities carried out to date. context given. Ref 20.4.1, 20.4.3
Appendix 2

Appendix 2
Ref 19.4.4 C. Correct. Project support creates the product status
Resources

Resources
D. Incorrect. Although it is late in stage 3, an exception plan account to check that the products, in this case the
has been requested, along with an end stage report, because ‘launch event plan’, have been approved by the authority
an exception has occurred, not because the project is identified in the product description. Ref 20.4.1
approaching the end of stage 3. Ref 19.4.4 D. Incorrect. Project support creates the product status
66 B 3.7.1a A. Incorrect. As part of the 'evaluate the project' activity, the account to check that the products, in this case the ‘launch
project manager works with the project management team to event plan’, have been approved by the authority identified in
summarize how the project has performed. Ref 20.4.4 the product description. Project support is not responsible for
B. Correct. As part of the ‘prepare planned closure’ the support of products, so have no support to transfer. Ref
activity, the project manager confirms that the project has 20.4.1, 20.4.3
delivered what is defined in the project product
description and that the acceptance criteria have been
met. Ref 20.4.1
C. Incorrect. As part of the ‘evaluate the project’ activity, the
project manager completes an assessment of the project's
results against the expected benefits in the business case
within the end project report. Ref 20.4.4
D. Incorrect. As part of the 'hand over products' activity, the
project manager prepares follow-on action recommendations
to include any incomplete work, issues and risks. Ref 20.4.3

294 PRINCE2® Practitioner | Copyright© PeopleCert International Ltd. PRINCE2® Practitioner | Copyright© PeopleCert International Ltd. 295
Sample Paper 2: PRINCIPLES
1. The record company's finance director has asked the executive to ensure that work
packages agreed with team managers are signed off by the project board before the team
starts the work.

Question Booklet Is this an appropriate application of the 'manage by stages' principle?

Multiple Choice A. Yes, because the executive should ensure that the business investment provides value
for money
Examination Duration: 2 hours and 30 minutes B. Yes, because the project should be divided into at least two management stages
C. No, because the finance director should sign off work packages within a stage
D. No, because the project manager should have authority to manage the project on a day-
Instructions to-day basis

2. At the end of stage 2, the risk that production costs could exceed the sales for the music
1. You should attempt all 68 questions. Each question is worth one mark. album is estimated to be high, as all of the cost tolerance for stage 2 has been used. As a
2. There is only one correct answer per question. result, the project manager has defined tight cost and time tolerances for the work packages
to produce the 'artwork' and record the album during stage 3.
3. You need to answer 38 questions correctly to pass the exam.
Which principle is being applied, and why?
4. Mark your answers on the answer sheet provided. Use a pencil (NOT pen).
A. 'Manage by exception', because the level of control retained by the project manager
5. You have 2 hours and 30 minutes to complete this exam. must match the tolerance available
6. This is an ‘open book’ exam. You can use the Managing Successful Projects with PRINCE2 B. 'Manage by exception', because action should be taken so that production costs do not
(2017) Official Book. exceed the album sales
Appendix 2

Appendix 2
C. 'Manage by stages', because cost tolerance should be set for each work package in a
7. No other material is allowed.
Resources

Resources
stage
8. Read the ‘Project Scenario’ in the Scenario Booklet. D. 'Manage by stages', because team managers should be given authority before work can
commence
9. You will also need the ‘Additional Information’ in the Scenario Booklet to answer some
questions.
10. Each question is separate. Do not use information from one question to answer another 3. The Graphic Designer appointed by the external agency is the team manager responsible for
question. creation of the 'artwork'. Before assigning a work package to the Graphic Designer, the
project manager ensured that the 'artwork' product description attached to the work
package had measurable and prioritized features defined in it.
Is this an appropriate application of the 'focus on products' principle, and why?
A. Yes, because it reduces disputes during handover, by agreeing what will be delivered
B. Yes, because the 'artwork' is a specialist product and the team members are responsible
for its delivery
C. No, because as an external party, the graphics designer should not access the product
description
D. No, because the 'artwork' product description is a management product, and not a
specialist product

296 PRINCE2® Practitioner | Copyright© PeopleCert International Ltd. PRINCE2® Practitioner | Copyright© PeopleCert International Ltd. 297
4. The Music Album Project team has identified that another department within the company is 7. The project manager has been recruited from a large, multi-national record company. To
producing a similar album of the same type of music. As it is only a small company, it cannot reduce the time spent on the initiation stage, the project manager decided to use PRINCE2
resource two similar album projects. management approaches from projects at the previous company.
Which principle should have been applied more effectively to avoid this situation, and why? Is this appropriate, and why?
A. 'Defined roles and responsibilities', because cross-functional projects involve people A. Yes, because the project manager is learning from experience on previous projects
from different departments B. Yes, because the project manager is improving business justification by reducing costs
B. 'Defined roles and responsibilities', because a project management team structure C. No, because the previous company's project management approaches are unlikely to
enables effective communication between team members meet the needs of the project
C. 'Continued business justification', because linking projects to the organizational D. No, because the project should focus on specialist products rather than project
objectives ensures benefits are aligned to strategy management approaches
D. 'Continued business justification', because the justification for projects should be
reviewed regularly throughout the project lifecycle
8. The record company is concerned that the songs to be included on the album should be
agreed before work on the 'recorded album' starts. Therefore, the project board has decided
5. It was found that one of the songs approved for the album had quality issues even though a that no changes to the list of songs will be accepted after the work package has been agreed.
quality inspection was carried out. It was therefore decided that a further objective check of
Which principle is being applied when making this decision, and why?
recording quality should be conducted by the studio for the songs that are yet to be
approved. A. 'Continued business justification', because it sets limits on delegated change authority
B. 'Continued business justification', because it enables funds to be invested in worthwhile
Is this an appropriate application of the 'learn from experience' principle, and why?
objectives
A. Yes, because lessons should be learnt and applied during a project C. 'Focus on products', because this will ensure that the album is delivered on budget
B. Yes, because an objective check should accompany a quality inspection D. 'Focus on products', because it should ensure the album will meet expectations
C. No, because lessons should be learnt at the beginning of a project
D. No, because lessons are passed on at the end of a project
BUSINESS CASE
Appendix 2

Appendix 2
Resources

Resources
6. The project's products are being delivered by both external suppliers and record company
Here are three statements from the business case for the Music Album Project.
staff. A member of the recording studio staff has been appointed as a senior supplier on the
project board. It has also been decided that the Vice President of the record company, who Under which heading of the business case (A-F) should the statements be recorded?
is the executive for the project, will also be a senior supplier, representing the record
Choose only ONE heading for each statement. Each heading can be used once, more than once,
company.
or not at all.

Is this an appropriate application of the 'defined roles and responsibilities' principle, and 9. It has been decided to invest in a new singer A. Reason
why?
B. Business options
10. We need to keep up with our competitors who
A. Yes, because the Vice President should be able to represent both the business and C. Expected benefits
have seen an increase in sales from the new
supplier interests
singers they are signing with D. Expected dis-benefits
B. Yes, because the Vice President is responsible for achieving the expected sales from the
music album E. Timescale
C. No, because there should be one senior supplier representing the supplier interests on 11. Current customers may not buy from us if they see
F. Major risks
the project board a change in the style of music we sell. This may
D. No, because the executive role cannot be shared and therefore cannot also act as the cause a decrease in overall sales
senior supplier

298 PRINCE2® Practitioner | Copyright© PeopleCert International Ltd. PRINCE2® Practitioner | Copyright© PeopleCert International Ltd. 299
12. During the 'initiating a project' process, the Vice President (VP) estimated that the Music 17. During the project, the CEO decides that the project should become a pilot project within a
Album Project should generate sales that exceed production costs. However, the sales programme to establish new ways to launch new singers quickly into the market.
estimate may be too high, and therefore the record company may not make a profit. The VP
Which action is appropriate, and why?
will undertake business assurance and the Marketing Manager, who is the senior user, will
undertake user assurance. A. Project and programme management roles should be integrated, to ensure there are
clear lines of authority from top to bottom
Is it appropriate for the VP to be responsible for assessing the effect of low sales on the B. Project and programme management roles should be integrated, to ensure there is
business case, and why? continued business justification for the project
C. Additional project roles should be defined, to ensure the programme roles and
A. Yes, because the VP should monitor whether the album sales are high enough responsibilities are mapped to the team roles
B. Yes, because business assurance reviews risks and their impact on the business case D. Additional project roles should be defined, to ensure the business role represents the
C. No, because the executive is ultimately accountable for the project's success wider customer view at the delivery level
D. No, because the Marketing Manager should take responsibility for any risk to sales

13. The Vice President (VP) was appointed as executive of the Music Album Project, which is now
18. During stage 1, when preparing the communication management approach, the project
closing. During the project, extra funding was required and approved by the CEO. However,
manager included the record company's Marketing Manager as a stakeholder. The
this funding was given on the condition that, when closing the project, the executive
Marketing Manager will be checking with focus groups that the music in the album has a
prepares an end project report showing the results of the project against its planned targets
market. Without a market for the album there will be no business justification for the project
in the business case.
to continue.
Is this an appropriate condition, and why?
A. Yes, because the VP secures project funding and is accountable for the business case for
How well does this action apply the ‘organization’ theme, and why?
the duration of the project
B. Yes, because the VP is responsible for assessing the results of the project against the
Appendix 2

Appendix 2
expected benefits in the business case
A. It applies it well, because stakeholders external to the customer organization can exert a
Resources

Resources
C. No, because the project manager is responsible for preparing the end project report to
powerful influence on the project's success
assess project performance and results
B. It applies it well, because the Marketing Manager will need to be aware of progress
D. No, because the project manager verifies that the project was justified against external
information about the stage and receive reports from the focus groups
events and project progress
C. It applies it poorly, because stakeholders are those groups or individuals who will be
affected by the project's outputs, such as delivery channels
ORGANIZATION D. It applies it poorly, because the focus groups should be included in the communication
management approach as stakeholders, not the Marketing Manager

Here are three roles relating to the Music Album Project.


Which individual (A-F) would be most appropriate for each role?
Choose only ONE individual for each role. Each individual can be used ONCE, more than once, or
not at all.

14. Executive A. CEO

B. Vice President
15. Senior supplier
C. Contracts Manager

D. Marketing Director
16. Delegated business assurance
E. Singer’s agent

F. Singer

300 PRINCE2® Practitioner | Copyright© PeopleCert International Ltd. PRINCE2® Practitioner | Copyright© PeopleCert International Ltd. 301
QUALITY PLANS
The Music Album Project is approaching the end of stage 3 and the project manager is planning Here are three items of content included in the stage plan for stage 3 of the Music Album Project.
stage 4.
Under which heading (A-F) should each item be recorded?
Here are three actions related to managing quality for stage 4.
Choose only ONE heading for each item of information. Each heading can be used once, more
Which role (A-F) should be responsible for each action? than once, or not at all.
Choose only ONE role for each action. Each role can be used once, more than once, or not at all. 24. Inexperienced singers usually need several A. Plan prerequisites
attempts at recording a song. Therefore, more
19. Review the quality criteria for the ‘registered A. Executive B. External dependencies
time has been allowed for this activity in the stage
artwork’ to recommend additional people to be
B. Senior user plan C. Lessons incorporated
involved in the quality review
C. Project manager D. Monitoring and control
20. Agree that the 'launch event plan' should be 25. The project manager will review the issue register
D. Team manager E. Budgets
presented to the project board in the format of a daily for priority 1 and 2 issues
slide deck E. Project assurance F. Tolerances
F. Project support 26. The recording studio's health and safety policies
21. Document that the 'launch event plan' should be and procedures
presented to the project board in the format of a
slide deck
27. During stage 3, it was decided to change the design of the 'artwork' to attract attention and
help promote sales. The project board agreed to pay for the change using the change
22. The product description for the 'launch event plan' was updated at the end of stage 3 to budget. The project plan was later updated to show the amount of change budget still
include more detailed quality criteria. The 'launch event plan' was created during stage 4. available to the project.
Appendix 2

Appendix 2
The 'launch event plan' was then approved by the project manager after checking that it met
Resources

Resources
Was it appropriate to update the project plan, and why?
the defined quality criteria.
A. Yes, because the project board will use the project plan and project justification when
Is this an appropriate way to manage quality, and why?
deciding whether to carry out future changes or not
A. Yes, because the project manager should ensure that the quality controls for the 'launch B. Yes, because the project board will have used the project plan and project justification
event plan' will meet the defined quality criteria when deciding whether to carry out this change or not
B. Yes, because the project manager should ensure that product descriptions are updated C. No, because the project plan provides a baseline against which the project board
when planning the next stage monitors project progress
C. No, because it is the senior user who should confirm the customer quality expectations D. No, because using the change budget for the change will not impact the project cost
and acceptance criteria for the project budget
D. No, because it is the senior user who should provide resources to review and approve
the 'launch event plan' according to the defined quality criteria
28. The project is in the initiation stage. In consultation with the senior user, the project
manager has made some changes to the description of the final product 'album ready for
23. The album will be recorded by an external supplier. The project manager is concerned that launch'. The project manager's next task is to plan the project.
the record company will have no access to check the quality of the recorded songs before
Which action demonstrates the project manager's immediate priority, and why?
the album is completed. Therefore, the project manager has requested that the contract
with the supplier includes a requirement to review the quality of the songs after each one is A. Create a hierarchy of products for the 'album ready for launch', because it is necessary
recorded. to understand the major products and their component products
B. Create a hierarchy of products for the 'album ready for launch', because the
Is this appropriate, and why?
interdependencies between the component products need to be understood
A. Yes, because the customer should specify their rights of inspection as well as the quality C. Identify the product interdependencies between the 'recorded album' and the 'artwork',
expectations for a product because this is necessary in order to decide on activities and resources
B. Yes, because the senior supplier should approve the quality methods to be used to D. Identify the product interdependencies between the 'recorded album' and the 'artwork',
check the quality of the recorded songs because this is necessary in order to explore their more detailed characteristics
C. No, because the team manager should ensure that the recorded songs meet the quality
criteria specified in the product description
D. No, because there should be a shared understanding between the customer and the
supplier of the quality levels required

302 PRINCE2® Practitioner | Copyright© PeopleCert International Ltd. PRINCE2® Practitioner | Copyright© PeopleCert International Ltd. 303
RISK CHANGE
An issue has been raised that the singer wants to add an extra song to the recorded album.
When negotiating a contract with the singer, information was received that the singer may sign a
contract with another record company. Making this change would take stage 3 beyond its cost tolerance.
Here are three actions relating to the management of this risk. Here are three actions that are carried out when managing the issue.
During which activity relating to the recommended risk management procedure (A-F) should they During which step of the issue and change control procedure (A-E) should they be carried out?
occur?
Choose only ONE step for each action. Each step can be used once, more than once, or not at all.
Choose only ONE activity for each action. Each activity can be used once, more than once, or not
at all. 34. The project manager asks the senior user to A. Capturing issues
confirm the priority of adding the song, before B. Assessing issues
29. The reason that the singer may sign with another A. Identify the risk considering what action to recommend
company is that this other company may offer more C. Proposing corrective actions
B. Estimate the risk
favourable terms D. Deciding on corrective
35. When the impact is reviewed, it is decided that the
C. Plan the response actions
issue should be managed formally, rather than
E. Implementing corrective
30. The contract terms are updated to make them D. Implement the response informally
actions
more favourable
E. Communicate about the
risk 36. The project board requests a plan to record the
31. The Contracts Manager signs a revised contract F. Fund the response from the extra song
with more favourable terms than the previous offer risk budget

37. The recording studio has carried out improvements and can now deliver recordings to a
32. The singer has signed a contract with the record company. The contract specified that the higher quality than specified in the product description for the 'recorded album'. The
Recording Studio Manager has recorded the songs to the new improved standard, at no
Appendix 2

Appendix 2
singer would perform at a large international festival on a Friday night. It is hoped that this
additional cost. This change has no other impact on the project.
Resources

Resources
may increase international sales of the album. However, the audience is usually bigger on a
Saturday. Therefore, the project manager managed to negotiate for the singer to perform on How should the project manager categorize this issue, and why?
Saturday, rather than Friday.
A. As a request for change, because the recording will be of better quality
Has the project manager exploited the opportunity to increase album sales, and why? B. As a request for change, because the product description will need to be updated
A. Yes, because the positive effect on album sales is not guaranteed C. As an off-specification, because the recording of the songs will not fulfil the criteria
B. Yes, because the project manager’s response will make the risk cause a certainty specified
C. No, because the singer now has the opportunity to appear on Saturday D. As an off-specification, because a proposal to change a baseline has been received
D. No, because performing on Saturday may have a greater impact on album sales
38. During stage 3, the project board grants a concession to permit the use of 'artwork' that
33. The Marketing Director will analyse the opinions of focus groups to check that the album will does not fully meet the quality criteria. As a result, the project manager has updated the
be marketable and generate sufficient sales. At the end of the initiation stage, the following product description for the ‘artwork’.
risk was recorded in the risk register: Is this action appropriate, and why?
"There is a threat that the project will be no longer viable which would result in the project A. Yes, because the product description should be updated to reflect the revised quality
needing to be stopped." criteria
How well does this identify the risk, and why? B. Yes, because a product description should be updated to show the status of a delivered
product
A. It identifies it well, because the impact that the risk could have on the project and its C. No, because product descriptions are baselined documents and should not be changed
viability is clearly understood D. No, because project support should update product descriptions following a concession
B. It identifies it well, because risks should be identified continuously throughout the
project and recorded in the risk register
C. It identifies it poorly, because the source of the risk has not been understood so the
relevant risk responses cannot be selected
D. It identifies it poorly, because the effect of the risk has not been understood so the
relevant risk responses cannot be selected

304 PRINCE2® Practitioner | Copyright© PeopleCert International Ltd. PRINCE2® Practitioner | Copyright© PeopleCert International Ltd. 305
PROGRESS STARTING UP A PROJECT
Here are three actions relating to controlling progress on the Music Album Project. Here are three actions that occur during the 'starting up a project' process for the Music Album
Project.
Which role (A-E) should carry them out?
In which activity (A-F) should the action occur?
Choose only ONE role for each action. Each role can be used once, more than once, or not at all.
Choose only ONE action for each activity. Each activity can be used once, more than once, or not
39. Set the time tolerance of stage 3 as +1 week A. Team manager
at all.
B. Senior user
40. Inform the project manager that the 'artwork' 44. Discuss, with the Marketing Manager, how focus A. Appoint the executive and
C. Project assurance
production is forecast to exceed its time tolerance groups have been used previously the project manager
D. Executive
B. Capture previous lessons
E. Project support
41. Assist the project manager in using project 45. Obtain any standard contracts relevant to the
C. Design and appoint the
planning software singer's contract
project management team
D. Prepare the outline business
46. Create the daily log to record that the project
case
42. The work package for the 'launch event plan' is in progress and checkpoint reports are being mandate has been checked and the promotional
produced every two weeks. The project manager has reviewed the issue register and is E. Select the project approach
video is outside the scope of the project
concerned about the number of issues related to this work package. and assemble the project
brief
Which action should the project manager take, and why? F. Plan the initiation stage
A. Increase the frequency of checkpoint reports to weekly, so that work package level
exceptions are prevented
B. Increase the frequency of checkpoint reports to weekly, to increase the level of control 47. The record company wants to keep the project confidential in order to avoid another record
over this work package company launching a similar album at the same time. During the 'starting up a project'
Appendix 2

Appendix 2
Resources

Resources
C. Increase tolerances for the work package, so that there is increased permissible process, the project manager discusses the project with colleagues who have experience of
deviation and fewer issues will be raised keeping a project confidential. Their experiences are used to develop the draft project
D. Increase tolerances for the work package, so that they are the same as those for the approach.
stage to make reporting consistent
Is this appropriate, and why?
A. Yes, because the risk of another company launching a similar album should be
43. The songs have been recorded and meet the minimum quality requirements. However, the documented in the project approach
team manager recommends that they are re-recorded as better quality songs may improve B. Yes, because the requirement to keep the project confidential is relevant when deciding
album sales. Re-recording will cause the stage to exceed its time tolerance. As a result, the the project approach
project manager has raised an issue to the project board. C. No, because lessons related to communications are relevant for developing the
Is this action appropriate, and why? communication management approach
D. No, because lessons captured from previous experiences on similar projects should be
A. Yes, because opportunities to improve business justification should be considered by the
recorded in the lessons log
project board
B. Yes, because the stage would be in exception and an exception plan would be required
C. No, because the project manager should reject the recommendation as the work
package has been completed within the quality tolerance
D. No, because the project manager should take corrective action to improve the quality of
the recorded songs

306 PRINCE2® Practitioner | Copyright© PeopleCert International Ltd. PRINCE2® Practitioner | Copyright© PeopleCert International Ltd. 307
DIRECTING A PROJECT INITIATING A PROJECT
Here are three actions that are carried out during the 'directing a project' process. Here are three actions relating to the 'initiating a project' process.
Which role (A-E) should carry out these actions? In which activity (A-F) should each action be recorded?
Choose only ONE activity for each action. Each activity can be used once, more than once, or not Choose only ONE activity for each action. Each activity can be used once, more than once, or not
at all. at all.

48. Approve extra management actions to ensure that A. Executive 52. Check with project assurance that the proposed A. Agree the tailoring
the 'launch event plan' results in sufficient people B. Senior user format of the benefits management approach requirements
listening to the music and contributes to achieving will meet the needs of the record company B. Prepare the risk management
C. Senior supplier
the expected album sales approach
D. Project assurance
53. The singer’s agent will require regular C. Prepare the change control
E. Project manager approach
49. Advise the project manager of a change in information on the feedback from the focus
corporate business case standards that will affect group on the sample songs D. Prepare the quality
how the business case is produced management approach
E. Prepare the communication
54. Identify when approval will be given for work to
management approach
50. Review the time tolerance set for the delivery of commence on the 'recorded album' and 'artwork'
F. Create the project plan
the 'album ready for launch' to check that it is
realistic

55. The Music Album Project will be part of a programme for the record company to sell a
51. The project is approaching the end of the initiation stage. The project manager has different type of music. The project is now in the 'initiating a project' process and the project
requested that the project board meet to 'authorize the project'. The project manager has manager has requested support from the programme office when producing the risk
management approach.
Appendix 2

Appendix 2
prepared a set of slides showing the project controls, the business case and the scope of the
Resources

Resources
project, to present to the project board. Which statement explains why this is an appropriate way to produce the risk management
How well is the 'authorize the project' activity being carried out? approach?

A. It is being carried out well, because the members of the project board should meet in A. Because the project manager should produce the risk management approach during the
person to approve the project initiation documentation 'initiating a project' process
B. It is being carried out well, because the project initiation documentation can be B. Because the risk management approach should define how the risk register will be
authorized by the project board in any format, provided the decision is traceable managed and maintained
C. It is being carried out poorly, because applying the principle of 'manage by exception' C. Because in a simple project the risk management approach should be combined with
removes the need for meetings the programme's risk management strategy
D. It is being carried out poorly, because the project board should authorize the next stage D. Because the programme office typically provides assistance to ensure that the project
plan as well as the project initiation documentation complies with programme standards

308 PRINCE2® Practitioner | Copyright© PeopleCert International Ltd. PRINCE2® Practitioner | Copyright© PeopleCert International Ltd. 309
CONTROLLING A STAGE MANAGING PRODUCT DELIVERY
60. The Graphic Designer contracted to create the 'artwork' in stage 3 will be following an agile
Here are three actions taken by the project manager as part of the 'controlling a stage' process.
approach. The project manager is acting as the team manager for the work package and has
Which theme (A-F) do they relate to? produced a team plan.
Choose only ONE theme for each action. Each theme can be used once, more than once, or not Why is the use of a team plan appropriate?
at all.
A. Because the team plan will show whether the 'artwork' can be completed within the agile
56. Record in the daily log that the graphic designer A. Business case timebox
will be unavailable the following Friday B. Because the project manager should produce a team plan when they act as a team
B. Quality
manager
C. Risk C. Because a team plan is required for an external work package
57. Check whether authorizing overtime has resulted
D. Because a team plan is required when following an agile approach
in the sample songs being recorded on time D. Plans

E. Change
61. The 'artwork' will be delivered by a graphic design company using an agile delivery approach.
58. After receiving the sample song recordings,
F. Organization The work package is being negotiated before work on the 'artwork' commences.
execute any required benefit management actions
Who should approve the work package for the 'artwork'?
A. Project assurance
59. At the beginning of stage 3, the singer suggested that a video could be taken during the
B. Project manager
recording of the songs and used at the launch of the album to promote sales. This change
C. Team manager
can be accommodated in stage 3 and has been approved. The recording studio has agreed
D. Executive
to record the video. As a result, the project manager has created a product description for
the product 'recorded video', and amended the work package for the studio.
62. The Music Album Project will be part of a programme for the record company to sell a
Appendix 2

Appendix 2
Are the project manager's actions appropriate, and why?
different type of music. As a result, the team manager for the 'launch event plan' work
Resources

Resources
A. Yes, because the recording studio work package should define the work for the package has been asked to provide weekly checkpoint reports directly to the programme
'recorded album' and 'recorded video' manager. This will allow the programme manager to monitor the launch of the music.
B. Yes, because the product description for the 'recorded video' is needed to justify the
Is it appropriate for the team manager to send the programme manager checkpoint reports,
request for change
and why?
C. No, because the project manager should create product descriptions when a stage plan
is prepared A. Yes, because checkpoint reports should be tailored to suit the needs of the programme
D. No, because the project manager should update the stage plan when taking corrective manager
action to implement the change B. Yes, because checkpoint reports should be produced at the frequency defined in the
work package
C. No, because the team manager should advise the project manager when the artwork is
complete
D. No, because the 'managing product delivery' process interfaces with the 'controlling a
stage' process

MANAGING A STAGE BOUNDARY


63. Stage 3 is in exception. The project board has requested an exception plan from the project
manager, who has triggered the 'managing a stage boundary' process as a result.
Which action is OPTIONAL?
A. Prepare an end stage report
B. Revise the business case
C. Update the benefits management approach
D. Revise the project plan

310 PRINCE2® Practitioner | Copyright© PeopleCert International Ltd. PRINCE2® Practitioner | Copyright© PeopleCert International Ltd. 311
64. A stage-level exception was identified while recording the sample songs. As a result, the 68. During the initiation stage, a risk was recorded that sales of the singer's album might impact
project board instructed the project manager to prematurely close the stage and create a sales of other albums produced by the record company. Therefore, when this risk was
plan to combine the remainder of stage 2 together with the production of the 'recorded reviewed during project closure, a follow-on action recommendation was made for the
album' and the 'artwork'. record company's audit department to report on the impact during post-project benefit
reviews.
Which activity should the project manager carry out in order to produce the combined plan?
Is this appropriate, and why?
A. Plan the next management stage
B. Prepare premature closure A. Yes, because the project cannot be closed until the impact from sales are measured
C. Produce an exception plan B. Yes, because the follow-on action will enable this risk to be closed in the risk register
D. Create the project plan C. No, because a follow-on action should be addressed to an individual rather than a
department
D. No, because the benefits management approach should include post-project activities
65. During stage 2, it is found that the graphics design company is in financial difficulty. As a
result, another supplier is selected to produce the 'artwork' and their account manager is
asked to join the project board as a senior supplier with immediate effect.
Is this appropriate, and why?
A. Yes, because PRINCE2 is flexible in order to avoid unnecessary delays in implementing
decisions
B. Yes, because changing suppliers is a big decision and should be treated as an exception
C. No, because there should be one senior supplier representing supplier interests on the
project board
D. No, because changes to roles should happen at a stage boundary unless there is an
exception
Appendix 2

Appendix 2
CLOSING A PROJECT
Resources

Resources
66. The Music Album Project has delivered the 'album ready for launch'. When closing the
project, the project manager considers that the Recording Studio Manager has been an
excellent team manager. As a result, the project manager prepares a recommendation that
the Recording Studio Manager should be contracted to record future albums.
In which activity of the 'closing a project' process should this recommendation be submitted
for approval?
A. Prepare planned closure
B. Hand over products
C. Evaluate the project
D. Recommend project closure

67. The project is now being closed.


Which action should the project manager take during the 'evaluate the project' activity?
A. Confirm acceptance of the 'artwork' by the Marketing Team of the record company
B. Identify how many errors were found after the 'artwork' had passed quality inspection
C. Confirm that the Marketing Team has the resources to promote the 'album ready for
launch'
D. Request a product status account for the 'album ready for launch' from project support

312 PRINCE2® Practitioner | Copyright© PeopleCert International Ltd. PRINCE2® Practitioner | Copyright© PeopleCert International Ltd. 313
Sample Paper 2: Answers and Rationales Q A Syllabus Ref Rationale
3 A 1.1.1f A. Correct. Applying the ‘focus on products’ principle
Q A Syllabus Ref Rationale helps a PRINCE2 project to be more product/output-
1 D 1.1.1d A. Incorrect. Although it is true that the executive needs to oriented rather than work-oriented. An output-oriented
ensure that business investment provides value for money, it project is one that agrees and defines the project’s
is not appropriate for work packages agreed with team products prior to undertaking the activities required to
managers to be signed off by the project board. It is not produce them, thus reducing the risk of user
appropriate because it conflicts with the ‘manage by stages’ dissatisfaction and acceptance disputes. Ref 3.6
principle as the project manager should have delegated B. Incorrect. It is true that the team is responsible for the
responsibility to manage the day-to-day delivery of the delivery of the product(s). However, this does not explain why
stages. Ref 3.4 the action described follows the 'focus on products' principle.
B. Incorrect. Although it is true that the project should be The product(s) should be defined in the work package to
divided into at least two management stages, it is not ensure that only the work that directly contributes to the
appropriate for work packages agreed with team managers delivery of the desired product is carried out. Ref 3.6
to be signed off by the project board. Ref 3.4 C. Incorrect. It is an appropriate application of the ‘focus on
C. Incorrect. It is not appropriate for work packages agreed products’ principle. Also, external team managers/members
with team managers to be signed off by the project board. should be able to access the description documents of the
This conflicts with the ‘manage by stages’ principle as the products they are creating or modifying. Ref 3.6
project manager should have delegated responsibility to D. Incorrect. It is true that the product description is a
manage the day-to-day delivery of the stages. Ref 3.4 management product, not a specialist product. However, this
D. Correct. The ‘manage by stages’ principle requires that does not explain why the action described follows the 'focus
the project board delegates the authority for day-to-day on products' principle. Ref 3.6
control of a management stage, within agreed 4 C 1.1.1a A. Incorrect. It is true that a project is typically cross-
tolerances, to the project manager. Requiring the project functional, may involve more than one organization, and may
board to sign off work packages before the project board involve a mix of full-time and part-time resources. However, it
starts work breaches this principle. Ref 3.4
Appendix 2

Appendix 2
is the 'continued business justification' principle that ensures
Resources

Resources
2 A 1.1.1e A. Correct. Applying the 'manage by exception' principle alignment with corporate strategies. Ref 3.1, 3.3
enables appropriate governance by defining distinct B. Incorrect. It is true that to be successful, projects must
responsibilities for directing, managing and delivering have an explicit project management team structure
the project and clearly defining accountability at each consisting of defined and agreed roles and responsibilities for
level. This is achieved by delegating authority from one the people involved in the project and a means for effective
management level to the next by setting tolerances for communication between them. However, it is the 'continued
the respective level of the plan. Ref 3.5 business justification' principle that ensures alignment with
B. Incorrect. It is true that action needs to be taken so that the corporate strategies. Ref 3.1, 3.3
production costs do not exceed the album sales. However, C. Correct. Organizations that lack rigour in business
this does not explain why the principle being applied is justification may find that projects proceed even where
'manage by exception'. Ref 3.5 there are few real benefits or where a project has only
C. Incorrect. While it is true that cost tolerance should be set tentative associations with corporate, programme or
for each work package in a stage, this is an application of the customer strategy. Poor alignment with corporate,
‘manage by exception’ principle, not the ‘manage by stages’ programme or customer strategies can also result in
principle. Ref 3.5, 3.4 organizations having a portfolio of projects that have
D. Incorrect. The ‘manage by stages’ principle ensures that mutually inconsistent or duplicated objectives. Ref 3.1
key decisions are made prior to the detailed work needed to D. Incorrect. The ‘continued business justification’ principle
implement them. However, this relates to decisions by the requires for all projects that the justification remains valid,
project board and not the authorization of work packages by and is re-validated, throughout the life of the project.
the project manager. Ref 3.5 However, this does not explain why this principle could have
assisted in avoiding a duplicate project being started. Ref 3.1

314 PRINCE2® Practitioner | Copyright© PeopleCert International Ltd. PRINCE2® Practitioner | Copyright© PeopleCert International Ltd. 315
Q A Syllabus Ref Rationale Q A Syllabus Ref Rationale
5 A 1.1.1b A. Correct. In this situation, the project team are learning 7 C 1.1.1g A. Incorrect. The project manager may be learning from
from an experience within the project. Lessons should be experience, but is not using that experience appropriately.
sought and learning applied throughout the project Without tailoring to suit the project, it is unlikely that the
lifecycle, as well as being sought and passed on at the project management effort and approach would be
beginning and closure of a project. Ref 3.2 appropriate for the needs of the project. Ref 3.7
B. Incorrect. It is not always necessary to carry out checks B. Incorrect. Cost may be reduced but the proposed course of
additional to a quality inspection. However, in this situation, action is contrary to the 'tailor to suit the project' principle. It
the project team have learnt, during the project, that this is is unlikely that the project management effort and approach
necessary. Ref 3.2, 8.3.13 would be appropriate for the needs of the project. Ref 3.7
C. Incorrect. Lessons are looked for, recorded and acted upon C. Correct. Unless PRINCE2 is tailored to suit the project
throughout the life of the project, not just at the beginning of environment, size, complexity, importance, team
a project. Ref 3.2 capability and risk, it is unlikely that the project
D. Incorrect. Lessons are looked for, recorded and acted upon management effort and approach would be appropriate
throughout the life of the project, not just at the closure of a for the needs of the project. Using PRINCE2 approaches
project. Ref 3.2 from a large multi-national record company are unlikely
6 A 1.1.1c A. Correct. One person can carry out more than one role, to be appropriate. Ref 3.7
as long as that person has the capacity to undertake the D. Incorrect. The project should focus on specialist products,
combined responsibilities and there is no conflict of but also ensure that the project management approaches are
interest. In this situation it may be necessary to have two appropriate. Without tailoring to suit the project, it is unlikely
or more senior suppliers to represent different supplier that the project management effort and approach would be
stakeholders. The Vice President can carry out the role of appropriate for the needs of the project. Ref 3.7
executive and represent the record company, which is 8 D 1.1.1f A. Incorrect. The 'focus on products', not the 'continued
supplying some of the products itself. Ref 7.2.1.10, 7.2.1.4 business justification' principle, is being applied, as the
B. Incorrect. It is an appropriate application of 'defined roles project board is making sure there is a common
Appendix 2

Appendix 2
and responsibilities' principle that one person can carry out understanding of the product required. Moreover,
Resources

Resources
more than one role, as long as that person has the capacity to establishing limits of delegated authority is necessary to
undertake the combined responsibilities and there is no apply the 'manage by exception' principle. Ref 3.5, 3.6
conflict of interest. However, the senior user, not the senior B. Incorrect. The 'focus on products', not the 'continued
supplier, should represent the user interest and ensure that business justification' principle, is being applied, as the
the expected benefits (derived from the project’s outcomes) project board is making sure there is a common
are realized. Ref 3.3, tab 6.1 understanding of the product required. This does not ensure
C. Incorrect. It is an appropriate application of ‘defined roles that funds are invested in worthwhile objectives. Ref 3.6, 3.1
and responsibilities' principle. There can be more than one C. Incorrect. It is true that applying the ‘focus on products’
senior supplier on the project board. Ref 3.3 principle helps in delivering products to budget, however it
D. Incorrect. It is an appropriate application of 'defined roles does not ensure this. Applying other principles, primarily
and responsibilities' principle. The executive role cannot be 'manage by exception’ ensures that budget limits are not
shared between two or more individuals, however the exceeded. Ref 3.5, 3.6
executive can carry out another role in addition. The Vice D. Correct. The 'focus on products' principle is being
President of the record company supervises the production applied, as the project board is making sure there is a
of the album and can act as senior supplier to represent the common understanding of the product required.
record company. Ref 3.3 Agreeing what is to be produced reduces the risk of user
dissatisfaction and acceptance disputes. Ref 3.6
9 B 2.1.1a B. Correct. This identifies why the recommended
business option was chosen. Ref A.2.2
A, C, D, E, F. Incorrect. Ref A.2.2
10 A 2.1.1a A. Correct. This is one of the reasons why the project is
required. Ref A.2.2
B, C, D, E, F. Incorrect. Ref A.2.2
11 F 2.1.1a F. Correct. The focus of this project is to increase sales.
This is a threat to the company’s existing sales, and is
therefore a major risk. Ref A.2.2
A, B, C, D, E. Incorrect. Ref A.2.2

316 PRINCE2® Practitioner | Copyright© PeopleCert International Ltd. PRINCE2® Practitioner | Copyright© PeopleCert International Ltd. 317
Q A Syllabus Ref Rationale Q A Syllabus Ref Rationale
12 B 2.1.2 A. Incorrect. Sales are generated and monitored after the 16 D 2.2.1c D. Correct. According to the scenario, the Marketing
project. However, this is not why the Vice President (VP), who Director verifies whether the sales of the album will
is undertaking business assurance, should assess the impact exceed the production costs and should therefore fulfil a
of the risk of low sales on the business case during the business project assurance role. Ref C.7.1
project. Ref C.7, A.2.1 A, B, C, E, F. Incorrect. Ref 7.3.4, C.2, C.4, C.7.1
B. Correct. The role of business assurance, which the Vice 17 A 2.2.2 A. Correct. The programme and project management
President (VP) is undertaking, should assess the impact of team structures and roles need to be integrated so that
the risk of low sales on the business case. Ref C.7, A.2.1 there are clear lines of responsibility from top to bottom.
C. Incorrect. The executive is ultimately accountable for the Ref 7.3.3
project’s success. However, this does not invalidate the VP B. Incorrect. Although roles need to be integrated, this is not
carrying out business assurance on the executive’s behalf. Ref because of the need for continued business justification. Ref
C.7, A.2.1 7.3.3
D. Incorrect. The Marketing Manager could undertake the C. Incorrect. This is an activity for a project within an agile
role of risk owner/actionee for this risk. However, this is not a environment, not a programme. Ref 7.3.5
reason why the VP, undertaking business assurance, should D. Incorrect. This is an activity for a project within an agile
not assess the impact of the risk of low sales on the business environment, not a programme. Ref 7.3.5
case. Ref C.7, A.2.1 18 B 2.2.2 A. Incorrect. It is true that stakeholders external to the
13 C 2.1.2 A. Incorrect. The executive is responsible for both the customer organization can exert influence on the project.
business case and the benefits review plan for the duration of However, this does not explain why defining the Marketing
the project. During the ‘closing a project’ process, it is the Manager as an internal stakeholder with communication
project manager who is responsible for reporting the results needs is an appropriate application of the ‘organization’
of the project against the expected benefits in the business theme. Ref 7.3.9
case. Ref tab 6.1, C.2 B. Correct. Stakeholders are individuals or groups that
B. Incorrect. The end project report is used to report project have an interest in the project. They may be internal or
Appendix 2

Appendix 2
performance during project closure. Preparing this report is external to the customer organization and may affect or
Resources

Resources
the responsibility of the project manager. Ref tab 6.1, 20.4.4, be affected by the project. The communication
C.2 management approach is used to document how
C. Correct. The project manager is responsible for stakeholder engagement will be carried out. The
preparing the end project report. One section in this Marketing Manager is an internal stakeholder with a
report is the assessment of the results of the project clear need for bi-directional communication with the
against the expected benefits in the business case. Ref focus groups. The information shared could affect the
tab 6.1, 20.4.4, C.5.1, A.8.2 business justification for the project. Ref 7.3.9, glossary
D. Incorrect. The project manager is responsible for preparing C. Incorrect. This is an appropriate application of the
the end project report which includes an assessment of the ‘organization’ theme. Also, a stakeholder is any individual or
results of the project against the expected benefits in the group that has an interest in a project, who may affect or be
business. However, project assurance verifies and monitors affected by it. Ref 7.1, 7.3.9
the business case against external events and project D. Incorrect. The focus groups are stakeholders who should
progress. Ref tab 6.1, 20.4.4, C.5.1, C.7.1 be included in the communication management approach.
14 B 2.2.1c B. Correct. The CEO, part of corporate management, However, this does not mean that the Marketing Manager is
delegates management of new singers to the Vice not also a stakeholder whose communication needs should
President. The Vice President supervises the production be included. Ref 7.3.9, glossary
of albums and has an interest in their success. Therefore, 19 E 2.3.1b E. Correct. Project assurance should assist the project
the Vice President is the most appropriate person to be board and project manager by reviewing the product
the executive and take responsibility for the business descriptions and advising the project manager on
case. Ref C.2 suitable quality reviewers. Ref tab 8.1
A, C, D, E, F. Incorrect. Ref 7.3.4, C.2, C.4, C.7.1 A, B, C, D, F. Incorrect. Ref tab 8.1, A.17.2
15 C 2.2.1c C. Correct. There are a number of specialist suppliers to 20 B 2.3.1b B. Correct. The ‘launch event plan’ is a key user product
this project. The Contracts Manager is responsible for and the format and presentation of this product would
each of their contracts and is therefore the most be included in the product description. The senior user
appropriate person to be senior supplier. Ref 7.3.4, C.4 approves product descriptions for key user products. Ref
A, B, D, E, F. Incorrect. Ref 7.3.4, C.2, C.4, C.7.1 tab 8.1, A.17.2
A, C, D, E, F. Incorrect. Ref tab 8.1, A.17.2

318 PRINCE2® Practitioner | Copyright© PeopleCert International Ltd. PRINCE2® Practitioner | Copyright© PeopleCert International Ltd. 319
Q A Syllabus Ref Rationale Q A Syllabus Ref Rationale
21 C 2.3.1b C. Correct. The project manager prepares and maintains 25 D 2.4.1a D. Correct. Monitoring and control details of how the plan
product descriptions. Ref tab 8.1 will be monitored and controlled. The project manager
A, B, D, E, F. Incorrect. Ref tab 8.1, A.17.2 will determine how and when they want to monitor
22 D 2.3.2 A. Incorrect. The project manager should ensure that team aspects of the project. Ref A.16.2
managers implement the quality control measures agreed in A, B, C, E, F. Incorrect. Ref A.16.2
product descriptions and work packages. However, the senior 26 B 2.4.1a B. Correct. The work in stage 3 will be dependent on
user should provide resources to undertake user quality compliance with the recording studio's health and safety
activities and product approval, rather than the project policies and procedures. These already exist outside of
manager giving approval. Ref tab 8.1 the project, so therefore are an external dependency. Ref
B. Incorrect. It is true that as part of the ‘plan the next A.16.2
management stage’ activity, product descriptions should be A, C, D, E, F. Incorrect. Ref A.16.2
updated for the products to be delivered in the next stage. 27 A 2.4.2 A. Correct. It is appropriate to update the amount of
However, this does not explain why it is the senior user, not change budget remaining, as this information will be of
the project manager, who should approve the delivered value when making decisions regarding requests for
product. Ref 19.4.1, tab 8.1 change in the future. Ref 15.4.4, 9.2.1.1, A.16.2
C. Incorrect. It is true that the senior user should confirm the B. Incorrect. The project plan provides the business case with
customer quality expectations and acceptance criteria for the planned costs and is used by the project board as a baseline
project. However, this does not explain why it is the senior to monitor project progress against. The project board will
user, not the project manager, who should approve the consider business justification and impact to the project
delivered product. Ref tab 8.1 when they make decisions relating to requests for change.
D. Correct. The senior user should provide resources to However, this does not explain why the action described is
undertake user quality activities and product approval. appropriate, as the decision to approve the request for
Therefore, the project manager should not approve the change has already taken place. Ref 15.4.4, 9.2.1.1
'launch event plan'. Ref tab 8.1 C. Incorrect. The project plan is used by the project board as a
Appendix 2

Appendix 2
23 A 2.3.2 A. Correct. It is important to be clear about who is baseline for measuring actual progress. However, this does
Resources

Resources
responsible for which aspect of quality. This is not explain why updating the amount of change budget
particularly important in commercial customer/supplier remaining is an appropriate action in this situation. Ref
situations, where the contract needs to make clear what 9.2.1.1
the quality expectations are. It is also advisable to define D. Incorrect. The use of the change budget will not require a
the customer’s rights of inspection and audit in terms of change to the project budget. However, this does not explain
what can be inspected or audited, how often and how why it is appropriate to update the amount of change budget
much notice needs to be given for any inspection or remaining. Ref A.16.2, glossary
audit. Ref 8.3.7
B. Incorrect. The senior supplier approves the quality
methods, techniques and tools adopted in product
development. However, this does not explain why it is also
advisable to define the customer’s rights of inspection. Ref
8.3.7, tab 8.1
C. Incorrect. The team manager is responsible for producing
products consistent with product descriptions. However, this
does not explain why it is also advisable to define the
customer’s rights of inspection. Ref 8.3.7, tab 8.1
D. Incorrect. There must be a shared understanding between
the customer and the supplier of the quality expected.
However, this does not explain why the customer should
specify their right to inspect the quality of a product or
products. Ref 8.3.6, 8.3.7
24 C 2.4.1a C. Correct. Lessons incorporated are details of relevant
lessons from previous similar projects, which have been
reviewed. Therefore, this is an example of a lesson being
incorporated. Ref A.16.2
A, B, D, E, F. Incorrect. Ref A.16.2

320 PRINCE2® Practitioner | Copyright© PeopleCert International Ltd. PRINCE2® Practitioner | Copyright© PeopleCert International Ltd. 321
Q A Syllabus Ref Rationale Q A Syllabus Ref Rationale
28 A 2.4.2 A. Correct. After finalizing the description of the project 32 B 2.5.2 A. Incorrect. Arranging for the singer to perform at the festival
product, a hierarchy of products is produced, known as a on Saturday would exploit the opportunity offered, ensuring
product breakdown structure. The project product is that the larger audience is reached. While it is true that the
broken down into its major products, which are then increase in sales is not guaranteed, this does not explain why
further broken down until an appropriate level of detail this response exploits the opportunity. Ref tab 10.3
for the plan is reached. Only after identifying a product it B. Correct. The exploit response is about making the
is possible to explore and understand its detailed nature uncertain situation certain by removing the risk. This can
and other features such as purpose and function. Ref often be achieved by implementing the cause of an
9.3.1.2, fig 9.6, 9.2 opportunity. Therefore, funding the singer to perform at
B. Incorrect. The hierarchy of products, known as the product the festival on Saturday, instead of the Friday, would
breakdown structure, is helpful in identifying the products at exploit the opportunity offered, ensuring that the larger
the level of detail necessary for a plan. It is a product flow audience is reached. Ref tab 10.3
diagram that identifies and defines the dependencies C. Incorrect. It is true that the singer now has the opportunity
between the products. Ref 9.3.1.2 to appear on the Saturday. However, as the project manager
C. Incorrect. A product flow diagram is used to identify and has negotiated for the singer to perform at the festival on
define the sequence in which the products will be developed Saturday this exploits the opportunity offered, ensuring that
and any dependencies between them. This is created in the the larger audience is reached. Ref tab 10.3
fourth step of 'defining and analyzing the products'. It is true D. Incorrect. It is true that performing on the Saturday may
that the products and their interdependencies should be have a greater impact on album sales. However, funding the
identified before deciding what activities and resources are singer to perform at the festival on Saturday exploits the
required to deliver those products. However, identifying the opportunity offered, ensuring that the larger audience is
products is the immediate priority. Ref 9.3.1.2, fig 9.6, 9.1.3 reached. Ref tab 10.3
D. Incorrect. A product flow diagram is used to identify and 33 C 2.5.2 A. Incorrect. The risk effect should describe the impact(s) that
define the sequence in which the products will be developed the risk would have on the project objectives should the risk
and any dependencies between them. This is created after
Appendix 2

Appendix 2
materialize. The risk event and effect have been defined, but
Resources

Resources
producing a hierarchy of the products within the project, not the risk cause. Ref 10.4.1.2
known as a product breakdown structure, and then writing B. Incorrect. Risks can, and should, be identified at any time
more detailed product descriptions. Identifying the products during the management and delivery of the project. However,
is the immediate priority. Ref 9.3.1.2, fig 9.6 the risk event and effect have been defined, but not the risk
29 A 2.5.1c A. Correct. This identifies the risk cause that the singer cause. Ref 10.4.1.2
may be offered more favourable terms by another record C. Correct. The risk cause should describe the source of
company. Ref 10.4.1.2 the risk. An important aspect of identifying risks is being
B, C, D, E, F - Incorrect. Ref 10.4.1.2, 10.4.4 able to provide a clear and unambiguous expression of
30 D 2.5.1c D. Correct. This is an 'implement' the response activity as each one. Without understanding the risk cause, it is not
it is an action to reduce the chances of the singer signing possible to manage the risk effectively. Ref 10.4.1.2
with the other company. Ref 10.4.4 D. Incorrect. The risk effect should describe the impacts that
A, B, C, E, F - Incorrect. Ref 10.4.1.2, 10.4.4 the risk would have on the project objectives, should the risk
31 D 2.5.1c D. Correct. This is an 'implement' the response activity as materialize. The risk effect is clearly described. Therefore, this
it is an action to reduce the chances of the singer signing does not explain why the risk is poorly identified. Ref 10.4.1.2
with the other company. Ref 10.4.4 34 B 2.6.1c B. Correct. As part of the 'assessing issues' step, it may be
A, B, C, E, F - Incorrect. Ref 10.4.1.2, 10.4.4 necessary to request advice from the project board to
check their understanding of the issue’s priority or
severity before proposing resolutions. Ref 11.4.2
A, C, D, E. Incorrect. Ref 11.4.1-5
35 B 2.6.1c B. Correct. During the ‘assessing issues’ step, when the
impact is reviewed, the decision can be taken that the
issue needs to be managed formally rather than
informally, as may have been decided during ‘capturing
issues’. Ref 11.4.1, 11.4.2
A, C, D, E. Incorrect. Ref 11.4.1-5

322 PRINCE2® Practitioner | Copyright© PeopleCert International Ltd. PRINCE2® Practitioner | Copyright© PeopleCert International Ltd. 323
Q A Syllabus Ref Rationale Q A Syllabus Ref Rationale
36 D 2.6.1c D. Correct. As part of the 'deciding on corrective actions' 40 A 2.7.1.b A. Correct. Team managers notify the project manager of
step, the project manager checks if the issue could be any forecast deviation from work package tolerances. Ref
handled without the need to escalate it to the project tab 12.2
board. If not, the issue will be escalated to the project B, C, D, E. Incorrect. Ref tab 12.2
board for a decision and they may ask for an exception 41 E 2.7.1.b E. Correct. Project support contributes expertise in
plan. Ref 11.4.4 specialist project management tools (for example,
A, B, C, E. Incorrect. Ref 11.4.1-5 planning and control tools). Ref tab 12.2
37 C 2.6.2 A. Incorrect. A request for change is a request for permission A, B, C, D. Incorrect. Ref tab 12.2
to change a product baseline. In this case, the product has 42 B 2.7.2 A. Incorrect. Increasing the frequency of checkpoint reports
been produced to a higher quality than the original would provide the project manager with more frequent
specification. Therefore, this issue is an off-specification. Ref information about work package progress, however this
tab 11.1, glossary would not prevent exceptions. Ref 12.2.1
B. Incorrect. If a request for change is approved, the product B. Correct. The project manager controls the progress
description will be updated. However, this is not a request for within the stage by reviewing reports on the progress of
permission to change a product baseline. In this case, the work packages. Increasing the frequency of checkpoint
product has been produced to a higher quality than the reports increases the project manager’s control of the
original specification. Therefore, this issue is an off- team's work by giving the project manager more
specification. Ref tab 11.1, glossary frequent information. Ref 12.2.2.4, 12.2.2.2
C. Correct. An off-specification is something that should C. Incorrect. Increasing tolerances for the work package
be provided by the project, but currently is not (or is would increase the permissible deviation and fewer issues
forecast not to be) provided. This might be a missing would be raised. However, this is not what is required to
product or a product not meeting its specification. In this address the project manager’s concerns. Ref 12.2.1
case, although the recording quality of the songs will be D. Incorrect. Increasing tolerances for the work package so
improved, it will still be something different from what that they are the same as the stage-level tolerances is not an
Appendix 2

Appendix 2
was planned. Ref tab 11.1 appropriate course of action. Tolerances should be set
Resources

Resources
D. Incorrect. The issue is an off-specification as the product according to the level of control needed and the capability to
has been produced to a higher quality than the original manage by exception at each level of management. Ref 12.2.1
specification. However, this is not a proposal for a change to a
baseline. Ref tab 11.1
38 A 2.6.2 A. Correct. When a product is granted a concession, the
product description will need to be revised before the
product is handed over to the user. Ref tab 11.3
B. Incorrect. When a product is granted a concession, the
product description will need to be revised before the
product is handed over to the user. However, product
descriptions are not updated to show the status of a product.
It is the configuration item record or product status account
which records the current status of a product. Ref A.6.1,
A.18.1
C. Incorrect. When a product is granted a concession, the
product description will need to be revised before the
product is handed over to the user. Ref tab 11.3
D. Incorrect. When a product is granted a concession, the
product description will need to be revised before the
product is handed over to the user. Although project support
usually administers the issue and change control procedure,
it is not mandatory for project support to update the product
description. Ref tab 11.2, tab 11.3
39 D 2.7.1.b D. Correct. The executive makes decisions on stage
tolerances and exception plans should there be a
forecast in exceeding stage-level tolerances set. Ref tab
12.2
A, B, C, E. Incorrect. Ref tab 12.2

324 PRINCE2® Practitioner | Copyright© PeopleCert International Ltd. PRINCE2® Practitioner | Copyright© PeopleCert International Ltd. 325
Q A Syllabus Ref Rationale Q A Syllabus Ref Rationale
43 A 2.7.2 A. Correct. There are two reasons to implement a change: 47 B 3.1.2 A. Incorrect. Major risks should be identified and included in
to introduce a new benefit or to protect an existing the outline business case, but not in the ‘project approach’
benefit. The action described is appropriate, as the part of the project brief. A.2.2, A.19.2
project board should be given the opportunity to B. Correct. When selecting the project approach any
consider a change that may impact positively on the security constraints that apply to the project should be
project's justification. The situation should be reported considered. Ref 14.4.5
by raising an issue. The songs meet the minimum quality C. Incorrect. Lessons related to communications should be
requirements, therefore there is no exception at this used to develop the communications management approach
point. Ref 11.1, tab 11.2, tab 11.3 during the 'initiating a project' process. However, this does
B. Incorrect. If the change is approved, the stage would be in not explain why it is appropriate to use these lessons when
exception, as stage tolerances would be exceeded. An developing the project approach. Ref 16.4.5, 14.4.5
exception plan is only produced when the project board D. Incorrect. When selecting the project approach any
requests it, following the consideration of an exception security constraints that apply to the project should be
report. The project manager has raised an issue, not an considered, as well as recording lessons in the lessons log.
exception report, because at this point the change has not Ref 14.4.5, 14.4.2
been approved and there is no exception. Ref 12.2.3 48 A 3.2.1.b A. Correct. The executive approves the updated benefits
C. Incorrect. The project manager could reject the issue as the management approach as part of the ‘authorize a stage
songs have been recorded within the quality tolerance or exception plan’ and ‘authorize project closure’
required. However, the recommendation from the team activities. Ref tab 15.3, tab 15.5
manager to re-record songs could result in an improved B, C, D, E. Incorrect. Ref tab 15.3-5, tab 6.1
business justification and should be raised as an issue for 49 A 3.2.1.b A. Correct. As part of the ‘give ad hoc direction’ activity, in
project board consideration. The senior user will want to response to advice and decisions from corporate,
safeguard the expected benefits and the executive will want programme management or the customer, the project
to focus on the continued business justification. Ref tab 11.2, manager needs to be notified by raising an issue. All
tab 11.3, 12.2.3
Appendix 2

Appendix 2
members of the project board can do this. However, it is
Resources

Resources
D. Incorrect. The project manager does not have the authority the executive who oversees the development of a viable
to take this corrective action as requests for change must be business case, ensuring that the project is aligned with
escalated to the change authority and improving the quality corporate, programme management or customer
of the songs would lead to an exception. Ref 11.1, tab 11.2, strategies. Ref tab 15.4, tab 6.1
tab 11.3, 12.2.3 B, C, D, E. Incorrect. Ref tab 15.3-5, tab 6.1
44 B 3.1.1.a B. Correct. The 'capture previous lessons' activity 50 D 3.2.1.b D. Correct. Project assurance reviews the tolerances for
involves consulting with individuals or teams with the project provided by corporate, programme
previous experience of similar projects. Ref 14.4.2 management or the customer to ensure that they are
A, C, D, E, F. Incorrect. Ref 14.4.1, 14.4.2 appropriate and realistic as part of ‘authorize the project’
45 D 3.1.1b D. Correct. The 'prepare the outline business case' activities. Ref tab 15.1, 15.4.1
activity involves assembling any relevant background A, B, C, E. Incorrect. Ref tab 15.3-5, tab 6.1
information (e.g. contracts, feasibility reports, service-
level agreements). Ref 14.4.4
A, B, C, E, F. Incorrect. Ref 14.4.1, 14.4.2
46 A 3.1.1b A. Correct. Activities when appointing the executive and
the project manager include reviewing the project
mandate to check understanding of the scope of the
project and creating the daily log. Ref 14.4.1
B, C, D, E, F. Incorrect. Ref 14.4.1, 14.4.2

326 PRINCE2® Practitioner | Copyright© PeopleCert International Ltd. PRINCE2® Practitioner | Copyright© PeopleCert International Ltd. 327
Q A Syllabus Ref Rationale Q A Syllabus Ref Rationale
51 B 3.2.2 A. Incorrect. The degree of formality to authorize the project 55 D 3.3.2 A. Incorrect. It is true that the project manager produces the
may differ in different circumstances provided decisions are risk management approach as part of the ‘initiating a project’
explicit and traceable. Members of the project board do not process. However, it does not explain why the programme
have to meet in person. Ref 3.5, 15.5.1 office should provide support when producing the risk
B. Correct. The degree of formality to authorize the management approach to ensure consistency with
project may differ in different circumstances provided programme standards. Ref 16.5.3, 16.4.2
decisions are explicit and traceable, ranging from verbal B. Incorrect. It is true that the risk management approach
advice and decisions to those which are formally should define how the risk register will be managed and
documented in email, meeting minutes or other maintained. However, it does not explain why the
traceable formats. Management products can be in other programme office should provide support when producing
formats and do not necessarily need to be “text the risk management approach to ensure consistency with
documents”. These could be slides or spreadsheets. Ref programme standards. Ref 16.5.3, A.24.2
15.5.1, App A C. Incorrect. Initiating a simple project is likely to be less
C. Incorrect. Applying the 'manage by exception' principle formal than for a larger project, with management products
provides for very efficient use of senior management time as combined into a small number of documents. However, this
it reduces senior managers’ time burden without removing does not explain why the programme office should provide
their control by ensuring decisions are made at the right level support in producing the risk management approach. Ref
in the organization. However, this decision is being made at 16.5.4.1
the right level, and the project board can choose to meet in D. Correct. The project manager may assign whoever is
person. The degree of formality to authorize the project may appropriate to the task of producing the management
differ in different circumstances provided decisions are products. Often support may be provided by a higher
explicit and traceable. Ref 3.5, 15.5.1 level programme office or similar. The risk management
D. Incorrect. The 'authorize the project' activity will be approach should comply with any corporate, programme
triggered by a request from the project manager for management or customer strategies, standards or
authorization to deliver the project, and may be undertaken practices relating to risk management need to be applied
Appendix 2

Appendix 2
Resources

Resources
in parallel with authorizing a stage or exception plan. Ref by the project. Ref 16.5.3, 16.4.2
15.4.2 56 E 3.4.1.c E. Correct. Issues that can be handled informally are
52 A 3.3.1.a A. Correct. When agreeing the tailoring requirements, the recorded in the daily log. Issue and change control is part
project manager should consult with project assurance to of the ‘change’ theme. Ref 17.4.6, 11.4.1
check that any proposed tailoring will meet the needs of A, B, C, D, F. Incorrect. Ref 17.4.1-8
corporate, programme management, or the customer. 57 E 3.4.1.c E. Correct. As part of the 'review the management stage
Ref 16.4.1 status' activity, the project manager should check the
B, C, D, E, F. Incorrect. Ref 16.2, 16.4.1-7 status of any corrective actions. This is related to
53 E 3.3.1.a E. Correct. The communication management approach managing issues as part of the ‘change’ theme. Ref 17.4.4,
addresses both internal and external communications. It 17.4.6, 11.4.5
should contain details of how the project management A, B, C, D, F. Incorrect. Ref 17.4.1-8
team will send information to, and receive information 58 A 3.4.1.c A. Correct. As part of the 'review the management stage
from, the wider organization(s) involved with, or affected status' activity, the required benefit actions are carried
by, the project. Ref 16.4.5 out to ensure and confirm that benefits are realized. This
A, B, C, D, F. Incorrect. Ref 16.2, 16.4.1-7 is part of the ‘business case’ theme. Ref 6.2.3, 17.4.4
54 F 3.3.1.a F. Correct. As part of the 'create the project plan' activity, B, C, D, E, F. Incorrect. Ref 17.4.1-8
the activities, resources and timings for the project
controls should be identified and included in the plan.
This would include when a stage boundary will take
place. Ref 16.4.7
A, B, C, D, E. Incorrect. Ref 16.2, 16.4.1-7

328 PRINCE2® Practitioner | Copyright© PeopleCert International Ltd. PRINCE2® Practitioner | Copyright© PeopleCert International Ltd. 329
Q A Syllabus Ref Rationale Q A Syllabus Ref Rationale
59 A 3.4.2 A. Correct. Work packages are used to define and control 62 D 3.5.2 A. Incorrect. Management products should be tailored to the
the work to be done by teams and will contain the requirements and environment of each project. However,
product descriptions for the work. The project manager there is no interface between managing product delivery and
will create the product description for the new product programme management. Ref 18.5.4.4, App A
filmed recording whilst updating the stage plan in the B. Incorrect. Checkpoint reports should be provided in the
activity 'take corrective action'. Ref 17.2, 17.4.8, glossary manner and at the frequency defined in the work package.
B. Incorrect. The request for change will already have been However, this does not explain whether it is appropriate to
assessed by the project manager during the activity 'capture send checkpoint reports to programme management. Ref
and examine issues and risks'. This will have been in 18.4.2
accordance with the change control approach. A minimum C. Incorrect. The team manager should notify the project
requirement is to assess whether issues might have a manager that the work package is complete as part of the
material impact on the business justification of the project. 'deliver a work package' activity. However, this does not
This does not explain the project manager's action of creating explain why it is inappropriate for the team manager to
a product description. Ref 11.2, 17.4.6 provide checkpoint reports to the programme manager. Ref
C. Incorrect. The project manager creates or updates product 18.4.2, 18.4.3
descriptions, if used, during the activity 'plan the next D. Correct. The project should be managed at the four
management stage' in the 'managing a stage boundary' management levels. The 'managing product delivery'
process. However, in this case it is appropriate for the project process has no direct interface with programme
manager to create the product description for the new management. As part of the 'managing product delivery'
product ‘recorded video’ when updating the stage plan in the process, the progress of work packages is reported to the
activity 'take corrective action', and include this in the project manager, who then reports highlights as part of
amended work package. Ref 19.4.1, 17.4.8 the 'controlling a stage' process. Ref 18.5.4.4, fig 17.6
D. Incorrect. The project manager will create the product 63 A 3.6.1a A. Correct. For an exception plan, depending on the point
description for the new product ‘recorded video’ when within the management stage when the exception
updating the stage plan in the activity 'take corrective action'.
Appendix 2

Appendix 2
occurred, it may be appropriate to produce an end stage
Resources

Resources
However, the action to update the work package is report for the activities to date. Whether this is required
appropriate. Ref 17.4.8 will be advised by the project board in response to the
60 A 3.5.1a A. Correct. Although a team plan is optional, it is exception report. Ref 19.4.4
appropriate when following an agile approach to produce B. Incorrect. The business case is revised as part of ‘update
a team plan to show that a product can be completed the business case’ activity. Ref 19.4.3
within a timebox. Ref 18.4.1, 9.2.1.4, 9.3.3 C. Incorrect. The benefits management approach is updated
B. Incorrect. Team plans are optional, irrespective of whether as part of ‘update the business case’ activity. Ref 19.4.3
the team manager role is being carried out by the project D. Incorrect. The project plan is revised as part of ‘update the
manager. However, it is appropriate when following an agile project plan’ activity. Ref 19.4.2
approach to produce a team plan to show that a product can
be completed within a timebox. Ref 9.2.1.4, 18.4.1
C. Incorrect. Team plans are optional, irrespective of whether
the team is external or internal to the customer organization.
However, it is appropriate when following an agile approach
to produce a team plan to show that a product can be
completed within a timebox. Ref 9.2.1.4, 18.4.1
D. Incorrect. It is not mandatory when following an agile
approach to produce a team plan. Ref 9.2.1.4, 18.4.1, 9.3.3
61 C 3.5.1b A. Incorrect. Project assurance reviews the work package, but
does not approve it. Ref tab 18.1
B. Incorrect. The project manager produces the work package
as part of the ‘controlling a stage’ process. The team manager
approves it. Ref tab 18.1
C. Correct. The team manager approves the work
package which is created by the project manager. Ref tab
18.1
D. Incorrect. The executive is not involved in the accepting of
a work package. Ref tab 18.1

330 PRINCE2® Practitioner | Copyright© PeopleCert International Ltd. PRINCE2® Practitioner | Copyright© PeopleCert International Ltd. 331
Q A Syllabus Ref Rationale Q A Syllabus Ref Rationale
64 C 3.6.1.a A. Incorrect. The 'plan the next management stage' activity is 66 C 3.7.1.a A. Incorrect. During the ‘prepare planned closure’ activity, the
used to plan the next stage of a project when the previous project manager ensures that all the project products have
stage approaches its planned end. In this case, there has been achieved and delivered. Ref 20.4.1, 20.4.4
been an exception and the project manager has been asked B. Incorrect. During the ‘hand over products’ activity, the
to produce a plan to replace the remainder of the current projects products are passed to the operational and
stage and incorporate some additional products. The plan will maintenance environment. This is where follow-on action
therefore be an exception plan, not a stage plan. Ref 19.4.5 recommendations are created. Ref 20.4.3, 20.4.4
B. Incorrect. ‘Prepare premature closure’ is an activity that is C. Correct. The report is recording a lesson that should be
part of the ‘closing a project’ process. The current stage will considered by a future project, not a follow on action
be re-planned, but the project is not being closed. Ref 20.4.2, about a product from this project. The report is therefore
19.4.5 a lessons report which should be submitted for approval
C. Correct. The stage is in exception. The project board by corporate, programme management or the customer
has asked the project manager to produce a plan to as part of the 'evaluate the project' activity. Ref 20.4.4
replace the remainder of the current stage and D. Incorrect. During the ‘recommend project closure’ activity,
incorporate some additional products originally expected the project manager confirms to the project board that the
to be created during stage 3. The project manager is project can be closed. Ref 20.4.5, 20.4.4
therefore producing an exception plan, not the next 67 B 3.7.1.a A. Incorrect. Confirming acceptance from the operations and
stage plan. Ref 19.4.5, 19.4.1 maintenance organizations is an action during the ‘hand over
D. Incorrect. The stage is in exception and the project products’ activity. Ref 20.4.3
manager has been instructed to re-plan the remainder of the B. Correct. The project manager should, as part of the
stage together with additional products. Therefore, the 'evaluate a project' activity, review useful measurements
appropriate activity to use is ‘produce an exception plan’. It is such as how effective the quality management approach
unclear whether the project is also now in exception. was in designing, developing and delivering fit-for-
However, ‘create a project plan’ is an activity of the ‘initiating a purpose products (e.g. how many errors were found after
project’ process and is not used to re-plan a project that is in
Appendix 2

Appendix 2
products had passed quality inspections). Ref 20.4.4
Resources

Resources
exception. A project exception plan is, like a stage exception C. Incorrect. Confirming that the correct operational and
plan, created using the ‘produce an exception plan' activity. maintenance environment is in place is an action during the
Ref 19.4.5, 16.4.7 ‘hand over products’ activity. Ref 20.4.3
65 D 3.6.2 A. Incorrect. It is true that PRINCE2 can be tailored to meet D. Incorrect. Requesting a product status account from
the specific needs of the organization and scaled to the size project support is an action during the ‘prepare planned
and complexity of different projects. However, changing the closure’ activity. Ref 20.4.1
project management team members at any time without any
urgent requirement can interrupt the project flow. Ref 19.4.2,
1.1
B. Incorrect. Changing the graphics design company in stage
2 is not expected to result in an exception as production of
the 'artwork' is scheduled to happen in stage 3. The decision
should therefore be implemented as part of the 'managing a
stage boundary' process. Ref 19.4.2
C. Incorrect. The action is not appropriate as project
management team changes should happen at a stage
boundary, unless there is an exception or other urgent
situation. Also, there may be more than one individual on the
project board as a senior supplier. Ref 19.4.2, 7.2.1.4, 7.3.4
D. Correct. As part of ‘plan the next stage’ activity in the
'managing a stage boundary' process, the components of
the project initiation documentation are reviewed. This is
the right time for reviewing and, if necessary, updating
any change in the project management team or their role
descriptions (in particular the situation with regard to
external resources or suppliers). In the situation
described there is no urgency or exception that would
require an immediate change. Ref 19.4.2

332 PRINCE2® Practitioner | Copyright© PeopleCert International Ltd. PRINCE2® Practitioner | Copyright© PeopleCert International Ltd. 333
Q A Syllabus Ref Rationale
68 B 3.7.2 A. Incorrect. During the 'hand over products' activity of the
'closing a project' process, follow-on action recommendations
for the project’s products to include any uncompleted risks
should be created. Although some benefits cannot be
measured until after the project's products have been in
operational use for some time. However, the project can be
closed provided the benefits management approach includes
post-project activities to confirm these benefits. Ref 20.4.3
B. Correct. During the 'hand over products' activity of the
'closing a project' process, follow-on action
recommendations for the project’s products to include
any uncompleted risks should be created to enable the
risk to be closed in the register. Ref 20.4.3, 20.4.5
C. Incorrect. During the 'hand over products' activity of the
'closing a project' process, follow-on action recommendations
for the project’s products to include any uncompleted risks
should be created. Follow-on actions can be addressed to
individuals or distinct user groups. Ref 20.4.3
D. Incorrect. During the 'hand over products' activity of the
'closing a project' process, follow-on action recommendations
for the project’s products to include any uncompleted risks
should be created. The benefits management approach
should include post-project activities. These are in addition to
creating follow-on actions and may be derived from them. Ref
20.4.3
Appendix 2

Appendix 2
Resources

Resources
334 PRINCE2® Practitioner | Copyright© PeopleCert International Ltd. PRINCE2® Practitioner | Copyright© PeopleCert International Ltd. 335
Glossary approver
The person or group (for example a project
baseline
Reference levels against which an entity is
change authority
A person or group to which the project
board) who is identified as certified and monitored and controlled. board may delegate responsibility for the
authorized to approve a (management or consideration of requests for change or off-
accept (risk response) specialist) product as being complete and fit for baseline management product specifications. The change authority may
A risk response that means that the purpose. A type of management product that defines be given a change budget and can approve
organization takes the chance that the risk will asset aspects of the project and, when approved, is changes within that budget.
occur, with full impact on objectives if it does. An item, thing or entity that has potential subject to change control.
or actual value to an organization [ISO change budget
acceptance 55000:2014]. benefit The money allocated to the change authority
The formal act of acknowledging that the The measurable improvement resulting from available to be spent on authorized requests
project has met agreed acceptance criteria assumption an outcome perceived as an advantage by one for change.
and thereby met the requirements of its A statement that is taken as being true for the or more stakeholders.
stakeholders. purposes of planning, but which could change change control
later. An assumption is made where some facts benefits management approach The procedure that ensures that all changes
acceptance criteria are not yet known or decided, and is usually An approach that defines the benefits that may affect the project’s agreed objectives
A prioritized list of criteria that the project reserved for matters of such significance that, management actions and benefits reviews that are identified, assessed and then approved,
product must meet before the customer will if they change or turn out not to be true, there will be put in place to ensure that the project’s rejected or deferred.
accept it (that is, measurable definitions of the will need to be considerable replanning. outcomes are achieved and to confirm that the
attributes required for the set of products to be project’s benefits are realized. change control approach
acceptable to key stakeholders). assurance A description of how and by whom the project’s
All the systematic actions necessary to provide benefits tolerance products will be controlled and protected.
accountable confidence that the target (for example system, The permissible deviation in the expected
Personally answerable for an activity. process, organization, programme, project, benefit that is allowed before the deviation checkpoint
Appendix 2

Appendix 2
Accountability cannot be delegated, unlike outcome, benefit, capability, product output or needs to be escalated to the next level A team-level, time-driven review of progress.
Resources

Resources
responsibility. deliverable) is appropriate. Appropriateness of management. Benefits tolerance is
might be defined subjectively or objectively in documented in the business case. See also checkpoint report
activity different circumstances. The implication is that tolerance. A progress report of the information gathered
A process, function or task that occurs over assurance will have a level of independence at a checkpoint, which is given by a team to the
time, has recognizable results and is managed. from that which is being assured. See also burn chart project manager and which provides reporting
It is usually defined as part of a process or plan. project assurance; quality assurance. A technique for showing progress (such as with data as defined in the work package.
a timebox), where work that is completed and
agile and agile methods authority work still to do are shown with one or more closure recommendation
A broad term for a collection of behaviours, The right to allocate resources and make lines that are updated regularly or daily. A recommendation prepared by the project
frameworks, concepts and techniques that decisions (applies to project, management manager for the project board to send as a
go together to enable teams and individuals stage and team levels). business case project closure notification when the board is
to work in an agile way that is typified by The justification for an organizational activity satisfied that the project can be closed.
collaboration, prioritization, iterative and authorization (project), which typically contains timescales,
incremental delivery, and timeboxing. There The point at which an authority is granted. costs, benefits and risks, and against which communication management approach
are several specific methods (or frameworks) continuing viability is tested. A description of the means and frequency of
that are classed as agile, such as Scrum and avoid (risk response) communication between the project and its
Kanban. PRINCE2 is completely compatible with A risk response to a threat where the threat centre of excellence stakeholders.
working in an agile way. either can no longer have an impact or can no A corporate coordinating function for
longer happen. portfolios, programmes and projects providing concession
approval standards, consistency of methods and An off-specification that is accepted by the
The formal confirmation that a product is backlog processes, knowledge management, assurance project board without corrective action.
complete and meets its requirements (less A list of new features for a product. The list and training.
any concessions) as defined by its product may be made up of user stories which are configuration item
description. structured in a way that describes who wants An entity that is subject to change control. The
the feature and why. entity may be a component of a product, a
product or a set of products in a release.

336 PRINCE2® Practitioner | Copyright© PeopleCert International Ltd. PRINCE2® Practitioner | Copyright© PeopleCert International Ltd. 337
configuration item record customer’s quality expectations end project report exception assessment
A record that describes the status, version and A statement about the quality expected from A report given by the project manager to the A review by the project board to approve or
variant of a configuration item, and any details the project product, captured in the project project board, confirming the handover of all reject an exception plan.
of important relationships between them. product description. products. It provides an updated business case
and an assessment of how well the project has exception plan
configuration management daily log done against the original PID. A plan that often follows an exception report.
Technical and administrative activities A log used to record problems/concerns For a stage plan exception, it covers the period
concerned with the controlled change of a that can be handled by the project manager end stage assessment from the present to the end of the current
product. informally. The review by the project board and project management stage. If the exception were
manager of the end stage report to decide at project level, the project plan would be
configuration management system deliverable whether to approve the next stage plan. replaced.
The set of processes, tools and databases See output. Depending on the size and criticality of
that are used to manage configuration data. the project, the review may be formal or exception report
Typically, a project will use the configuration delivery approach informal. The authority to proceed should be A description of the exception situation,
management system of either the customer or The specialist approach used to create the documented as a formal record. its impact, options, recommendation and
supplier organization. products. impact of the recommendation. This report
end stage report is prepared by the project manager for the
constraints delivery step A report given by the project manager to the project board.
The restrictions or limitations by which the A step within the delivery approach. project board at the end of each management
project is bound. stage of the project. This provides information executive
dependency (plan) about the project’s performance during the The individual with overall responsibility for
contingent plan A dependency means that one activity is management stage and the project status at ensuring that a project meets its objectives and
A plan intended for use only if required (for dependent on another. There are at least two the management stage end. delivers the projected benefits. This individual
example if a risk response is not successful). types of dependency relevant to a project: should ensure that the project maintains its
Appendix 2

Appendix 2
Often called a fallback plan. internal and external. enhance (risk response) business focus, that it has clear authority,
Resources

Resources
An internal dependency is one between two A risk response to an opportunity where and that the work, including risks, is actively
corporate, programme management, or project activities. In these circumstances the proactive actions are taken to enhance both managed. The executive is the chair of the
customer standards project team has control over the dependency. the probability of the event occurring and the project board. They represent the customer
These are overarching standards to which impact of the event should it occur. and is responsible for the business case.
An external dependency is one between a
the project must adhere. They will influence
project activity and a non-project activity, where
the four project approaches (communication epic exploit (risk response)
non-project activities are undertaken by people
management, change control, quality A high-level definition of a requirement that has A risk response to an opportunity. It means
who are not part of the project team. In these
management and risk management) and the not been sufficiently refined or understood yet. seizing the opportunity to ensure that it will
circumstances the project team does not have
project controls. Eventually, an epic will be refined and broken happen and that the impact will be realized.
complete control over the dependency.
down into several user stories or requirements.
corrective action follow-on action recommendations
dis-benefit
A set of actions to resolve a threat to a plan’s event-driven control Recommended actions related to unfinished
A measurable decline resulting from an
tolerances or a defect in a product. A control that takes place when a specific event work, ongoing issues and risks, and any other
outcome perceived as negative by one or more
occurs. This could be, for example, the end of activities needed to take a product to the next
stakeholders, which reduces one or more
cost tolerance a management stage, the completion of the phase of its life. These are summarized and
organizational objective(s).
The permissible deviation in a plan’s cost that PID, or the creation of an exception report. It included in the end stage report (for phased
is allowed before it needs to be escalated to could also include organizational events that handover) and end project report.
embedding (PRINCE2)
the next level of management. Cost tolerance may affect the project, such as the end of the
The act of making something an integral part of
is documented in the respective plan. See also financial year. governance (corporate)
a bigger whole.
tolerance. The ongoing activity of maintaining a sound
Embedding is what an organization needs to exception system of internal control by which the
do to take on PRINCE2 as its corporate project
customer A situation where it can be forecast that there directors and officers of an organization ensure
management method and encourage its
The person or group who commissioned the will be a deviation beyond the tolerance levels that effective management systems, including
widespread use.
work and will benefit from the end results. agreed between the project manager and the financial monitoring and control systems, have
project board (or between the project board been put in place to protect assets, earning
and corporate, programme management or the capacity and the reputation of the organization.
customer).

338 PRINCE2® Practitioner | Copyright© PeopleCert International Ltd. PRINCE2® Practitioner | Copyright© PeopleCert International Ltd. 339
governance (project) issue management stage performance targets
Those areas of corporate governance that A relevant event that has happened, was not The section of a project that the project A plan’s goals for time, cost, quality, scope,
are specifically related to project activities. planned, and requires management action. It manager is managing on behalf of the project benefits and risk.
Effective governance of project management can be any concern, query, request for change, board at any one time, at the end of which the
ensures that an organization’s project portfolio suggestion or off-specification raised during a project board will wish to review progress to plan
is aligned with the organization’s objectives, is project. Project issues can be about anything to date, the state of the project plan, the business A detailed proposal for doing or achieving
delivered efficiently and is sustainable. do with the project. case and risks and the next stage plan, in order something which specifies the what, when,
to decide whether to continue with the project. how and by whom it will be achieved. In
handover issue register PRINCE2 there are only the following types of
The transfer of ownership of a set of products A register used to capture and maintain maturity plan: project plan, stage plan, team plan and
to the respective user(s). The set of products is information on all of the issues that are being A measure of the reliability, efficiency exception plan.
known as a release. There may be more than managed formally. The issue register should be and effectiveness of a process, function,
one handover in the life of a project (phased monitored by the project manager on a regular organization, and so on. The most mature planned closure
delivery). The final handover takes place in the basis. processes and functions are formally aligned The PRINCE2 activity to close a project.
closing a project process. with business objectives and strategy, and
issue report are supported by a framework for continual planning horizon
highlight report A report containing the description, impact improvement. The period of time for which it is possible to
A time-driven report from the project manager assessment and recommendations for a plan accurately.
to the project board on management stage request for change, off-specification or a maturity model
progress. problem/concern. It is created only for those A method of assessing organizational capability portfolio
issues that need to be handled formally. in a given area of skill. The totality of an organization’s investment (or
host site segment thereof) in the changes required to
A location where project work is being key performance indicator (KPI) milestone achieve its strategic objectives.
undertaken (for example an office or A measure of performance that is used to A significant event in a plan’s schedule,
Appendix 2

Appendix 2
construction site). help an organization define and evaluate how such as completion of key work packages, a premature closure
Resources

Resources
successful it is in making progress towards its development step or a management stage. The PRINCE2 activity to close a project before
impact (of risk) organizational objectives. its planned closure. The project manager must
The result of a particular threat or opportunity off-specification ensure that work in progress is not simply
actually occurring, or the anticipation of such a lessons log Something that should be provided by the abandoned, but that the project salvages any
result. An informal repository for lessons that apply to project, but currently is not (or is forecast value created to date, and checks that any gaps
this project or future projects. not to be). It might be a missing product or a left by the cancellation of the project are raised
information radiator product not meeting its specifications. It is one to corporate, programme management or the
A general term used to describe the use of log type of issue. customer.
walls or boards containing information that An informal repository managed by the project prerequisites (plan)
can be readily accessed by people working manager that does not require any agreement operational and maintenance acceptance Any fundamental aspects that must be in place,
on the project. It can contain any information, by the project board on its format and A specific type of acceptance by the person and remain in place, for a plan to succeed.
although it would typically show such things as composition. PRINCE2 has two logs: the daily or group who will support the product after
work to do and how work is progressing. log and the lessons log. it has been handed over into the operational PRINCE2 principles
environment. The guiding obligations for good project
inherent risk management product management practice that form the basis of a
The exposure arising from a specific risk before A product that will be required as part of outcome project being managed using PRINCE2.
any action has been taken to manage it. managing the project, and establishing and The result of change, normally affecting
maintaining quality (for example highlight real-world behaviour and/or circumstances. PRINCE2 project
initiation stage report, end stage report). The management Outcomes are desired when a change is A project that applies the PRINCE2 principles.
The period from when the project board products are constant, whatever the type of conceived. They are achieved as a result of the
authorizes initiation to when it authorizes the project, and can be used as described, or with activities undertaken to effect the change. probability
project (or decides not to go ahead with it). any relevant modifications, for all projects. This is the evaluated likelihood of a particular
The detailed planning and establishment of the There are three types of management product: output threat or opportunity actually happening,
project management infrastructure is covered baselines, records and reports. A specialist product that is handed over to including a consideration of the frequency with
by the initiating a project process. a user (or users). Note that management which this may arise.
products are not outputs but are created solely
for the purpose of managing the project.

340 PRINCE2® Practitioner | Copyright© PeopleCert International Ltd. PRINCE2® Practitioner | Copyright© PeopleCert International Ltd. 341
problem product status account project closure notification project mandate
A type of issue (other than a request for change A report on the status of products. The Advice from the project board to inform all An external product generated by the authority
or off-specification) that the project manager required products can be specified by identifier stakeholders and the host sites that the project commissioning the project that forms the
needs to resolve or escalate. Also known as a or the part of the project in which they were resources can be disbanded and support trigger for starting up a project.
concern. developed. services, such as space, equipment and access,
demobilized. It should indicate a closure date project office
procedure product-based planning for costs to be charged to the project. A temporary office set up to support the
A series of actions for a particular aspect of An approach for developing a comprehensive delivery of a specific change initiative being
project management established specifically plan based on the creation and delivery of project initiation documentation (PID) delivered as a project. If used, the project office
for the project (for example a risk management required outputs. The approach considers A logical set of documents that brings together undertakes the responsibility of the project
procedure). prerequisite products, quality requirements the key information needed to start the support role.
and the dependencies between products. project on a sound basis and that conveys the
process information to all concerned with the project. project plan
A structured set of activities designed to programme A high-level plan showing the major products
accomplish a specific objective. A process takes A temporary structure designed to lead project initiation notification of the project, when they will be delivered and
one or more defined inputs and turns them multiple interrelated projects and other work Advice from the project board to inform all at what cost. An initial project plan is presented
into defined outputs. in order to progressively achieve outcomes of stakeholders and the host sites that the project as part of the PID. This is revised as information
benefit for one or more organizations. is being initiated and to request any necessary on actual progress appears. It is a major control
producer logistical support (for example communication document for the project board to measure
The person or group responsible for project facilities, equipment and any project support) actual progress against expectations.
developing a product. A temporary organization that is created for sufficient for the initiation stage.
the purpose of delivering one or more business project product
product products according to an agreed business case. project lifecycle What the project must deliver in order to gain
An input or output, whether tangible or The period from initiation of a project to the acceptance.
Appendix 2

Appendix 2
intangible, that can be described in advance, project approach acceptance of the project product.
Resources

Resources
created and tested. PRINCE2 has two types of A description of the way in which the work of project product description
products: management products and specialist the project is to be approached. For example, project management A special type of product description used
products. are we building a product from scratch or The planning, delegating, monitoring and to gain agreement from the user on the
buying in a product that already exists? control of all aspects of the project, and project’s scope and requirements, to define
product breakdown structure the motivation of those involved, to achieve the customer’s quality expectations and the
A hierarchy of all the products to be produced project assurance the project objectives within the expected acceptance criteria for the project.
during a plan. The project board’s responsibilities to assure performance targets for time, cost, quality,
itself that the project is being conducted scope, benefits and risk. project support
product checklist correctly. The project board members each An administrative role in the project
A list of the major products of a plan, plus key have a specific area of focus for project project management team management team. Project support can be
dates in their delivery. assurance, namely business assurance for The entire management structure of the in the form of advice and help with project
the executive, user assurance for the senior project board, and the project manager, plus management tools, guidance, administrative
product description user(s), and supplier assurance for the senior any team manager, project assurance and services such as filing, and the collection of
A description of a product’s purpose, supplier(s). project support roles. actual data.
composition, derivation and quality criteria. It is
produced at planning time, as soon as possible project brief project management team structure proximity (of risk)
after the need for the product is identified. A statement that describes the purpose, cost, An organization chart showing the people The time factor of risk (that is, when the risk
time and performance requirements, and assigned to the project management team may occur). The impact of a risk may vary in
product flow diagram constraints for a project. It is created before the roles to be used, and their delegation and severity depending on when the risk occurs.
A diagram showing the sequence of production project begins, during the starting up a project reporting relationships.
and interdependencies of the products listed in process, and is used during the initiating quality
a product breakdown structure. a project process to create the PID and its project manager The degree to which a set of inherent
components. It is superseded by the PID and The person given the authority and characteristics of a product, service, process,
not maintained. responsibility to manage the project on a day- person, organization, system or resource fulfils
to-day basis to deliver the required products requirements.
within the constraints agreed with the project
board.

342 PRINCE2® Practitioner | Copyright© PeopleCert International Ltd. PRINCE2® Practitioner | Copyright© PeopleCert International Ltd. 343
quality assurance quality register reports risk estimation
A planned and systematic process that A register containing summary details of all Management products providing a snapshot of The estimation of probability and impact
provides confidence that outputs will match planned and completed quality activities. The the status of certain aspects of the project. of an individual risk, taking into account
their defined quality criteria when tested under quality register is used by the project manager predetermined standards, target risk levels,
quality control. It is carried out independently and project assurance as part of reviewing request for change interdependencies and other relevant factors.
of the project team. progress. A proposal for a change to a baseline. It is a
type of issue. risk evaluation
quality control quality review The process of understanding the net effect of
The process of monitoring specific project See quality inspection. residual risk the identified threats and opportunities on an
results to determine whether they comply The risk remaining after the risk response has activity when aggregated together.
with relevant standards and of identifying quality review technique been applied.
ways to eliminate causes of unsatisfactory A technique with defined roles and a specific risk exposure
performance. structure, designed to assess whether a responsible The extent of risk borne by the organization at
product in the form of a document (or similar, Used to describe the individual who has the the time.
quality criteria such as a presentation) is complete, adheres to authority and is expected to deliver a task or
A description of the quality specification standards and meets the quality criteria agreed activity; responsibility can be delegated. risk management
that the product must meet, and the quality for it in the relevant product description. The The systematic application of principles,
measurements that will be applied by those participants are drawn from those with the responsible authority approaches and processes to the tasks of
inspecting the finished product. necessary competence to evaluate its fitness The person or group commissioning the project identifying and assessing risks, planning
for purpose. (typically corporate, programme management and implementing risk responses and
quality inspection or the customer) who has the authority to communicating risk management activities with
A systematic, structured assessment of a quality tolerance commit resources and funds on behalf of the stakeholders.
product carried out by two or more carefully The tolerance identified for a product for a commissioning organization.
selected people (the review team) in a planned, quality criterion defining an acceptable range risk management approach
Appendix 2

Appendix 2
documented and organized fashion. of values. Quality tolerance is documented in reviewer An approach describing the goals of applying
Resources

Resources
the project product description (for the project- A person or group independent of the risk management, as well as the procedure
quality management level quality tolerance) and in the product producer who assesses whether a product that will be taken on, roles and responsibilities,
The coordinated activities to direct and control description for each product to be delivered. meets its requirements as defined in its risk tolerances, the timing of risk management
an organization with regard to quality. product description. interventions, the tools and techniques that will
records be used, and the reporting requirements.
quality management approach Dynamic management products that maintain risk
An approach defining the quality techniques information regarding project progress. An uncertain event or set of events that, should risk owner
and standards to be applied, and the various it occur, will have an effect on the achievement A named individual who is responsible for
responsibilities for achieving the required reduce (risk response) of objectives. A risk is measured by a the management, monitoring and control
quality levels, during a project. A response to a risk where proactive actions combination of the probability of a perceived of all aspects of a particular risk assigned to
are taken to reduce the probability of the event threat or opportunity occurring, and the them, including the implementation of the
quality management system occurring by performing some form of control, magnitude of its impact on objectives. selected responses to address the threats or to
The complete set of quality standards, and/or to reduce the impact of the event maximize the opportunities.
procedures and responsibilities for an should it occur. risk actionee
organization or specific entity (site, business A nominated owner of an action to address a risk profile
unit, and so on.) within that organization. registers risk. Some actions may not be within the remit A description of the types of risk that are faced
Formal repositories managed by the project of the risk owner to control explicitly; in that by an organization and its exposure to those
quality records manager that require agreement by the project situation there should be a nominated owner risks.
Evidence kept to demonstrate that the required board on their format, composition and use. of the action to address the risk. They will
quality assurance and quality control activities PRINCE2 has three registers: issue register, risk need to keep the risk owner apprised of the risk register
have been carried out. register and quality register. situation. A record of identified risks relating to an
initiative, including their status and history.
release risk appetite
The set of products in a handover. The An organization’s unique attitude towards risk-
contents of a release are managed, tested and taking that in turn dictates the amount of risk
deployed as a single entity. See also handover. that it considers acceptable.

344 PRINCE2® Practitioner | Copyright© PeopleCert International Ltd. PRINCE2® Practitioner | Copyright© PeopleCert International Ltd. 345
risk response scope tolerance specialist product quality, timescale and at a cost acceptable to
Actions that may be taken to bring a situation The permissible deviation in a plan’s A product whose development is the subject the project board. This role reports to, and
to a level where exposure to risk is acceptable scope that is allowed before the deviation of the plan. The specialist products are takes direction from, the project manager. If
to the organization. These responses fall into a needs to be escalated to the next level of specific to an individual project (for example a team manager is not assigned, the project
number of risk response categories. management. Scope tolerance is documented an advertising campaign, a car park ticketing manager undertakes the responsibilities of the
in the respective plan in the form of a note or system, foundations for a building or a new team manager role.
risk response category reference to the product breakdown structure business process). Also known as a deliverable.
A category of risk response. For threats, the for that plan. See tolerance. See also output. team plan
individual risk response category can be An optional level of plan used as the basis for
to avoid, reduce, transfer, share, accept or Scrum sponsor team management control when executing
prepare contingent plans. For opportunities, An iterative, timeboxed approach to product The main driving force behind a programme work packages.
the individual risk response category can be delivery that is described as ‘a framework within or project. PRINCE2 does not define a role for
to exploit, enhance, transfer, share, accept or which people can address complex adaptive the sponsor, but the sponsor is most likely to theme
prepare contingent plans. problems, while productively and creatively be the executive on the project board, or the An aspect of project management that needs
delivering products of the highest possible person who has appointed the executive. to be continually addressed, and that requires
risk tolerance value’ (Schwaber and Sutherland, 2016). specific treatment for the PRINCE2 processes
The threshold levels of risk exposure that, with sprint to be effective.
appropriate approvals, can be exceeded, but Scrum master A fixed timeframe (typically of 2–4 weeks) for
which when exceeded will trigger some form of A Scrum role that is responsible for ensuring creating selected features from the backlog. threat
response (for example reporting the situation Scrum is understood and enacted and that the An uncertain event that could have a negative
to senior management for action). Scrum team adheres to Scrum theory, practice stage impact on objectives or benefits.
and rules. See management stage.
risk tolerance line time tolerance
A line drawn on the summary risk profile. Risks senior supplier stage plan The permissible deviation in a plan’s time that
Appendix 2

Appendix 2
that appear above this line cannot be accepted The project board role that provides knowledge A detailed plan used as the basis for is allowed before the deviation needs to be
Resources

Resources
(lived with) without referring them to a higher and experience of the main discipline(s) project management control throughout a escalated to the next level of management.
authority. For a project, the project manager involved in the production of the project’s management stage. Time tolerance is documented in the respective
would refer these risks to the project board. deliverable(s). The senior supplier represents plan. See also tolerance.
the supplier’s interests within the project and stakeholder
schedule provides supplier resources. Any individual, group or organization that can time-driven control
A graphical representation of a plan (for affect, be affected by or perceive itself to be A management control that is periodic in
example a Gantt chart), typically describing senior user affected by, an initiative (that is, a programme, nature, to enable the next higher authority to
a sequence of tasks, together with resource The project board role accountable for project, activity or risk). monitor progress (for example a control that
allocations, which collectively deliver the ensuring that user needs are specified correctly takes place every 2 weeks). PRINCE2 offers two
plan. In PRINCE2, project activities should be and that the solution meets those needs. start-up key time-driven progress reports: checkpoint
documented only in the schedules associated The pre-project activities undertaken by the report and highlight report.
with a project plan, stage plan or team plan. share (risk response) executive and the project manager to produce
Actions that are allocated from day-to-day A risk response to either a threat or an the outline business case, project brief and timebox
management may be documented in the opportunity through the application of a pain/ initiation stage plan. A finite period of time when work is carried
relevant project log (that is, risk register, daily gain formula: both parties share the gain out to achieve a goal or meet an objective. The
log, issue register or quality register) if they do (within pre-agreed limits) if the cost is less than supplier deadline should not be moved, as the method
not require significant activity. the cost plan, and both share the pain (again The person, group or groups responsible for of managing a timebox is to prioritize the
within pre-agreed limits) if the cost plan is the supply of the project’s specialist products. work inside it. At a low level a timebox will be a
scope exceeded. matter of days or weeks (for example a sprint).
For a plan, the sum total of its products and the tailoring Higher-level timeboxes act as aggregated
extent of their requirements. It is described by Adapting a method or process to suit the timeboxes and contain lower-level timeboxes
the product breakdown structure for the plan situation in which it will be used. (for example stages).
and associated product descriptions.
team manager
The person responsible for the production of
products allocated by the project manager (as
defined in a work package) to an appropriate

346 PRINCE2® Practitioner | Copyright© PeopleCert International Ltd. PRINCE2® Practitioner | Copyright© PeopleCert International Ltd. 347
tolerance
The permissible deviation above and below
a plan’s target for time and cost without
escalating the deviation to the next level of
management. There may also be tolerance
levels for quality, scope, benefits and risk.
Tolerance is applied at project, management
stage and team levels.

tranche
A programme management term describing a
group of projects structured around distinct
step changes in capability and benefit delivery.

transfer (risk response)


A response to a threat where a third party
takes on responsibility for some of the financial
impact of the threat (for example through
insurance or by means of appropriate clauses
in a contract).

transformation
A distinct change to the way an organization
conducts all or part of its business.
Appendix 2

Appendix 2
Resources

Resources
trigger
An event or decision that triggers a PRINCE2
process to begin.

user
The person or group who will use the project
product.

user acceptance
A specific type of acceptance by the person
or group who will use the product after it
has been handed over into the operational
environment.

user story
A tool used to write a requirement in the form
of who, what and why.

348 PRINCE2® Practitioner | Copyright© PeopleCert International Ltd. PRINCE2® Practitioner | Copyright© PeopleCert International Ltd. 349
Discover more at my.axelos.com

Index
A E procedure, 35 R
acceptance,1124, 129–131, 136, 173 end project report, 35, 173 process, 13, 22, 26–27, 29–30, 36, 41, 47–48, records, 173
acceptance criteria, 129, 173 end stage report, 26, 29, 35, 173 72, 74–75, 80, 92–94, 96–97, 108–109, 116, reports, 47, 92, 132, 138, 161, 173
accountable, 67, 96 exception, 35, 45, 89, 91, 122–123, 155,163– 121, 123–124, 129, 131–132, 136–137, 155, request for change, 139, 147
164, 202–203 172, 183, 185–190, 207, 217
activity, 3, 5, 7, 16–17, 21, 26–27, 29–30, 33, responsible, 2, 32, 42, 90–91, 96, 182
39, 42, 44–45, 47, 53, 65, 80, 89, 92,107–108, exception plan, 35, 122–123, 155 product, 2, 25–30, 32, 35, 40, 43, 45–47, 92,
97–98, 122, 124, 129–130, 132, 136–138, 147, risk, 15, 26, 29, 35, 40–41, 45–46, 65, 98–99,
114, 120, 123, 127, 131, 138, 145, 152, 162, exception report, 35, 45, 123 121, 124, 130, 163, 174, 176–183, 189, 191
172–173, 180, 185–188, 190, 196, 202 152–153, 158–160, 171–173, 183, 190–191,
approval, 47, 89, 92, 98, 122, 131–132, 138 executive, 90–91, 94, 131, 202–203 202–203 risk appetite, 182

assurance, 32, 35, 90–91, 169, 172 F product description, 35, 45, 152–153, 173 risk exposure, 182

authority, 25, 27–28, 30, 148, follow-on action recommendations, 130 product status account, 35 risk management, 35, 98
202–203authorization, 26, 30, 122, 131 H programme, 15, 32, 44, 65, 68, 94, 131 risk management approach, 35, 98
B highlight report, 35 project, 2–3, 6, 13, 15–17, 21, 25–30, 32–33, risk owner, 182
35, 38, 40–48, 65–68, 72, 75–76, 80, 89–94,
baseline, 25, 27–28, 30, 130, 139, 147–148 host site, 124, 130 risk register, 26, 29, 35, 176–178
96–102, 108, 121–124, 129–132, 138, 147–148,
benefit, 14–15, 26, 29, 35, 45, 98–102, 115, initiation stage, 89, 91–92, 94, 155 152–153, 155, 163–164, 172–174, 180, 182– risk response, 26, 29, 174, 182
124, 130, 163, 189, 196 183, 188, 190–191, 202–203, 215, 228
issue, 25–26, 28–29, 35, 45–47, 92, 99, 124, risk tolerance, 182
benefits management approach, 26, 29, 35, 130, 139, 145, 147–148, 183, 189, 191, 216 project approach, 94 S
98, 130
issue register, 35, 139, 148 project brief, 94 schedule, 45, 215, 225
business case, 25, 28, 35, 40, 46–47, 72, 83, 92,
L project lifecycle, 41, 47, 90–92, 148 scope, 45, 163, 189
94, 98–100, 102, 108, 129, 131, 183
lessons log, 35 project management, 2, 13, 15, 17, 32, 35, 41, senior supplier, 202–203
C
log, 35, 225, 227 67, 98, 123, 183, 190, 202–203, 228
change authority, 25, 28, 148 senior user, 25, 28, 147, 202–203
M project management team, 35, 67, 98, 123,
change control, 35, 98, 148 specialist product, 98
183, 190, 202–203
management product, 27, 30, 158–160 stage, 25–30, 35, 38, 41, 45, 47–48, 89–92, 94,
change control approach, 35, 98 project management team structure, 35,
management stage, 45, 122–123, 131, 155, 116, 121–123, 131, 155, 164, 173, 183, 185–
checkpoint, 35, 132, 138, 161 202–203
183, 189–191 191, 202–203
checkpoint report, 35, 132, 138, 161 project manager, 26–27, 29–30, 42, 45–47, 67,
O stage plan, 26, 30, 35, 122–123, 131, 155, 191
89, 91–92, 94, 96, 98, 132, 138, 147, 188, 190,
communication management approach, 35,
off-specification, 139, 147 202–203 stakeholder, 13, 25, 28, 65, 68, 94, 97, 202–
68, 98
203
outcome, 34–36, 44, 101–102 project mandate, 94
configuration item, 35
start-up, 47, 92
output, 35, 44, 101 project plan, 35, 98, 123, 155
configuration item record, 35
supplier, 25, 28, 43, 66–67, 90, 132, 147,202–
P project product, 35, 97, 129–130, 152–153,
constraints, 73, 108 203
173
contingent plan, 174 plan, 20, 25–30, 35, 42, 45–46, 94, 98, 114, T
121–123, 131–132, 138, 149–150, 154–155, project product description, 35, 152–153, 173
corrective action, 144, 183, 189, 191 tailoring, 98
163, 174, 191, 226 project support, 32
customer, 26, 29, 40, 43, 90, 94, 129, 131, 147, team manager, 32, 67, 98, 132, 138, 147,
planned closure, 127 Q
173 202–203
portfolio, 2, 13, 44, 228–229 quality, 2, 25, 28, 35, 45–47, 92, 98, 132,138,
customer’s quality expectations, 173 team plan, 35, 132, 138, 155
premature closure, 131 163, 165, 169, 171–173, 183, 189, 191
Index

Index
D theme, 13, 22, 25–26, 28–29, 35–36, 41, 46,
PRINCE2 principles, 22, 34, 52, 54, 56, 59, 74, quality assurance, 169, 172
daily log, 35 52, 54, 60, 65, 72, 74, 83, 99, 102, 108–109,
109, 217 quality control, 172quality criteria, 47, 92, 173 141–143, 146, 149, 154, 156, 163, 165, 171,
deliverable, 27, 30, 137 174, 181, 217
PRINCE2 project, 13, 32, 41, 45, 131 quality management, 35, 98, 172–173
dis-benefit, 35, 101 threat, 26, 29–30, 40, 174, 182
probability, 174, 182 quality management approach, 35, 98
problem, 139, 147, 216 quality register, 35, 173 time tolerance, 89, 91

352 PRINCE2® Practitioner | Copyright© PeopleCert International Ltd. PRINCE2® Practitioner | Copyright© PeopleCert International Ltd. 353
tolerance, 26–27, 30, 35, 45, 89, 91, 123, 132,
137, 163–164, 182–183, 189–190
trigger, 94
U
user, 25, 28, 66–67, 98, 124, 130, 147, 202–203
user acceptance, 124, 130
V
version, 13, 215
W
work package, 35, 132, 138, 155, 191, 202–203
Index

Index
354 PRINCE2® Practitioner | Copyright© PeopleCert International Ltd. PRINCE2® Practitioner | Copyright© PeopleCert International Ltd. 355
Thank you for
completing the
PRINCE2 Practitioner
course.
Please take a few minutes to give us your
feedback on your experiences and learning
from the course by completing the online
course evaluation survey here.

Powering
Best
Practice

You might also like